[obm-l] Re: [obm-l] Re: [obm-l] Re: [obm-l] Re: [obm-l] Combinatória

2024-03-16 Por tôpico Anderson Torres
Em qua., 13 de mar. de 2024 às 13:07, Claudio Buffara
 escreveu:
>
> Mas este caso tem 7 pessoas. E o enunciado fala em 3 A e 3 C.
>
> On Wed, Mar 13, 2024 at 9:28 AM Pedro Júnior  
> wrote:
>>
>> Eu pensei sim, mas e os casos do tipo ACCACAC. Esse caso não entra na conta 
>> 6! - 2* 3!* 3!.
>>
>> Em qua., 13 de mar. de 2024 às 09:09, Claudio Buffara 
>>  escreveu:
>>>
>>> Pense no oposto: de quantas maneiras as crianças e adultos podem se sentar 
>>> separados uns dos outros.
>>>
>>> On Wed, Mar 13, 2024 at 8:39 AM Pedro Júnior  
>>> wrote:

 Olá pessoal, bom dia.
 Alguém poderia me ajudar nesse problema?

 Seis poltronas enfileiradas em um cinema e entram 3 adultos e 3 crianças. 
 De quantas maneiras podem sentar-se 2 crianças juntas e dois adultos 
 juntos?


Eu pensei numa maneira muito feia: enumeração na raça!

AAA CCC +

AACACC +
AACCAC +
AACCCA +

ACAACC +
ACACAC -
ACACCA -

CAAACC +
CAACAC -
CAACCC +

ACCAAC +
ACCACA -
ACCCAA +

CACAAC -
CACACA -
CACCAA +

CCAAAC +
CCAACA +
CCACAA +

CCC AAA +

20 possibilidades. Apenas 14 servem.
Cada uma nos permite permutar as crianças seis vezes, os adultos outras seis.
6x6x14, que dá meh.


 Desde já fico grato!

 --
 Esta mensagem foi verificada pelo sistema de antivírus e
 acredita-se estar livre de perigo.
>>>
>>>
>>> --
>>> Esta mensagem foi verificada pelo sistema de antivírus e
>>> acredita-se estar livre de perigo.
>>
>>
>>
>> --
>>
>> Pedro Jerônimo S. de O. Júnior
>>
>> Professor de Matemática
>>
>> Geo João Pessoa – PB
>>
>>
>> --
>> Esta mensagem foi verificada pelo sistema de antivírus e
>> acredita-se estar livre de perigo.
>
>
> --
> Esta mensagem foi verificada pelo sistema de antivírus e
> acredita-se estar livre de perigo.

-- 
Esta mensagem foi verificada pelo sistema de antiv�rus e
 acredita-se estar livre de perigo.


=
Instru��es para entrar na lista, sair da lista e usar a lista em
http://www.mat.puc-rio.br/~obmlistas/obm-l.html
=


[obm-l] Re: [obm-l] Re: [obm-l] Re: [obm-l] Combinatória

2024-03-13 Por tôpico Claudio Buffara
Mas este caso tem 7 pessoas. E o enunciado fala em 3 A e 3 C.

On Wed, Mar 13, 2024 at 9:28 AM Pedro Júnior 
wrote:

> Eu pensei sim, mas e os casos do tipo ACCACAC. Esse caso não entra na
> conta 6! - 2* 3!* 3!.
>
> Em qua., 13 de mar. de 2024 às 09:09, Claudio Buffara <
> claudio.buff...@gmail.com> escreveu:
>
>> Pense no oposto: de quantas maneiras as crianças e adultos podem se
>> sentar separados uns dos outros.
>>
>> On Wed, Mar 13, 2024 at 8:39 AM Pedro Júnior 
>> wrote:
>>
>>> Olá pessoal, bom dia.
>>> Alguém poderia me ajudar nesse problema?
>>>
>>> Seis poltronas enfileiradas em um cinema e entram 3 adultos e 3
>>> crianças. De quantas maneiras podem sentar-se 2 crianças juntas e dois
>>> adultos juntos?
>>>
>>>
>>> Desde já fico grato!
>>>
>>> --
>>> Esta mensagem foi verificada pelo sistema de antivírus e
>>> acredita-se estar livre de perigo.
>>
>>
>> --
>> Esta mensagem foi verificada pelo sistema de antivírus e
>> acredita-se estar livre de perigo.
>
>
>
> --
>
> Pedro Jerônimo S. de O. Júnior
>
> Professor de Matemática
>
> Geo João Pessoa – PB
>
> --
> Esta mensagem foi verificada pelo sistema de antivírus e
> acredita-se estar livre de perigo.

-- 
Esta mensagem foi verificada pelo sistema de antiv�rus e
 acredita-se estar livre de perigo.



[obm-l] Re: [obm-l] Re: [obm-l] Re: [obm-l] Pesagens ( Balança Eletrônica)

2023-11-19 Por tôpico Claudio Buffara
Por que você não começa com um caso menor, tal como 4, 6 ou 9 moedas no
total?

Como você não consegue distinguir, numa dada pesagem, um grupo só com
moedas verdadeiras e um grupo com 2 moedas falsas, um algoritmo
pra resolver este problema com o menor número possível de pesagens não me
parece óbvio.  Daí, a análise de um caso menor pode dar alguma luz.

[]s,
Claudio.


On Sun, Nov 19, 2023 at 3:50 PM Jeferson Almir 
wrote:

> Eu tinha errado umas contas, mas sua cota está correta Ralph, preciso
> montar um exemplo com 21 pesagens
>
> Em dom., 19 de nov. de 2023 às 15:00, Jeferson Almir <
> jefersonram...@gmail.com> escreveu:
>
>> Pelo visto, está sim Ralph!! Agora temos que montar uma estratégia que
>> com 21 pesagens.
>>
>> Em dom., 19 de nov. de 2023 às 13:55, Ralph Costa Teixeira <
>> ralp...@gmail.com> escreveu:
>>
>>> Existem 2022*2021/2 possibilidades para as 2 falsas. Qualquer estratégia
>>> que seja criada com k pesagens que dão apenas 2 respostas cada distingue no
>>> máximo dentre 2^k possibilidades. Então devemos ter 2^k >= 2022*2021/2...
>>> hmm, isso daria k como pelo menos 21? Errei algo?
>>>
>>> On Sun, Nov 19, 2023, 12:16 Jeferson Almir 
>>> wrote:
>>>
 Amigos, encontrei como K mínimo o valor 11 mas desconfio que seja
 menos. Se alguém souber uma ideia que acabe o problema serei grato.


 Em Villa Par todas as moedas autênticas pesam uma quantidade par de
 gramas e todas as moedas falsas pesam uma quantidade impar de gramas.

 Se você tiver 2022 moedas entre as quais sabe que exatamente 2 são
 falsas.

 Se tiver uma balança eletrônica que informe apenas se o peso total dos
 objetos colocados nela é par ou impar.

 Determine o valor mínimo de k para qualquer estratégia que permita
 identificar as moedas falsas usando a balança  no máximo k vezes.

 --
 Esta mensagem foi verificada pelo sistema de antivírus e
 acredita-se estar livre de perigo.
>>>
>>>
>>> --
>>> Esta mensagem foi verificada pelo sistema de antivírus e
>>> acredita-se estar livre de perigo.
>>
>>
> --
> Esta mensagem foi verificada pelo sistema de antivírus e
> acredita-se estar livre de perigo.

-- 
Esta mensagem foi verificada pelo sistema de antiv�rus e
 acredita-se estar livre de perigo.



[obm-l] Re: [obm-l] Re: [obm-l] Re: [obm-l] Comunicação

2023-08-28 Por tôpico Tiago Sandino
https://t.me/+jz8XW7bgRqNlOTg5

Criei esse grupo no telegram. A principal vantagem do Telegram em relação
ao Whatsapp é que quem entra pode ter acesso a todas as mensagens e
arquivos anteriores. A quantidade de membros que podem entrar é de 200.000.
Tô pensando aqui em umas regras também, tais como proibição de pirataria,
spam e algumas coisas a mais que podemos discutir lá. Melhor que isso, só
se alguém fizesse um fórum e fosse possível escrever em latex lá. Tem o
AOPS ok, mas nada nosso mesmo. Fiquem à vontade para entrar. Se o link
expirar, podem me solicitar outro por aqui.

Att.
*Prof. Tiago Sandino*
*85 999134896*

Em seg., 28 de ago. de 2023 02:02, Rogerio Ponce 
escreveu:

> Ola pessoal!
>
> Nesta lista, da qual participamos, qualquer um (mesmo que não esteja
> inscrito na lista) pode acessar os arquivos, fazer pesquisas e ler
> todos os problemas e suas solucoes.
>
> No whatsapp, isto seria impossivel, a menos que o individuo ja
> estivesse participando desde o inicio.
>
> []'s
> Rogerio Ponce
>
> On Sat, Aug 26, 2023 at 1:36 PM Esdras Muniz 
> wrote:
> >
> > Seria muito legal se existisse.
> >
> > Em sex, 25 de ago de 2023 18:24, Priscila Santana <
> priscila@hotmail.com> escreveu:
> >>
> >>
> >> Olá!
> >>
> >> Existe algum grupo de discussão de questões olímpicas no WhatsApp?
> >>
> >> Atte.
> >>
> >> Priscila S. da Paz
> >>
> >>
> >>
> >> --
> >> Esta mensagem foi verificada pelo sistema de antivírus e
> >> acredita-se estar livre de perigo.
> >
> >
> > --
> > Esta mensagem foi verificada pelo sistema de antivírus e
> > acredita-se estar livre de perigo.
>
> --
> Esta mensagem foi verificada pelo sistema de antivírus e
>  acredita-se estar livre de perigo.
>
>
> =
> Instru�ões para entrar na lista, sair da lista e usar a lista em
> http://www.mat.puc-rio.br/~obmlistas/obm-l.html
> =
>

-- 
Esta mensagem foi verificada pelo sistema de antiv�rus e
 acredita-se estar livre de perigo.



[obm-l] Re: [obm-l] Re: [obm-l] Re: teoria dos números

2023-08-13 Por tôpico Israel Meireles Chrisostomo
Muito obrigado!

Em qui., 10 de ago. de 2023 22:27, Ian Barquette <
ianbarquettelou...@gmail.com> escreveu:

> Se a função já está definida, e você quer apenas pontuar os limites dela,
> seria o conceito de imagem da função:
>
> Im(f) = (0, 1) = ]0, 1[
>
>
>
> Caso a função não esteja definida, a restrição seria o contradomínio da
> função:
>
> CD(f) = (0, 1) = ]0, 1[
>
> Ao definir a função, considerando C um conjunto qualquer,
>
> f: C -> (0, 1)
>
> Em qui., 10 de ago. de 2023 20:11, Israel Meireles Chrisostomo <
> israelmchrisost...@gmail.com> escreveu:
>
>> Como faço para definir em notação de conjuntos uma função com a
>> restrição, tipo 0> Seria (0,1]x(0,1]?
>>
>> Em qui., 10 de ago. de 2023 às 20:15, Israel Meireles Chrisostomo <
>> israelmchrisost...@gmail.com> escreveu:
>>
>>> Como faço para definir em notação de conjuntos uma função com a
>>> restrição, tipo f(x)<1
>>> Seria (0,1]x(0,1]?
>>>
>>> --
>>> Israel Meireles Chrisostomo
>>>
>>
>>
>> --
>> Israel Meireles Chrisostomo
>>
>> --
>> Esta mensagem foi verificada pelo sistema de antivírus e
>> acredita-se estar livre de perigo.
>
>
> --
> Esta mensagem foi verificada pelo sistema de antivírus e
> acredita-se estar livre de perigo.

-- 
Esta mensagem foi verificada pelo sistema de antiv�rus e
 acredita-se estar livre de perigo.



[obm-l] Re: [obm-l] Re: [obm-l] Re: [obm-l] cadeira de 3 pés

2023-01-23 Por tôpico Bernardo Freitas Paulo da Costa
On Mon, Jan 23, 2023 at 12:52 PM Claudio Buffara
 wrote:
>
> Obrigado, Wagner e Ponce:
>
> Eu tinha pensado em algo na linha do que o Ponce escreveu, ainda que em 
> certos casos patológicos (pelo menos de piso...) o terceiro pé pode não 
> encontrar apoio: imagine um piso com um pico fino em algum lugar (p.ex. a 
> superfície gerada pela revolução de z = 1/(1+(x^2+y^2)^20) em torno do eixo 
> z). Neste caso, precisaríamos de uma cadeira bem pequena, ou pelo menos com 
> as pontas dos pés bem próximas umas das outras.

Pois é, eu acho que tem uma questão interessante de estabilidade que é
a seguinte: se você pegar um banco de 3 pés e botar na encosta de uma
montanha (bem íngreme!), ele vai rolar ladeira abaixo.  Porque a
projeção do centro de gravidade cai fora do triângulo determinado pelo
contato com o solo (que define as normais que podem segurar a cadeira
em posição estática).  Uma cadeira de 4 pés só é bamba porque uma
pessoa não consegue ficar (infinitesimalmente) parada, e daí o centro
de gravidade oscila entre os triângulos formados por dois pares de
três apoios, por exemplo ABC e ACD.

Bernardo

-- 
Esta mensagem foi verificada pelo sistema de antiv�rus e
 acredita-se estar livre de perigo.


=
Instru��es para entrar na lista, sair da lista e usar a lista em
http://www.mat.puc-rio.br/~obmlistas/obm-l.html
=


[obm-l] Re: [obm-l] Re: [obm-l] Re: [obm-l] cadeira de 3 pés

2023-01-23 Por tôpico Anderson Torres
Em seg, 23 de jan de 2023 11:15, Claudio Buffara 
escreveu:

> Será que o argumento usando apenas o plano é suficiente?  Pois um banco de
> 3 pés também fica estável num piso irregular.
>

Mas nem toda cadeira de quatro pés fica estável em qualquer piso irregular.

A ideia subjacente ainda é a de "planidade". Os pés da cadeira estão na
intersecção entre o "plano dos pés" e o piso.
Mas o mesmo não se aplica às cadeiras quadrúpedes, pois nem sempre existe
um "plano dos pés".

Se bem que neste último caso é possível que os pés da mesa sejam coplanares
mas a superfície não o seja. Ou pior ainda, forçando um pouco na
continuidade, pode ser que qualquer cadeira de quatro pés se encaixe em
qualquer superfície não-plana.

Ou não. Se pegarmos por exemplo o mapa topográfico de uma planície com um
poço escavado e a distância entre dois pés da mesa sempre for maior que o
diâmetro do poço, não tem como encaixar uma mesa torta de quatro pés.

Estranho...

O resultado mais geral em que pensei foi o seguinte: dada qualquer
> superfície bi-dimensional contínua (por exemplo, que seja o gráfico de uma
> função contínua de RxR em R - uma suposição razoável se estamos tentando
> modelar um piso), você sempre consegue encostar nela as pontas dos 3 pés do
> banco, de modo que o banco fique "firme" ou sem folgas.
> Ou, mais formalmente, dado um triângulo ABC no espaço, existe uma
> isometria (do espaço) tal que as imagens de A, B e C por esta isometria
> estão em S.
>

Pensei algo do gênero, mas de maneira bem menos elaborada: dados um
triângulo ABC e dois pontos A',B' em uma superfície contínua que distam AB,
existe um ponto C´ tal que ABC = A'B'C'


> Enfim, esse talvez seja um problema mais de topologia do que de geometria.
> Pois, no fim das contas, "3 pontos não colineares determinam um único
> plano", assim como "2 pontos determinam uma única reta" são afirmações que
> têm um certo ar topológico, pelo menos pra mim.
>

> []s,
> Claudio.
>
> On Mon, Jan 23, 2023 at 7:02 AM Anderson Torres <
> torres.anderson...@gmail.com> wrote:
>
>>
>>
>> Em dom, 22 de jan de 2023 23:23, Claudio Buffara <
>> claudio.buff...@gmail.com> escreveu:
>>
>>> Achei na internet duas explicações distintas para a estabilidade de uma
>>> cadeira (ou mesa ou banco) de 3 pés.
>>> Aqui estão:
>>> https://www.somatematica.com.br/curiosidades/c98.php
>>>
>>> http://colegiofarroupilha.com.br/site/qual-cadeira-e-mais-firme-a-que-tem-tres-ou-quatro-pes/
>>>
>>
>> Nesse caso específico, a primeira me parece mais correta. Ou melhor, a
>> segunda tem falhas.
>>
>> A rigidez dos triângulos (TCC caso LLL de igualdade de triângulos) é
>> irrelevante para a questão da cadeira bamba. Em qualquer cadeira física e
>> palpável, as pernas são rígidas - portanto o polígono formado pelas
>> extremidades dessas pernas é rígido também.
>>
>> Mas um polígono rígido não é necessariamente um polígono bidimensional -
>> o que a cadeira bamba de 4 pernas exemplifica perfeitamente.
>>
>> Contate o webmaster da segunda página sugerindo correções!
>>
>>
>>> Qual das duas é a explicação correta?
>>> Ou nenhuma das duas? E, nesse caso, qual a explicação?
>>>
>>> []s,
>>> Claudio.
>>>
>>> --
>>> Esta mensagem foi verificada pelo sistema de antivírus e
>>> acredita-se estar livre de perigo.
>>
>>
>> --
>> Esta mensagem foi verificada pelo sistema de antivírus e
>> acredita-se estar livre de perigo.
>
>
> --
> Esta mensagem foi verificada pelo sistema de antivírus e
> acredita-se estar livre de perigo.

-- 
Esta mensagem foi verificada pelo sistema de antiv�rus e
 acredita-se estar livre de perigo.



[obm-l] Re: [obm-l] Re: [obm-l] Re: [obm-l] Caracterização de Inteiros

2022-11-17 Por tôpico Claudio Buffara
Só completando...

Apesar de números irracionais serem conhecidos desde a época de Pitágoras
(vide a famosa historinha do pitagórico Hipaso, que supostamente foi
afogado por ter "vazado" o segredo da existência dos irracionais), me
parece que eles só começaram a realmente fazer falta no século 19, quando
os matemáticos estavam preocupados em formalizar a análise. Foi naquele
ponto que o Dedekind se deu conta da importância da completeza (essa
palavra tá certa?...) dos reais e da necessidade dela ser garantida por um
axioma. Se não me engano, o teorema que ele queria provar é o de que uma
sequência monótona e limitada de números reais sempre converge. E descobriu
que não conseguiria sem fazer uma hipótese adicional sobre o corpo
dos reais, capaz de diferenciá-lo dos racionais (também um corpo ordenado),
mas dentro do qual nem toda sequência converge (por exemplo, x(n+1) =
(1/2)*(x(n) + 2/x(n)), com x(0) = 1, a qual converge pra raiz(2): um
irracional). A princípio, bastaria ele incluir o axioma de completeza dos
reais e provar que este axioma não contradiz os demais axiomas. Mas ele foi
mais longe e acabou inventando uma construção dos reais a partir dos
racionais (via os chamados "cortes de Dedekind"). Talvez (estou
conjecturando aqui) ele só tenha feito isso por uma necessidade psicológica
de provar que algo é possível (no caso a extensão dos racionais aos reais)
exibindo uma construção explícita. Isso talvez seja um reflexo da ojeriza
que matemáticos da época tinham (e alguns poucos têm até hoje) de
argumentos não construtivos pra provar a existência de fatos ou objetos
matemáticos. Por exemplo, demonstrações usando o princípio das casas de
pombos são desse tipo.

Sobre "entender sem compreender", lembrei de um ditado do John von
Neumann: "Em matemática, você nunca entende as coisas. Apenas se acostuma
com elas."

[]s,
Claudio.

On Wed, Nov 16, 2022 at 6:52 PM Claudio Buffara 
wrote:

> Não entendi como uma homotetia poderia reduzir um par ordenado a um único
> número... enfim...
>
> O que se faz, no caso da relação de equivalência que descrevi, é
> representar o par (a,b) pela notação a-b.
> Daí, (a,b) e (c,d) são equivalentes sss a-b = c-d.
> E a novidade são os números negativos: as classes de equivalência de pares
> (a,b) com a < b, representadas, por exemplo, pelo par (0,c), onde c = b-a.
> Ou, na notação usual, -c.
>
> Mas não acho que se deva perder muito tempo com a construção de sistemas
> numéricos via classes de equivalência, estendendo naturais para inteiros
> para racionais para reais e para complexos.  Até porque é extremamente
> sacal, a cada etapa, checar que as operações usuais (+ e *), quando
> aplicadas aos novos números, têm todas as propriedades que conhecemos da
> escola.
> Essas construções foram a maneira que os matemáticos acharam pra
> formalizar os sistemas numéricos, a partir de conceitos mais básicos (no
> caso, pares ordenados e relações de equivalência) - é o programa do Hilbert
> (ou de Russell e Whitehead), de reduzir toda a matemática à teoria dos
> conjuntos.
> Mas, no fundo, esta é uma construção artificial, ex post.  Pois
> matemáticos já usavam todos os números muito antes dessa formalização ser
> inventada.
> E não acho que ela renda muitos frutos, nem pedagógicos (a menos que seu
> objetivo seja "entender sem compreender") e nem pra ampliação da fronteira
> do conhecimento, exceto colocar os sistemas numéricos numa base axiomática
> sólida.
> Em particular, no que diz respeito aos números reais, a única coisa que
> interessa é que eles são um corpo ordenado completo. Tanto é que vários
> livros de análise partem deste axioma e não se preocupam em construir os
> reais a partir dos naturais.
>
> []s,
> Claudio.
>
>
>
> On Tue, Nov 15, 2022 at 5:07 PM Pedro José  wrote:
>
>> Obrigado a você e ao Cláudio. Mas não sou criativo para inventar. Mas já
>> vi que terei que fazer uma homotetia, para as classes de equivalência para
>> representar só como um número e não como um par, creio eu.
>>
>> Cordialmente,
>> PJMS
>>
>> Em ter., 15 de nov. de 2022 às 16:00, Anderson Torres <
>> torres.anderson...@gmail.com> escreveu:
>>
>>>
>>>
>>> Em ter, 15 de nov de 2022 14:33, Pedro José 
>>> escreveu:
>>>
 Boa tarde!
 Para os |Naturais, temos os postulados de Peano.

 Para os Inteiros há alguma formalização?

>>>
>>> invente uma!
>>>
>>> Pode ser por exemplo o conjunto de pares (p,q) tais que p-q é constante.
>>>
>>> ou melhor (p1,q1)=(p2,q2) se e só se p1+q2=p2+q1.
>>>
>>>
 Acho pobre dizer que é necessário ter outros números devido ao problema
 de fechamento nos naturais para a subtração que é fato e daí introduzir os
 simétricos que são inteiros e ainda não foram caracterizados.

 No meu antigo ginásio aprendi que os Reais era a união dos conjuntos
 disjuntos irracionais e racionais. Os racionais haviam sido bem definidos.
 Aí questionei e o que são irracionais? resposta: são os Reais que não são
 racionais, os que não podem 

[obm-l] Re: [obm-l] Re: [obm-l] Re: [obm-l] Caracterização de Inteiros

2022-11-16 Por tôpico Claudio Buffara
Não entendi como uma homotetia poderia reduzir um par ordenado a um único
número... enfim...

O que se faz, no caso da relação de equivalência que descrevi, é
representar o par (a,b) pela notação a-b.
Daí, (a,b) e (c,d) são equivalentes sss a-b = c-d.
E a novidade são os números negativos: as classes de equivalência de pares
(a,b) com a < b, representadas, por exemplo, pelo par (0,c), onde c = b-a.
Ou, na notação usual, -c.

Mas não acho que se deva perder muito tempo com a construção de sistemas
numéricos via classes de equivalência, estendendo naturais para inteiros
para racionais para reais e para complexos.  Até porque é extremamente
sacal, a cada etapa, checar que as operações usuais (+ e *), quando
aplicadas aos novos números, têm todas as propriedades que conhecemos da
escola.
Essas construções foram a maneira que os matemáticos acharam pra formalizar
os sistemas numéricos, a partir de conceitos mais básicos (no caso, pares
ordenados e relações de equivalência) - é o programa do Hilbert (ou de
Russell e Whitehead), de reduzir toda a matemática à teoria dos conjuntos.
Mas, no fundo, esta é uma construção artificial, ex post.  Pois matemáticos
já usavam todos os números muito antes dessa formalização ser inventada.
E não acho que ela renda muitos frutos, nem pedagógicos (a menos que seu
objetivo seja "entender sem compreender") e nem pra ampliação da fronteira
do conhecimento, exceto colocar os sistemas numéricos numa base axiomática
sólida.
Em particular, no que diz respeito aos números reais, a única coisa que
interessa é que eles são um corpo ordenado completo. Tanto é que vários
livros de análise partem deste axioma e não se preocupam em construir os
reais a partir dos naturais.

[]s,
Claudio.



On Tue, Nov 15, 2022 at 5:07 PM Pedro José  wrote:

> Obrigado a você e ao Cláudio. Mas não sou criativo para inventar. Mas já
> vi que terei que fazer uma homotetia, para as classes de equivalência para
> representar só como um número e não como um par, creio eu.
>
> Cordialmente,
> PJMS
>
> Em ter., 15 de nov. de 2022 às 16:00, Anderson Torres <
> torres.anderson...@gmail.com> escreveu:
>
>>
>>
>> Em ter, 15 de nov de 2022 14:33, Pedro José 
>> escreveu:
>>
>>> Boa tarde!
>>> Para os |Naturais, temos os postulados de Peano.
>>>
>>> Para os Inteiros há alguma formalização?
>>>
>>
>> invente uma!
>>
>> Pode ser por exemplo o conjunto de pares (p,q) tais que p-q é constante.
>>
>> ou melhor (p1,q1)=(p2,q2) se e só se p1+q2=p2+q1.
>>
>>
>>> Acho pobre dizer que é necessário ter outros números devido ao problema
>>> de fechamento nos naturais para a subtração que é fato e daí introduzir os
>>> simétricos que são inteiros e ainda não foram caracterizados.
>>>
>>> No meu antigo ginásio aprendi que os Reais era a união dos conjuntos
>>> disjuntos irracionais e racionais. Os racionais haviam sido bem definidos.
>>> Aí questionei e o que são irracionais? resposta: são os Reais que não são
>>> racionais, os que não podem ser escritos na forma p/q p e q inteiros e
>>> q<>0. Mas me deram um tombo. Definiram os |Reais com base nos irracionais e
>>> os irracionais com base nos |Reais. 3 +2i também não pode ser inscrito na
>>> forma p/q. Só mais tarde no científico, é que meu professor definiu
>>> irracional como um número que não podia ser escrito na forma p/q e cuja
>>> representação decimal tinha uma infinidade de algarismos, sem haver uma
>>> periodicidade.
>>> Na época foi o maior nó que tive com a matemática. O mestre demonstrou
>>> que os racionais eram densos, mas entre eles ainda cabiam os irracionais.
>>> Não satisfeito mostrou que os racionais eram enumeráveis e por absurdo
>>> mostrou que os |Reais não. Não satisfeito mostrou que a cardinalidade do
>>> intervalo [0,1] era maior que a dos |Naturais. Não conseguia conceber que
>>> havia um infinito maior que outro. Outra coisa que demorei a aceitar,mesmo
>>> vendo a bijeção, era que os inteiros e naturais tinham a mesma
>>> cardinalidade. Na minha cabeça, os inteiros têm todos os naturais ainda
>>> sobram os negativos, como é igual?
>>> Hoje, depois de velho, arrumei uma enteada, que muito me pergunta e
>>> estou enrolado. Para dar um ar de superioridade, questionei se conhecia os
>>> inteiros de Gaus, que 5 não era primo nos inteiros de Gaus. Estrepei-me, a
>>> danada foi pesquisar e me questiona sobre o que não tenho um domínio pleno.
>>> Em suma, como apresentei a ela os postulados de Peano para a
>>> caracterização dos Naturais, ela me cobra por algo semelhante para os
>>> Inteiros, e não sei responder.
>>> HELP! SOCORRO! AU SECOURS! AYUDA! AIUTO! HILFE!
>>> Cordialmente,
>>> PJMS
>>>
>>> --
>>> Esta mensagem foi verificada pelo sistema de antivírus e
>>> acredita-se estar livre de perigo.
>>
>>
>> --
>> Esta mensagem foi verificada pelo sistema de antivírus e
>> acredita-se estar livre de perigo.
>
>
> --
> Esta mensagem foi verificada pelo sistema de antivírus e
> acredita-se estar livre de perigo.

-- 
Esta mensagem foi verificada pelo sistema de 

[obm-l] Re: [obm-l] Re: [obm-l] Re: [obm-l] Caracterização de Inteiros

2022-11-15 Por tôpico Anderson Torres
Em ter, 15 de nov de 2022 17:07, Pedro José  escreveu:

> Obrigado a você e ao Cláudio. Mas não sou criativo para inventar. Mas já
> vi que terei que fazer uma homotetia, para as classes de equivalência para
> representar só como um número e não como um par, creio eu.
>

Eu lembro de quando li o Guidorizzi formalizando os reais. Até hoje sinto
que entendo sem compreender, haha!

Por outro lado, números reais (irracionais, no caso) são bem menos
palpáveis que os outros. Dívidas e frações são fáceis de entender, afinal.


> Cordialmente,
> PJMS
>
> Em ter., 15 de nov. de 2022 às 16:00, Anderson Torres <
> torres.anderson...@gmail.com> escreveu:
>
>>
>>
>> Em ter, 15 de nov de 2022 14:33, Pedro José 
>> escreveu:
>>
>>> Boa tarde!
>>> Para os |Naturais, temos os postulados de Peano.
>>>
>>> Para os Inteiros há alguma formalização?
>>>
>>
>> invente uma!
>>
>> Pode ser por exemplo o conjunto de pares (p,q) tais que p-q é constante.
>>
>> ou melhor (p1,q1)=(p2,q2) se e só se p1+q2=p2+q1.
>>
>>
>>> Acho pobre dizer que é necessário ter outros números devido ao problema
>>> de fechamento nos naturais para a subtração que é fato e daí introduzir os
>>> simétricos que são inteiros e ainda não foram caracterizados.
>>>
>>> No meu antigo ginásio aprendi que os Reais era a união dos conjuntos
>>> disjuntos irracionais e racionais. Os racionais haviam sido bem definidos.
>>> Aí questionei e o que são irracionais? resposta: são os Reais que não são
>>> racionais, os que não podem ser escritos na forma p/q p e q inteiros e
>>> q<>0. Mas me deram um tombo. Definiram os |Reais com base nos irracionais e
>>> os irracionais com base nos |Reais. 3 +2i também não pode ser inscrito na
>>> forma p/q. Só mais tarde no científico, é que meu professor definiu
>>> irracional como um número que não podia ser escrito na forma p/q e cuja
>>> representação decimal tinha uma infinidade de algarismos, sem haver uma
>>> periodicidade.
>>> Na época foi o maior nó que tive com a matemática. O mestre demonstrou
>>> que os racionais eram densos, mas entre eles ainda cabiam os irracionais.
>>> Não satisfeito mostrou que os racionais eram enumeráveis e por absurdo
>>> mostrou que os |Reais não. Não satisfeito mostrou que a cardinalidade do
>>> intervalo [0,1] era maior que a dos |Naturais. Não conseguia conceber que
>>> havia um infinito maior que outro. Outra coisa que demorei a aceitar,mesmo
>>> vendo a bijeção, era que os inteiros e naturais tinham a mesma
>>> cardinalidade. Na minha cabeça, os inteiros têm todos os naturais ainda
>>> sobram os negativos, como é igual?
>>> Hoje, depois de velho, arrumei uma enteada, que muito me pergunta e
>>> estou enrolado. Para dar um ar de superioridade, questionei se conhecia os
>>> inteiros de Gaus, que 5 não era primo nos inteiros de Gaus. Estrepei-me, a
>>> danada foi pesquisar e me questiona sobre o que não tenho um domínio pleno.
>>> Em suma, como apresentei a ela os postulados de Peano para a
>>> caracterização dos Naturais, ela me cobra por algo semelhante para os
>>> Inteiros, e não sei responder.
>>> HELP! SOCORRO! AU SECOURS! AYUDA! AIUTO! HILFE!
>>> Cordialmente,
>>> PJMS
>>>
>>> --
>>> Esta mensagem foi verificada pelo sistema de antivírus e
>>> acredita-se estar livre de perigo.
>>
>>
>> --
>> Esta mensagem foi verificada pelo sistema de antivírus e
>> acredita-se estar livre de perigo.
>
>
> --
> Esta mensagem foi verificada pelo sistema de antivírus e
> acredita-se estar livre de perigo.

-- 
Esta mensagem foi verificada pelo sistema de antiv�rus e
 acredita-se estar livre de perigo.



[obm-l] Re: [obm-l] Re: [obm-l] Re: [obm-l] Re: [obm-l] Re: [obm-l] Questão de probabilidade

2022-06-24 Por tôpico Rogerio Ponce
Otima explicacao!
Obrigado, Ralph!

PS: e sim, a provocacao foi pra voce mesmo!
:)

[]'s
Rogerio Ponce


On Wed, Jun 22, 2022 at 1:00 PM Ralph Costa Teixeira  wrote:
>
> Ponce está provocando a gente... senti que esta flecha tinha um bocado a 
> minha direção...  :D :D :D
>
> Olha, tem duas "visões" sobre o que "probabilidade" significa.
>
> A primeira vai na linha de que só podemos falar de probabilidade sobre coisas 
> que ainda não aconteceram. Vai nessa linha: se os eventos estão no passado, 
> então já aconteceram, já estão definidos, e não faz sentido dizer que tinha x 
> de chance de ser assim ou y de ser assado. Se você já jogou a moeda justa, 
> não é mais 50/50 --  é 100% de ser cara, ou 100% de ser coroa, dependendo do 
> que ocorreu. Quem pensa assim vai dizer que dado um certo evento (sempre no 
> futuro), ele tem uma probabilidade dada; se duas pessoas diferentes derem 
> duas probabilidades diferentes para o mesmo evento, uma delas errou.
>
> Outra linha diz que podemos falar de probabilidade sempre que houver 
> incerteza; não interessa o que aconteceu ou o que vai acontecer, o que 
> interessa é o que você SABE sobre o acontecimento. Se você jogou a moeda 
> justa mas eu não sei nada mais sobre o lançamento, continua sendo 50/50 
> **PARA MIM**. Probabilidade passa a ser um conceito sobre INFORMAÇÃO, não 
> sobre os fatos em si (a probabilidade não está na moeda, está no que você 
> sabe sobre a moeda). Quem pensa assim vai dizer que a probabilidade do evento 
> depende não apenas do evento em si, mas da informação que se tem em mãos. 
> Quem pensa assim admite que duas pessoas diferentes podem dar probabilidades 
> diferentes ao mesmo evento SE SOUBEREM FATOS DIFERENTES a respeito do evento, 
> ou seja, probabilidade passa a ser bastante "subjetivo".
>
> Eu talvez tenha descrito mal a primeira interpretação, pois sou ferrenho 
> defensor da segunda. Ela simplesmente engloba a primeira, porque você pode 
> ter informação parcial sobre fatos que ocorrem no futuro. E falar de 
> probabilidade para descrever incerteza presente ou passada é MUITO útil! Eu 
> quero poder expressar incerteza sobre fatos passados com frases do tipo 
> "fulano tem x% de probabilidade de ter cometido tal crime", ou "tem y% de 
> probabilidade de ter petróleo nesse poço", ou "tem z% de chance de eu ter 
> COVID"... Se você tem uma reação negativa a essas frases, lembre o que elas 
> realmente significam (na segunda interpretação): claro que ou o cara cometeu 
> o crime ou não, não faz sentido dizer que ele cometeu o crime x% das vezes em 
> que fizermos um experimento de ele cometer o crime... mas o que aquilo 
> significa é "com a informação que eu tenho, numa escala de 0 a 1, eu tenho 
> x/100 de certeza que fulano cometeu o crime". E "certeza baseada em informaç�!
 �es" é sim quantificável -- e satisfaz exatamente as leis das probabilidades 
com as quais concordamos. "Subjetivo" não significa "posso falar qualquer 
coisa", significa apenas que a conta pode variar de pessoa para pessoa... mas, 
de novo, SE ESSAS PESSOAS TIVEREM INFORMAÇÕES DIFERENTES sobre o evento.
>
> Abraço, Ralph.
>
> On Wed, Jun 22, 2022 at 12:09 PM Rogerio Ponce  wrote:
>>
>> Olá Pedro e pessoal da lista!
>>
>> Segundo a opinião do Pedro, nao faz sentido perguntar qual a probabilidade 
>> de Jose ter conseguido um 6 ao jogar o dado ontem, pois isso ja' aconteceu, 
>> e, portanto, ja' esta' definido.
>>
>> Sera' que e' isso mesmo?
>>
>> []'s
>> Rogerio Ponce
>>
>>
>> On Mon, Jun 20, 2022 at 9:45 PM Pedro José  wrote:
>>>
>>> Eu na minha humilde opinião creio que a probabilidade exista quando pode 
>>> ser uma coisa ou outra. No caso já é definido o que os animais são. Então 
>>> já está tudo errado. A questão seria viável se dessem esses limitantes para 
>>> uma criança que pintaria os desenhos dos animais. Aí sim há probabilidade.
>>>
>>> Em sáb., 18 de jun. de 2022 03:33, Rogerio Ponce  
>>> escreveu:

 Ola' Vanderlei e pessoal da lista!

 Sem perda de generalidade, podemos imaginar que vamos fazer o seguinte:

 - uma pintura preta em um dos caes, escolhido aleatoriamente

 - uma pintura "malhada" em um dos animais, escolhido aleatoriamente entre 
 os 7 animais nao pintados

 - duas pintura pretas, em dois animais, escolhidos aleatoriamente entre os 
 6 animais restantes,

 - quatro pinturas brancas nos 4 animais restantes


 Analisando a afirmacao 04, por exemplo, verificamos que, no segundo passo 
 (pintura malhada) existem 4 opcoes de cachorro e 3 opcoes de gato.

 Assim, a probabilidade de haver um cachorro malhado (4/7) e' maior que a 
 probabilidade de haver um gato malhado (3/7).
 Portanto, a afirmacao 04 esta' correta.
 (e o gabarito esta' errado).

 []'s
 Rogerio Ponce



 On Wed, Mar 16, 2022 at 8:08 AM Professor Vanderlei Nemitz 
  wrote:
>
> Bom dia!
> Na questão a seguir, do vestibular da UEM, 

[obm-l] Re: [obm-l] Re: [obm-l] Re: [obm-l] Re: [obm-l] Questão de probabilidade

2022-06-22 Por tôpico Ralph Costa Teixeira
Ponce está provocando a gente... senti que esta flecha tinha um bocado a
minha direção...  :D :D :D

Olha, tem duas "visões" sobre o que "probabilidade" significa.

A primeira vai na linha de que só podemos falar de probabilidade sobre
coisas que ainda não aconteceram. Vai nessa linha: se os eventos estão no
passado, então já aconteceram, já estão definidos, e não faz sentido dizer
que tinha x de chance de ser assim ou y de ser assado. Se você já jogou a
moeda justa, não é mais 50/50 --  é 100% de ser cara, ou 100% de ser coroa,
dependendo do que ocorreu. Quem pensa assim vai dizer que dado um certo
evento (sempre no futuro), ele tem uma probabilidade dada; se duas pessoas
diferentes derem duas probabilidades diferentes para o mesmo evento, uma
delas errou.

Outra linha diz que podemos falar de probabilidade sempre que houver
incerteza; não interessa o que aconteceu ou o que vai acontecer, o que
interessa é o que você SABE sobre o acontecimento. Se você jogou a moeda
justa mas eu não sei nada mais sobre o lançamento, continua sendo 50/50
**PARA MIM**. Probabilidade passa a ser um conceito sobre INFORMAÇÃO, não
sobre os fatos em si (a probabilidade não está na moeda, está no que você
sabe sobre a moeda). Quem pensa assim vai dizer que a probabilidade do
evento depende não apenas do evento em si, mas da informação que se tem em
mãos. Quem pensa assim admite que duas pessoas diferentes podem dar
probabilidades diferentes ao mesmo evento SE SOUBEREM FATOS DIFERENTES a
respeito do evento, ou seja, probabilidade passa a ser bastante "subjetivo".

Eu talvez tenha descrito mal a primeira interpretação, pois sou ferrenho
defensor da segunda. Ela simplesmente engloba a primeira, porque você pode
ter informação parcial sobre fatos que ocorrem no futuro. E falar de
probabilidade para descrever incerteza presente ou passada é MUITO útil! Eu
quero poder expressar incerteza sobre fatos passados com frases do tipo
"fulano tem x% de probabilidade de ter cometido tal crime", ou "tem y% de
probabilidade de ter petróleo nesse poço", ou "tem z% de chance de eu ter
COVID"... Se você tem uma reação negativa a essas frases, lembre o que elas
realmente significam (na segunda interpretação): claro que ou o cara
cometeu o crime ou não, não faz sentido dizer que ele cometeu o crime x%
das vezes em que fizermos um experimento de ele cometer o crime... mas o
que aquilo significa é "com a informação que eu tenho, numa escala de 0 a
1, eu tenho x/100 de certeza que fulano cometeu o crime". E "certeza
baseada em informações" é sim quantificável -- e satisfaz exatamente as
leis das probabilidades com as quais concordamos. "Subjetivo" não significa
"posso falar qualquer coisa", significa apenas que a conta pode variar de
pessoa para pessoa... mas, de novo, SE ESSAS PESSOAS TIVEREM INFORMAÇÕES
DIFERENTES sobre o evento.

Abraço, Ralph.

On Wed, Jun 22, 2022 at 12:09 PM Rogerio Ponce  wrote:

> Olá Pedro e pessoal da lista!
>
> Segundo a opinião do Pedro, nao faz sentido perguntar qual a probabilidade
> de Jose ter conseguido um 6 ao jogar o dado ontem, pois isso ja' aconteceu,
> e, portanto, ja' esta' definido.
>
> Sera' que e' isso mesmo?
>
> []'s
> Rogerio Ponce
>
>
> On Mon, Jun 20, 2022 at 9:45 PM Pedro José  wrote:
>
>> Eu na minha humilde opinião creio que a probabilidade exista quando pode
>> ser uma coisa ou outra. No caso já é definido o que os animais são. Então
>> já está tudo errado. A questão seria viável se dessem esses limitantes para
>> uma criança que pintaria os desenhos dos animais. Aí sim há probabilidade.
>>
>> Em sáb., 18 de jun. de 2022 03:33, Rogerio Ponce da Silva <
>> abrlw...@gmail.com> escreveu:
>>
>>> Ola' Vanderlei e pessoal da lista!
>>>
>>> Sem perda de generalidade, podemos imaginar que vamos fazer o seguinte:
>>>
>>> - uma pintura preta em um dos caes, escolhido aleatoriamente
>>>
>>> - uma pintura "malhada" em um dos animais, escolhido aleatoriamente
>>> entre os 7 animais nao pintados
>>>
>>> - duas pintura pretas, em dois animais, escolhidos aleatoriamente entre
>>> os 6 animais restantes,
>>>
>>> - quatro pinturas brancas nos 4 animais restantes
>>>
>>>
>>> Analisando a afirmacao 04, por exemplo, verificamos que, no segundo
>>> passo (pintura malhada) existem 4 opcoes de cachorro e 3 opcoes de gato.
>>>
>>> Assim, a probabilidade de haver um cachorro malhado (4/7) e' maior que a
>>> probabilidade de haver um gato malhado (3/7).
>>> Portanto, a afirmacao 04 esta' correta.
>>> (e o gabarito esta' errado).
>>>
>>> []'s
>>> Rogerio Ponce
>>>
>>>
>>>
>>> On Wed, Mar 16, 2022 at 8:08 AM Professor Vanderlei Nemitz <
>>> vanderma...@gmail.com> wrote:
>>>
 Bom dia!
 Na questão a seguir, do vestibular da UEM, penso que o espaço amostral
 tem 105 elementos, pois um cachorro é preto (desconsideramos esse). Porém,
 com esse pensamento, não consigo obter o gabarito, que diz que 02 e 16 são
 corretas.
 Alguém poderia ajudar?
 Muito obrigado!

 *Em um pet shop há 3 gatos e 5 cães. 

[obm-l] Re: [obm-l] Re: [obm-l] Re: [obm-l] Questão de probabilidade

2022-06-22 Por tôpico Rogerio Ponce
Olá Pedro e pessoal da lista!

Segundo a opinião do Pedro, nao faz sentido perguntar qual a probabilidade
de Jose ter conseguido um 6 ao jogar o dado ontem, pois isso ja' aconteceu,
e, portanto, ja' esta' definido.

Sera' que e' isso mesmo?

[]'s
Rogerio Ponce


On Mon, Jun 20, 2022 at 9:45 PM Pedro José  wrote:

> Eu na minha humilde opinião creio que a probabilidade exista quando pode
> ser uma coisa ou outra. No caso já é definido o que os animais são. Então
> já está tudo errado. A questão seria viável se dessem esses limitantes para
> uma criança que pintaria os desenhos dos animais. Aí sim há probabilidade.
>
> Em sáb., 18 de jun. de 2022 03:33, Rogerio Ponce da Silva <
> abrlw...@gmail.com> escreveu:
>
>> Ola' Vanderlei e pessoal da lista!
>>
>> Sem perda de generalidade, podemos imaginar que vamos fazer o seguinte:
>>
>> - uma pintura preta em um dos caes, escolhido aleatoriamente
>>
>> - uma pintura "malhada" em um dos animais, escolhido aleatoriamente entre
>> os 7 animais nao pintados
>>
>> - duas pintura pretas, em dois animais, escolhidos aleatoriamente entre
>> os 6 animais restantes,
>>
>> - quatro pinturas brancas nos 4 animais restantes
>>
>>
>> Analisando a afirmacao 04, por exemplo, verificamos que, no segundo passo
>> (pintura malhada) existem 4 opcoes de cachorro e 3 opcoes de gato.
>>
>> Assim, a probabilidade de haver um cachorro malhado (4/7) e' maior que a
>> probabilidade de haver um gato malhado (3/7).
>> Portanto, a afirmacao 04 esta' correta.
>> (e o gabarito esta' errado).
>>
>> []'s
>> Rogerio Ponce
>>
>>
>>
>> On Wed, Mar 16, 2022 at 8:08 AM Professor Vanderlei Nemitz <
>> vanderma...@gmail.com> wrote:
>>
>>> Bom dia!
>>> Na questão a seguir, do vestibular da UEM, penso que o espaço amostral
>>> tem 105 elementos, pois um cachorro é preto (desconsideramos esse). Porém,
>>> com esse pensamento, não consigo obter o gabarito, que diz que 02 e 16 são
>>> corretas.
>>> Alguém poderia ajudar?
>>> Muito obrigado!
>>>
>>> *Em um pet shop há 3 gatos e 5 cães. Sabemos que 3 desses animais são
>>> pretos, 4 são brancos e 1 é malhado. Além disso, pelo menos 1 cachorro é
>>> preto. Assinale o que for correto. *
>>> *01) A probabilidade de haver exatamente 1 cachorro preto é de 1/6. *
>>> *02) A probabilidade de haver pelo menos 1 gato branco e pelo menos 2
>>> cachorros brancos é de 2/3.*
>>> *04) A probabilidade de haver um cachorro malhado é maior do que a
>>> probabilidade de haver um gato malhado. *
>>> *08) Se um animal for escolhido ao acaso, a probabilidade de ele ser um
>>> cachorro preto é de 1/8. *
>>> *16) Se um animal for escolhido ao acaso, a probabilidade de ele ser um
>>> gato malhado é de 1/16.   *
>>>
>>> --
>>> Esta mensagem foi verificada pelo sistema de antivírus e
>>> acredita-se estar livre de perigo.
>>
>>
>> --
>> Esta mensagem foi verificada pelo sistema de antivírus e
>> acredita-se estar livre de perigo.
>
>
> --
> Esta mensagem foi verificada pelo sistema de antivírus e
> acredita-se estar livre de perigo.

-- 
Esta mensagem foi verificada pelo sistema de antiv�rus e
 acredita-se estar livre de perigo.



[obm-l] Re: [obm-l] Re: [obm-l] Re: [obm-l] Re: matemática discreta

2021-09-20 Por tôpico Anderson Torres
Não consegui entender esse texto.

Em seg., 20 de set. de 2021 às 22:37, Israel Meireles Chrisostomo
 escreveu:
>
> Obrigado
>
> Em seg, 20 de set de 2021 22:00, Israel Meireles Chrisostomo 
>  escreveu:
>>
>> Tome n maior que n
>>
>> Em seg, 20 de set de 2021 20:49, Marcelo Salhab Brogliato 
>>  escreveu:
>>>
>>> Oi Israel,
>>>
>>> Não consegui entender a questão.
>>>
>>> Exemplo:
>>>
>>> n = 10, m = 3, Fib(10 - 3 + 1) = Fib(8) = 21
>>>
>>> (alpha**(2*n)) / (alpha**(n - m)) = alpha**(n + m) = 521.0019193787257
>>>
>>> Pela sua igualdade, alpha**(n + m) deveria ser 1/21, correto?
>>>
>>> Abraços,
>>> Marcelo
>>>
>>> Il giorno lun 20 set 2021 alle ore 15:54 Israel Meireles Chrisostomo 
>>>  ha scritto:

 já tentei de tudo, por favor me ajudem.

 Em seg., 20 de set. de 2021 às 19:39, Israel Meireles Chrisostomo 
  escreveu:
>
> Alguém poderia resolver o problema no link abaixo?
>
> https://mathoverflow.net/questions/404417/alpha2n-fracf-n-m1-alphan-m-1-how-to-prove-that-equality-is-true
>
> --
> Israel Meireles Chrisostomo



 --
 Israel Meireles Chrisostomo

 --
 Esta mensagem foi verificada pelo sistema de antivírus e
 acredita-se estar livre de perigo.
>>>
>>>
>>> --
>>> Esta mensagem foi verificada pelo sistema de antivírus e
>>> acredita-se estar livre de perigo.
>
>
> --
> Esta mensagem foi verificada pelo sistema de antivírus e
> acredita-se estar livre de perigo.

-- 
Esta mensagem foi verificada pelo sistema de antiv�rus e
 acredita-se estar livre de perigo.


=
Instru��es para entrar na lista, sair da lista e usar a lista em
http://www.mat.puc-rio.br/~obmlistas/obm-l.html
=


[obm-l] Re: [obm-l] Re: [obm-l] Re: matemática discreta

2021-09-20 Por tôpico Israel Meireles Chrisostomo
Obrigado

Em seg, 20 de set de 2021 22:00, Israel Meireles Chrisostomo <
israelmchrisost...@gmail.com> escreveu:

> Tome n maior que n
>
> Em seg, 20 de set de 2021 20:49, Marcelo Salhab Brogliato <
> msbro...@gmail.com> escreveu:
>
>> Oi Israel,
>>
>> Não consegui entender a questão.
>>
>> Exemplo:
>>
>> n = 10, m = 3, Fib(10 - 3 + 1) = Fib(8) = 21
>>
>> (alpha**(2*n)) / (alpha**(n - m)) = alpha**(n + m) = 521.0019193787257
>>
>> Pela sua igualdade, alpha**(n + m) deveria ser 1/21, correto?
>>
>> Abraços,
>> Marcelo
>>
>> Il giorno lun 20 set 2021 alle ore 15:54 Israel Meireles Chrisostomo <
>> israelmchrisost...@gmail.com> ha scritto:
>>
>>> já tentei de tudo, por favor me ajudem.
>>>
>>> Em seg., 20 de set. de 2021 às 19:39, Israel Meireles Chrisostomo <
>>> israelmchrisost...@gmail.com> escreveu:
>>>
 Alguém poderia resolver o problema no link abaixo?


 https://mathoverflow.net/questions/404417/alpha2n-fracf-n-m1-alphan-m-1-how-to-prove-that-equality-is-true

 --
 Israel Meireles Chrisostomo

>>>
>>>
>>> --
>>> Israel Meireles Chrisostomo
>>>
>>> --
>>> Esta mensagem foi verificada pelo sistema de antivírus e
>>> acredita-se estar livre de perigo.
>>
>>
>> --
>> Esta mensagem foi verificada pelo sistema de antivírus e
>> acredita-se estar livre de perigo.
>
>

-- 
Esta mensagem foi verificada pelo sistema de antiv�rus e
 acredita-se estar livre de perigo.



[obm-l] Re: [obm-l] Re: [obm-l] Re: matemática discreta

2021-09-20 Por tôpico Israel Meireles Chrisostomo
Tome n maior que n

Em seg, 20 de set de 2021 20:49, Marcelo Salhab Brogliato <
msbro...@gmail.com> escreveu:

> Oi Israel,
>
> Não consegui entender a questão.
>
> Exemplo:
>
> n = 10, m = 3, Fib(10 - 3 + 1) = Fib(8) = 21
>
> (alpha**(2*n)) / (alpha**(n - m)) = alpha**(n + m) = 521.0019193787257
>
> Pela sua igualdade, alpha**(n + m) deveria ser 1/21, correto?
>
> Abraços,
> Marcelo
>
> Il giorno lun 20 set 2021 alle ore 15:54 Israel Meireles Chrisostomo <
> israelmchrisost...@gmail.com> ha scritto:
>
>> já tentei de tudo, por favor me ajudem.
>>
>> Em seg., 20 de set. de 2021 às 19:39, Israel Meireles Chrisostomo <
>> israelmchrisost...@gmail.com> escreveu:
>>
>>> Alguém poderia resolver o problema no link abaixo?
>>>
>>>
>>> https://mathoverflow.net/questions/404417/alpha2n-fracf-n-m1-alphan-m-1-how-to-prove-that-equality-is-true
>>>
>>> --
>>> Israel Meireles Chrisostomo
>>>
>>
>>
>> --
>> Israel Meireles Chrisostomo
>>
>> --
>> Esta mensagem foi verificada pelo sistema de antivírus e
>> acredita-se estar livre de perigo.
>
>
> --
> Esta mensagem foi verificada pelo sistema de antivírus e
> acredita-se estar livre de perigo.

-- 
Esta mensagem foi verificada pelo sistema de antiv�rus e
 acredita-se estar livre de perigo.



[obm-l] Re: [obm-l] Re: [obm-l] Re: [obm-l] Re: [obm-l] Função

2021-05-29 Por tôpico Israel Meireles Chrisostomo
obrigado


Livre
de vírus. www.avast.com
.
<#DAB4FAD8-2DD7-40BB-A1B8-4E2AA1F9FDF2>

Em qua., 19 de mai. de 2021 às 10:56, Anderson Torres <
torres.anderson...@gmail.com> escreveu:

> Em seg., 26 de abr. de 2021 às 17:18, Israel Meireles Chrisostomo
>  escreveu:
> >
> > Mas aí então a+bi e b+ai são os mesmos números
>
> Não são.
>
> 4+5i e 5+4i são diferentes, e 4+5i < 5+4i por essas regras.
>
> >
> > Em seg, 26 de abr de 2021 13:36, Anderson Torres <
> torres.anderson...@gmail.com> escreveu:
> >>
> >> Em qui., 22 de abr. de 2021 às 07:19, Israel Meireles Chrisostomo
> >>  escreveu:
> >> >
> >> > Me desculpem se eu estou falando bobagem, mas considere uma função
> com domínio complexo, então essa função não pode ser bijetora, pois toda
> função bijetora ou é crescente ou é decrescente, mas não há ordem nos
> complexos
>
> Você não entendeu nada aqui, suponho. Primeiramente, funções não são
> coisas limitadas a números.
>
> Segundamente, quando usamos esse teorema de que funções contínuas são
> monótonas, é óbvio que estamos supondo de antemão que estamos
> trabalhando com um sistema numérico que admita a ideia de ordem.
> Especialmente, a de um corpo ordenado completo.
>
> Por exemplo, não faz sentido falar de "continuidade" quando se fala de
> funções de naturais para naturais, porque números naturais não formam
> um sistema numérico contínuo.
>
> >>
> >> Não é correto dizer que não existe ordem nos complexos. É só atribuir
> >> o seguinte: o complexo A é maior que o complexo B se e somente se ou o
> >> módulo de A é maior que o de B ou os módulos são iguais mas o
> >> argumento de A é maior que o de B (tomando este módulo no intervalo de
> >> 0 a tau).
> >>
> >> >
> >> > --
> >> > Esta mensagem foi verificada pelo sistema de antivírus e
> >> > acredita-se estar livre de perigo.
> >>
> >> --
> >> Esta mensagem foi verificada pelo sistema de antivírus e
> >>  acredita-se estar livre de perigo.
> >>
> >>
> >>
> =
> >> Instru�ões para entrar na lista, sair da lista e usar a lista em
> >> http://www.mat.puc-rio.br/~obmlistas/obm-l.html
> >>
> =
> >
> >
> > --
> > Esta mensagem foi verificada pelo sistema de antivírus e
> > acredita-se estar livre de perigo.
>
> --
> Esta mensagem foi verificada pelo sistema de antivírus e
>  acredita-se estar livre de perigo.
>
>
> =
> Instru�ões para entrar na lista, sair da lista e usar a lista em
> http://www.mat.puc-rio.br/~obmlistas/obm-l.html
> =
>


-- 
Israel Meireles Chrisostomo

-- 
Esta mensagem foi verificada pelo sistema de antiv�rus e
 acredita-se estar livre de perigo.



[obm-l] Re: [obm-l] Re: [obm-l] Re: [obm-l] Função

2021-05-19 Por tôpico Anderson Torres
Em seg., 26 de abr. de 2021 às 17:18, Israel Meireles Chrisostomo
 escreveu:
>
> Mas aí então a+bi e b+ai são os mesmos números

Não são.

4+5i e 5+4i são diferentes, e 4+5i < 5+4i por essas regras.

>
> Em seg, 26 de abr de 2021 13:36, Anderson Torres 
>  escreveu:
>>
>> Em qui., 22 de abr. de 2021 às 07:19, Israel Meireles Chrisostomo
>>  escreveu:
>> >
>> > Me desculpem se eu estou falando bobagem, mas considere uma função com 
>> > domínio complexo, então essa função não pode ser bijetora, pois toda 
>> > função bijetora ou é crescente ou é decrescente, mas não há ordem nos 
>> > complexos

Você não entendeu nada aqui, suponho. Primeiramente, funções não são
coisas limitadas a números.

Segundamente, quando usamos esse teorema de que funções contínuas são
monótonas, é óbvio que estamos supondo de antemão que estamos
trabalhando com um sistema numérico que admita a ideia de ordem.
Especialmente, a de um corpo ordenado completo.

Por exemplo, não faz sentido falar de "continuidade" quando se fala de
funções de naturais para naturais, porque números naturais não formam
um sistema numérico contínuo.

>>
>> Não é correto dizer que não existe ordem nos complexos. É só atribuir
>> o seguinte: o complexo A é maior que o complexo B se e somente se ou o
>> módulo de A é maior que o de B ou os módulos são iguais mas o
>> argumento de A é maior que o de B (tomando este módulo no intervalo de
>> 0 a tau).
>>
>> >
>> > --
>> > Esta mensagem foi verificada pelo sistema de antivírus e
>> > acredita-se estar livre de perigo.
>>
>> --
>> Esta mensagem foi verificada pelo sistema de antivírus e
>>  acredita-se estar livre de perigo.
>>
>>
>> =
>> Instru�ões para entrar na lista, sair da lista e usar a lista em
>> http://www.mat.puc-rio.br/~obmlistas/obm-l.html
>> =
>
>
> --
> Esta mensagem foi verificada pelo sistema de antivírus e
> acredita-se estar livre de perigo.

-- 
Esta mensagem foi verificada pelo sistema de antiv�rus e
 acredita-se estar livre de perigo.


=
Instru��es para entrar na lista, sair da lista e usar a lista em
http://www.mat.puc-rio.br/~obmlistas/obm-l.html
=


[obm-l] Re: [obm-l] Re: [obm-l] Re: [obm-l] Re: [obm-l] Re: [obm-l] Re: [obm-l] Re: [obm-l] Re: Equações funcionais

2021-02-16 Por tôpico Anderson Torres
Em ter., 16 de fev. de 2021 às 21:26, joao pedro b menezes
 escreveu:
>
> Eu sei, temos f(-1)= 0, f(0) = 1, e f é bijetora. Após trabalhar a equação 
> que cheguei na expressão:
> f( x + f(x) ) - f( f(x)) = x.  Queria saber se essa identidade, junto com a 
> do enunciado, é suficiente para provar a linearidade de f.
>

Seriosamente, não me parece útil perder tempo provando que isso é
linear. O processo que você levaria provando que f(x)=Ax+B basicamente
se resumiria a finalizar o problema.

Outra identidade que pode ser útil para você é provar que f(f(x)) -
f(x) = f(x) -x.
Essa, junto com a identidade acima que você provou, te deixam em 70%
do problema.

> --
> Esta mensagem foi verificada pelo sistema de antivírus e
> acredita-se estar livre de perigo.

-- 
Esta mensagem foi verificada pelo sistema de antiv�rus e
 acredita-se estar livre de perigo.


=
Instru��es para entrar na lista, sair da lista e usar a lista em
http://www.mat.puc-rio.br/~obmlistas/obm-l.html
=


[obm-l] Re: [obm-l] Re: [obm-l] Re: [obm-l] Re: [obm-l] Re: [obm-l] Re: [obm-l] Re: Equações funcionais

2021-02-16 Por tôpico joao pedro b menezes
Eu sei, temos f(-1)= 0, f(0) = 1, e f é bijetora. Após trabalhar a equação
que cheguei na expressão:
f( x + f(x) ) - f( f(x)) = x.  Queria saber se essa identidade, junto com a
do enunciado, é suficiente para provar a linearidade de f.

-- 
Esta mensagem foi verificada pelo sistema de antiv�rus e
 acredita-se estar livre de perigo.



[obm-l] Re: [obm-l] Re: [obm-l] Re: [obm-l] Re: [obm-l] Re: [obm-l] Re: Equações funcionais

2021-02-16 Por tôpico Anderson Torres
Em ter., 16 de fev. de 2021 às 20:43, joao pedro b menezes
 escreveu:
>
> Foi da OBM 2006, nível 3,  3° fase:
> “Determine todas as funções f: R -> R tais que
> f( xf(y) + f(x) ) = 2f(x) + xy

Isso dá bem mais informação!

Por exemplo essa função é sobrejetora. Afinal, qualquer número pode
ser escrito na forma 2f(x)+xy - faça por exemplo x=1 e y=z-2f(1).

Daí a ideia é resolver as equacoes f(A)=0 e f(B)=1.

> para todos x,y reais”
>
> --
> Esta mensagem foi verificada pelo sistema de antivírus e
> acredita-se estar livre de perigo.

-- 
Esta mensagem foi verificada pelo sistema de antiv�rus e
 acredita-se estar livre de perigo.


=
Instru��es para entrar na lista, sair da lista e usar a lista em
http://www.mat.puc-rio.br/~obmlistas/obm-l.html
=


[obm-l] Re: [obm-l] Re: [obm-l] Re: [obm-l] Re: [obm-l] Sequência Injetiva

2021-02-16 Por tôpico Anderson Torres
Nada como uma bijeção N -> Q para encerrar o dia!

Se pensar nas operacoes INC e REV, podemos usar um algoritmo assim:

- Se o número é maior que 1, usa DEC (inversa de INC)
- Se o número é menor que 1, usa INV
- Se o número é 1, pare

Como demonstrar que este procedimento sempre encerrará em 1, não
importando que número racional começou? Acho que no fundo isso é só
uma maneira de encodar fracoes continuas mesmo.

Em ter., 16 de fev. de 2021 às 20:35, Matheus Secco
 escreveu:
>
> Esse problema caiu na Olimpíada Iberoamericana de 2009 que eu participei. Foi 
> o problema 5 da prova e lá pedia para provar injetividade e sobrejetividade.
>
> Em qua, 17 de fev de 2021 00:16, Anderson Torres 
>  escreveu:
>>
>> Em dom., 14 de fev. de 2021 às 17:20, Claudio Buffara
>>  escreveu:
>> >
>> > Será que essa sequência é sobrejetiva (sobre os racionais positivos)?
>> > Porque como a(2^n) = n+1, ela certamente atinge todos os naturais, de modo 
>> > que é ilimitada, superiormente e inferiormente (já que a(2^n + 1) = 
>> > 1/(n+1) ).
>> > Mesmo que não seja, seria interessante descobrir que racionais positivos 
>> > ela não atinge.
>> > É suficiente provar que todos os racionais entre 0 e 1 são atingidos (no 
>> > caso, pelos termos de ordem ímpar), mas não sei se isso facilita.
>> > Vale uma exploração numérica, talvez com uma planilha.
>>
>>
>> Se eu não errei as contas, acredito que sim. Afinal basta reverter a
>> fracao continua.
>>
>> As operacoes parecem ser bem limitadas, contudo nao e necessario muito
>> mais que isso para gerar um racional qualquer:
>>
>> - Função INC: x -> x+1
>> - Função REV: x -> 1/x
>>
>> Talvez haja algum invariante que permita prever que cada operacao esta
>> fadada a cair em 1
>>
>> >
>> >
>> > Abs,
>> > Claudio.
>> >
>> > Enviado do meu iPhone
>> >
>> > Em 14 de fev. de 2021, à(s) 13:57, Anderson Torres 
>> >  escreveu:
>> >
>> > 
>> >
>> >
>> > Em sáb., 13 de fev. de 2021 às 17:56, Jeferson Almir 
>> >  escreveu:
>> >>
>> >> Amigos, peço ajuda em provar a injetividade dessa sequência que seria 
>> >> uma saída para provar a unica ocorrência do racional que aparece nela. 
>> >> Estou andando em círculos tentando montar uma possível indução.
>> >>
>> >>
>> >> Dado a sequência a_1 = 1 e a_2n = a_n  + 1 e a_2n+1 = 1/a_2n.
>> >>
>> >> Prove que para todo racional positivo que ocorre na sequência, ocorre 
>> >> uma única vez.
>> >>
>> >>
>> >
>> > Acho que e uma boa usar fracao continua aqui.
>> >
>> > Se a_n = [c0; c1, c2, ..., ck], temos entao a_1 = [1] e
>> >
>> > a_2n =Â [(1+c0); c1, c2, ..., ck] (chamemos isso de operacao E)
>> > a_2n+1 = [0; (1+c0), c1, c2, ..., ck] (chamemos isso de operacao O)
>> >
>> >
>> > A partir disso, acredito que a bijecao fica quase obvia, bastando 
>> > formalizar algumas inducoes marotas.
>> >
>> > Primeiramente, nenhuma representacao da forma [...,N,1] vai surgir dai a 
>> > partir de a_2. Isso pode ser demonstrado por inducao mesmo: ck=1 somente 
>> > no caso [1], e depois dele a funcao a_n so modifica o comeco da cadeia, 
>> > nunca o final dela.
>> >
>> > Assim sendo, temos certeza que nao tem como um racional aparecer uma vez 
>> > na forma canonica e outra na forma alternativa. E, por conseguinte, se 
>> > duas fracoes tem comprimentos diferentes, elas devem ser diferentes. E 
>> > fracoes com comprimentos iguais diferem se e somente se pelo menos um dos 
>> > componentes diferir.
>> >
>> > Agora, a funcao recursiva age de duas formas. Uma delas altera o 
>> > comprimento em 1, e a outra mantém. A que altera, só altera 
>> > acrescentando o 0 na cabeceira. A que não altera, incrementa a cabeceira.
>> >
>> > Desta forma, é possível gerar de maneira unica qualquer numero racional 
>> > comecando do 1.
>> >
>> > - Qualquer fracao de comprimento 1 pode ser gerada simplesmente aplicando 
>> > a operacao E tantas vezes quantas forem necessarias. E tambem nao e 
>> > possivel fazer isso de outra maneira, pois a operacao O aumentara o 
>> > comprimento de maneira irreversivel.
>> >
>> > - Dada uma fracao com comprimento K, temos duas sub inducoes para fazer:
>> >
>> > + A fracao tem comprimento K e comeca com 0.
>> >
>> > Â  Entao ela foi gerada por uma operacao O. O elemento que a gerou tinha 
>> > menos componentes, os quais satisfazem a hipotese de inducao.
>> >
>> > + A fracao tem comprimento K e comeca com algo maior que 0.
>> >
>> > Entao ela foi gerada por uma operacao E. A fracao da qual ela foi gerada 
>> > difere unicamente no primeiro elemento, o qual antes era menor. Assim 
>> > sendo, e possivel reduzir isso ate chegar no caso anterior.
>> >
>> > E isso demonstra recursivamente a unicidade e existencia!
>> >
>> >
>> >
>> >> --
>> >> Esta mensagem foi verificada pelo sistema de antivírus e
>> >> acredita-se estar livre de perigo.
>> >
>> >
>> > --
>> > Esta mensagem foi verificada pelo sistema de antivírus e
>> > acredita-se estar livre de perigo.
>> >
>> >
>> > --
>> > Esta mensagem foi verificada pelo sistema de 

[obm-l] Re: [obm-l] Re: [obm-l] Re: [obm-l] Re: [obm-l] Re: Equações funcionais

2021-02-16 Por tôpico joao pedro b menezes
Foi da OBM 2006, nível 3,  3° fase:
“Determine todas as funções f: R -> R tais que
f( xf(y) + f(x) ) = 2f(x) + xy
para todos x,y reais”

-- 
Esta mensagem foi verificada pelo sistema de antiv�rus e
 acredita-se estar livre de perigo.



[obm-l] Re: [obm-l] Re: [obm-l] Re: [obm-l] Sequência Injetiva

2021-02-16 Por tôpico Matheus Secco
Esse problema caiu na Olimpíada Iberoamericana de 2009 que eu participei.
Foi o problema 5 da prova e lá pedia para provar injetividade e
sobrejetividade.

Em qua, 17 de fev de 2021 00:16, Anderson Torres <
torres.anderson...@gmail.com> escreveu:

> Em dom., 14 de fev. de 2021 às 17:20, Claudio Buffara
>  escreveu:
> >
> > Será que essa sequência é sobrejetiva (sobre os racionais positivos)?
> > Porque como a(2^n) = n+1, ela certamente atinge todos os naturais, de
> modo que é ilimitada, superiormente e inferiormente (já que a(2^n + 1) =
> 1/(n+1) ).
> > Mesmo que não seja, seria interessante descobrir que racionais positivos
> ela não atinge.
> > É suficiente provar que todos os racionais entre 0 e 1 são atingidos (no
> caso, pelos termos de ordem ímpar), mas não sei se isso facilita.
> > Vale uma exploração numérica, talvez com uma planilha.
>
>
> Se eu não errei as contas, acredito que sim. Afinal basta reverter a
> fracao continua.
>
> As operacoes parecem ser bem limitadas, contudo nao e necessario muito
> mais que isso para gerar um racional qualquer:
>
> - Função INC: x -> x+1
> - Função REV: x -> 1/x
>
> Talvez haja algum invariante que permita prever que cada operacao esta
> fadada a cair em 1
>
> >
> >
> > Abs,
> > Claudio.
> >
> > Enviado do meu iPhone
> >
> > Em 14 de fev. de 2021, à(s) 13:57, Anderson Torres <
> torres.anderson...@gmail.com> escreveu:
> >
> > 
> >
> >
> > Em sáb., 13 de fev. de 2021 às 17:56, Jeferson Almir <
> jefersonram...@gmail.com> escreveu:
> >>
> >> Amigos, peço ajuda em provar a injetividade dessa sequência que seria
> uma saída para provar a unica ocorrência do racional que aparece nela.
> Estou andando em círculos tentando montar uma possível indução.
> >>
> >>
> >> Dado a sequência a_1 = 1 e a_2n = a_n  + 1 e a_2n+1 = 1/a_2n.
> >>
> >> Prove que para todo racional positivo que ocorre na sequência, ocorre
> uma única vez.
> >>
> >>
> >
> > Acho que e uma boa usar fracao continua aqui.
> >
> > Se a_n = [c0; c1, c2, ..., ck], temos entao a_1 = [1] e
> >
> > a_2n =Â [(1+c0); c1, c2, ..., ck] (chamemos isso de operacao E)
> > a_2n+1 = [0; (1+c0), c1, c2, ..., ck] (chamemos isso de operacao O)
> >
> >
> > A partir disso, acredito que a bijecao fica quase obvia, bastando
> formalizar algumas inducoes marotas.
> >
> > Primeiramente, nenhuma representacao da forma [...,N,1] vai surgir dai a
> partir de a_2. Isso pode ser demonstrado por inducao mesmo: ck=1 somente
> no caso [1], e depois dele a funcao a_n so modifica o comeco da cadeia,
> nunca o final dela.
> >
> > Assim sendo, temos certeza que nao tem como um racional aparecer uma vez
> na forma canonica e outra na forma alternativa. E, por conseguinte, se duas
> fracoes tem comprimentos diferentes, elas devem ser diferentes. E fracoes
> com comprimentos iguais diferem se e somente se pelo menos um dos
> componentes diferir.
> >
> > Agora, a funcao recursiva age de duas formas. Uma delas altera o
> comprimento em 1, e a outra mantém. A que altera, só altera acrescentando
> o 0 na cabeceira. A que não altera, incrementa a cabeceira.
> >
> > Desta forma, é possível gerar de maneira unica qualquer numeroÂ
> racional comecando do 1.
> >
> > - Qualquer fracao de comprimento 1 pode ser gerada simplesmente
> aplicando a operacao E tantas vezes quantas forem necessarias. E tambem
> nao e possivel fazer isso de outra maneira, pois a operacao O aumentara o
> comprimento de maneira irreversivel.
> >
> > - Dada uma fracao com comprimento K, temos duas sub inducoes para fazer:
> >
> > + A fracao tem comprimento K e comeca com 0.
> >
> > Â  Entao ela foi gerada por uma operacao O. O elemento que a gerou tinha
> menos componentes, os quais satisfazem a hipotese de inducao.
> >
> > + A fracao tem comprimento K e comeca com algo maior que 0.
> >
> > Entao ela foi gerada por uma operacao E. A fracao da qual ela foi gerada
> difere unicamente no primeiro elemento, o qual antes era menor. Assim
> sendo, e possivel reduzir isso ate chegar no caso anterior.
> >
> > E isso demonstra recursivamente a unicidade e existencia!
> >
> >
> >
> >> --
> >> Esta mensagem foi verificada pelo sistema de antivírus e
> >> acredita-se estar livre de perigo.
> >
> >
> > --
> > Esta mensagem foi verificada pelo sistema de antivírus e
> > acredita-se estar livre de perigo.
> >
> >
> > --
> > Esta mensagem foi verificada pelo sistema de antivírus e
> > acredita-se estar livre de perigo.
>
> --
> Esta mensagem foi verificada pelo sistema de antivírus e
>  acredita-se estar livre de perigo.
>
>
> =
> Instru�ões para entrar na lista, sair da lista e usar a lista em
> http://www.mat.puc-rio.br/~obmlistas/obm-l.html
> =
>

-- 
Esta mensagem foi verificada pelo sistema de antiv�rus e
 acredita-se estar livre de perigo.



[obm-l] Re: [obm-l] Re: [obm-l] Re: [obm-l] Re: Equações funcionais

2021-02-16 Por tôpico Anderson Torres
Eu gostaria de saber da origem desse problema...

Em dom., 14 de fev. de 2021 às 14:32, joao pedro b menezes <
joaopedrobmene...@gmail.com> escreveu:

> Obrigado pela resposta, mas ainda tenho umas dúvidas. Poderia dar um
> exemplo de tal função ou explicar como construí-la? E se f fosse somente
> injetora, mudaria alguma coisa?
>
>
> --
> Esta mensagem foi verificada pelo sistema de antivírus e
> acredita-se estar livre de perigo.

-- 
Esta mensagem foi verificada pelo sistema de antiv�rus e
 acredita-se estar livre de perigo.



[obm-l] Re: [obm-l] Re: [obm-l] Re: Equações funcionais

2021-02-14 Por tôpico joao pedro b menezes
Obrigado pela resposta, mas ainda tenho umas dúvidas. Poderia dar um
exemplo de tal função ou explicar como construí-la? E se f fosse somente
injetora, mudaria alguma coisa?

-- 
Esta mensagem foi verificada pelo sistema de antiv�rus e
 acredita-se estar livre de perigo.



[obm-l] Re: [obm-l] Re: [obm-l] Re: [obm-l] Re: [obm-l] Sequência Injetiva

2021-02-14 Por tôpico Claudio Buffara
a(1) = 1
a(2n) = a(2n-1) + 1
a(2n+1) = 1/a(2n)

Fazendo a(n) = p(n)/q(n), obtemos duas sequências: p(n) e q(n).
E elas são tais que:
p(1) = q(1) = 1
p(2n) = p(2n-1) + q(2n-1)
q(2n) = q(2n-1)
p(2n+1) = q(2n)
q(2n+1) = p(2n)

Como as sequências começam com 1 e 1, que são primos entre si, e como
mdc(p,q) = mdc(q,p) = mdc(p+q,q), p(n) e q(n) sempre serão primos entre si.

Usando a definição de p e q:
p(2n+1) = q(2n) = q(2n-1) = p(2n-2) = p(2n-3) + q(2n-3) = p(2n-3) + q(2n-2)
= p(2n-3) + p(2n-1)
e
p(2n+2) = p(2n+1) + q(2n+1) = q(2n) + p(2n) = q(2n-1) + p(2n) = p(2n-2) +
p(2n)

Ou seja, os termos p(n) de ordem ímpar e de ordem par realmente formam uma
sequência de Fibonacci.
Os de ordem ímpar começam com p(1) = p(3) = 1 e os de ordem par com p(2) =
2 e p(4) = 3.

[]s,
Claudio.

On Sun, Feb 14, 2021 at 10:03 AM Claudio Buffara 
wrote:

> Ué!  Continua sendo. Só que é outra questão...
>
>
> On Sun, Feb 14, 2021 at 3:34 AM Ralph Costa Teixeira 
> wrote:
>
>> Sim, voce tem razao -- eu achei que era a_2n = a_{2n-1} +1. Que pena, era
>> uma boa questao com Fibonacci. :)
>>
>> On Sun, Feb 14, 2021 at 12:35 AM Claudio Buffara <
>> claudio.buff...@gmail.com> wrote:
>>
>>> Oi, Ralph:
>>>
>>> Eu posso ter entendido errado a definição da sequência, mas achei termos
>>> diferentes dos seus:
>>> 1:  1
>>> 2:  2
>>> 3:  1/2
>>> 4:  3
>>> 5:  1/3
>>> 6:  3/2
>>> 7:  2/3
>>> 8:  4
>>> 9:  1/4
>>> 10:  4/3
>>> 11:  3/4
>>> 12:  5/2
>>> 13:  2/5
>>> 14:  5/3
>>> 15:  3/5
>>> 16:  5
>>> ...
>>>
>>> []s,
>>> Claudio.
>>>
>>>
>>> On Sat, Feb 13, 2021 at 7:59 PM Ralph Costa Teixeira 
>>> wrote:
>>>
 Meio enrolado, vou escrever meio vagamente.

 Eu sugiro olhar primeiro para os caras com indice impar. Sao eles:
 a1=1/1
 a3=1/2
 a5=2/3
 a7=3/5
 a8=5/8
 ...
 Ou seja, mostre que eles sao quocientes de numeros de Fibonacci
 consecutivos (os caras de indice par sao os inversos desses). Agora tem
 varias maneiras de continuar:

 -- Voce pode mostrar que os numeros de Fibonacci consecutivos sao
 primos entre si; portanto cada fracao dessas fica unicamente determinada
 por numerador e denominador, e (como os numeros de Fibonacci formam uma
 sequencia crescente) vao ser distintos entre si;
 -- Se voce nao quiser entrar no merito do Fibonacci, tente mostrar
 (pode ser por inducao) que a3 < a7 < a11 <...>>> a_(4k+1) < ... < a13 < a9 < a5 < 1  (phi ali seria (raiz(5)-1) / 2, acho).

 De qualquer forma, como a_(2n+1)<1, a1=1 e os "a_2n" sao os inversos
 dos "a_2n+1, vao ser todos diferentes.

 Abraco, Ralph.


 On Sat, Feb 13, 2021 at 5:56 PM Jeferson Almir <
 jefersonram...@gmail.com> wrote:

> Amigos, peço ajuda em provar a injetividade dessa sequência que seria
> uma saída para provar a unica ocorrência do racional que aparece nela.
> Estou andando em círculos tentando montar uma possível indução.
>
>
> Dado a sequência a_1 = 1 e a_2n = a_n  + 1 e a_2n+1 = 1/a_2n.
>
> Prove que para todo racional positivo que ocorre na sequência, ocorre
> uma única vez.
>
> --
> Esta mensagem foi verificada pelo sistema de antivírus e
> acredita-se estar livre de perigo.


 --
 Esta mensagem foi verificada pelo sistema de antivírus e
 acredita-se estar livre de perigo.
>>>
>>>
>>> --
>>> Esta mensagem foi verificada pelo sistema de antivírus e
>>> acredita-se estar livre de perigo.
>>
>>
>> --
>> Esta mensagem foi verificada pelo sistema de antivírus e
>> acredita-se estar livre de perigo.
>
>

-- 
Esta mensagem foi verificada pelo sistema de antiv�rus e
 acredita-se estar livre de perigo.



[obm-l] Re: [obm-l] Re: [obm-l] Re: [obm-l] Re: [obm-l] Sequência Injetiva

2021-02-14 Por tôpico Claudio Buffara
Ué!  Continua sendo. Só que é outra questão...


On Sun, Feb 14, 2021 at 3:34 AM Ralph Costa Teixeira 
wrote:

> Sim, voce tem razao -- eu achei que era a_2n = a_{2n-1} +1. Que pena, era
> uma boa questao com Fibonacci. :)
>
> On Sun, Feb 14, 2021 at 12:35 AM Claudio Buffara <
> claudio.buff...@gmail.com> wrote:
>
>> Oi, Ralph:
>>
>> Eu posso ter entendido errado a definição da sequência, mas achei termos
>> diferentes dos seus:
>> 1:  1
>> 2:  2
>> 3:  1/2
>> 4:  3
>> 5:  1/3
>> 6:  3/2
>> 7:  2/3
>> 8:  4
>> 9:  1/4
>> 10:  4/3
>> 11:  3/4
>> 12:  5/2
>> 13:  2/5
>> 14:  5/3
>> 15:  3/5
>> 16:  5
>> ...
>>
>> []s,
>> Claudio.
>>
>>
>> On Sat, Feb 13, 2021 at 7:59 PM Ralph Costa Teixeira 
>> wrote:
>>
>>> Meio enrolado, vou escrever meio vagamente.
>>>
>>> Eu sugiro olhar primeiro para os caras com indice impar. Sao eles:
>>> a1=1/1
>>> a3=1/2
>>> a5=2/3
>>> a7=3/5
>>> a8=5/8
>>> ...
>>> Ou seja, mostre que eles sao quocientes de numeros de Fibonacci
>>> consecutivos (os caras de indice par sao os inversos desses). Agora tem
>>> varias maneiras de continuar:
>>>
>>> -- Voce pode mostrar que os numeros de Fibonacci consecutivos sao primos
>>> entre si; portanto cada fracao dessas fica unicamente determinada por
>>> numerador e denominador, e (como os numeros de Fibonacci formam uma
>>> sequencia crescente) vao ser distintos entre si;
>>> -- Se voce nao quiser entrar no merito do Fibonacci, tente mostrar (pode
>>> ser por inducao) que a3 < a7 < a11 <...>> < ... < a13 < a9 < a5 < 1  (phi ali seria (raiz(5)-1) / 2, acho).
>>>
>>> De qualquer forma, como a_(2n+1)<1, a1=1 e os "a_2n" sao os inversos dos
>>> "a_2n+1, vao ser todos diferentes.
>>>
>>> Abraco, Ralph.
>>>
>>>
>>> On Sat, Feb 13, 2021 at 5:56 PM Jeferson Almir 
>>> wrote:
>>>
 Amigos, peço ajuda em provar a injetividade dessa sequência que seria
 uma saída para provar a unica ocorrência do racional que aparece nela.
 Estou andando em círculos tentando montar uma possível indução.


 Dado a sequência a_1 = 1 e a_2n = a_n  + 1 e a_2n+1 = 1/a_2n.

 Prove que para todo racional positivo que ocorre na sequência, ocorre
 uma única vez.

 --
 Esta mensagem foi verificada pelo sistema de antivírus e
 acredita-se estar livre de perigo.
>>>
>>>
>>> --
>>> Esta mensagem foi verificada pelo sistema de antivírus e
>>> acredita-se estar livre de perigo.
>>
>>
>> --
>> Esta mensagem foi verificada pelo sistema de antivírus e
>> acredita-se estar livre de perigo.
>
>
> --
> Esta mensagem foi verificada pelo sistema de antivírus e
> acredita-se estar livre de perigo.

-- 
Esta mensagem foi verificada pelo sistema de antiv�rus e
 acredita-se estar livre de perigo.



[obm-l] Re: [obm-l] Re: [obm-l] Re: [obm-l] Sequência Injetiva

2021-02-13 Por tôpico Ralph Costa Teixeira
Sim, voce tem razao -- eu achei que era a_2n = a_{2n-1} +1. Que pena, era
uma boa questao com Fibonacci. :)

On Sun, Feb 14, 2021 at 12:35 AM Claudio Buffara 
wrote:

> Oi, Ralph:
>
> Eu posso ter entendido errado a definição da sequência, mas achei termos
> diferentes dos seus:
> 1:  1
> 2:  2
> 3:  1/2
> 4:  3
> 5:  1/3
> 6:  3/2
> 7:  2/3
> 8:  4
> 9:  1/4
> 10:  4/3
> 11:  3/4
> 12:  5/2
> 13:  2/5
> 14:  5/3
> 15:  3/5
> 16:  5
> ...
>
> []s,
> Claudio.
>
>
> On Sat, Feb 13, 2021 at 7:59 PM Ralph Costa Teixeira 
> wrote:
>
>> Meio enrolado, vou escrever meio vagamente.
>>
>> Eu sugiro olhar primeiro para os caras com indice impar. Sao eles:
>> a1=1/1
>> a3=1/2
>> a5=2/3
>> a7=3/5
>> a8=5/8
>> ...
>> Ou seja, mostre que eles sao quocientes de numeros de Fibonacci
>> consecutivos (os caras de indice par sao os inversos desses). Agora tem
>> varias maneiras de continuar:
>>
>> -- Voce pode mostrar que os numeros de Fibonacci consecutivos sao primos
>> entre si; portanto cada fracao dessas fica unicamente determinada por
>> numerador e denominador, e (como os numeros de Fibonacci formam uma
>> sequencia crescente) vao ser distintos entre si;
>> -- Se voce nao quiser entrar no merito do Fibonacci, tente mostrar (pode
>> ser por inducao) que a3 < a7 < a11 <...> < ... < a13 < a9 < a5 < 1  (phi ali seria (raiz(5)-1) / 2, acho).
>>
>> De qualquer forma, como a_(2n+1)<1, a1=1 e os "a_2n" sao os inversos dos
>> "a_2n+1, vao ser todos diferentes.
>>
>> Abraco, Ralph.
>>
>>
>> On Sat, Feb 13, 2021 at 5:56 PM Jeferson Almir 
>> wrote:
>>
>>> Amigos, peço ajuda em provar a injetividade dessa sequência que seria
>>> uma saída para provar a unica ocorrência do racional que aparece nela.
>>> Estou andando em círculos tentando montar uma possível indução.
>>>
>>>
>>> Dado a sequência a_1 = 1 e a_2n = a_n  + 1 e a_2n+1 = 1/a_2n.
>>>
>>> Prove que para todo racional positivo que ocorre na sequência, ocorre
>>> uma única vez.
>>>
>>> --
>>> Esta mensagem foi verificada pelo sistema de antivírus e
>>> acredita-se estar livre de perigo.
>>
>>
>> --
>> Esta mensagem foi verificada pelo sistema de antivírus e
>> acredita-se estar livre de perigo.
>
>
> --
> Esta mensagem foi verificada pelo sistema de antivírus e
> acredita-se estar livre de perigo.

-- 
Esta mensagem foi verificada pelo sistema de antiv�rus e
 acredita-se estar livre de perigo.



[obm-l] Re: [obm-l] Re: [obm-l] Re: [obm-l] Função parte inteira

2021-02-03 Por tôpico Ralph Costa Teixeira
Hm, confere o enunciado - era parte inteira, ou inteiro mais proximo?

On Wed, Feb 3, 2021, 18:39 joao pedro b menezes 
wrote:

> Obrigado pela dica! Honestamente creio que existe um erro nesse problema.
> Fazendo alguns casos na mão é possivel perceber que isso sempre resulta em
> 8n + 7. Essa é a prova:
> "Provar que ( n^(1/3) + ( n + 2)^(1/3) )³  < 8n + 8. Abrindo a potência,
> temos:
> 2n + 2 + 3 * ( (n² ( n + 2))^(1/3) + (n(n + 2)²)^(1/3)) < 8n + 8
>   (n² ( n + 2))^(1/3) + (n(n + 2)²)^(1/3)   < 2n + 2
> Porém temos que  (n² ( n + 2))^(1/3) < n + 2/3  , e  (n(n + 2)²)^(1/3) <
> n + 4/3 ( eu testei elevando ambos os lados ao cubo deu certo) . Isso
> confirma a inequação inicial.
> Agora se 8n + 7 <  ( n^(1/3) + ( n + 2)^(1/3) )³  o exercício acaba. De
> fato, trabalhando a expressão:
>(n² ( n + 2))^(1/3) + (n(n + 2)²)^(1/3)   > 2n + 5/3
> Mas novamente, tem se que  (n² ( n + 2))^(1/3) > n + 1/2 e  (n(n +
> 2)²)^(1/3) > n + 7/6 para qualquer n > 1 ( no caso n =1 basta testar na
> mão). E como 1/2 + 7/6 = 5/3 ,  tem se que ela é verdade, logo:
> 8n + 7 <  ( n^(1/3) + ( n + 2)^(1/3) )³  < 8n + 8 ==> [ ( n^(1/3) + ( n +
> 2)^(1/3) )³ ] = 8n + 7"
> Eu estranhei bastante porque nunca tinha acontecido de um exercicio do
> POTI estar errado.
> obs: Se a minha solução estiver errada de alguma forma, adoraria saber!
>
> On Wed, Feb 3, 2021 at 12:42 PM Ralph Costa Teixeira 
> wrote:
>
>> Sem tempo agora, mas olhando por alto eu aproximaria o que estah dentro
>> do () por 2(n+1)^(1/3), o que levaria imediatamente a 8(n+1). Serah que a
>> parte inteira daquela coisa eh 8(n+1)?
>>
>> Entao eu tentaria abrir os cubos, subtrair 8(n+1), e mostrar que o que
>> sobra eh menor que 1.
>>
>> Serah que funciona?
>>
>> On Wed, Feb 3, 2021 at 10:03 AM joao pedro b menezes <
>> joaopedrobmene...@gmail.com> wrote:
>>
>>> Olá, estava tentando fazer esta questão:
>>>   Prove que [ ( n^(1/3) + (n + 2)^(1/3)  )³] é divisível por 8.
>>> obs: não tinha a tecla de função parte inteira, por isso escolhi [ ]
>>> Se alguém tiver alguma dica ou souber como resolver, ajudaria bastante.
>>>
>>


[obm-l] Re: [obm-l] Re: [obm-l] Re: [obm-l] Ângulos de um triângulo

2020-12-04 Por tôpico Pedro Henrique de Almeida Ursino
Seja x a medida do ângulo DAC (logo DAB mede 48 -x). Por trig Ceva

sin x * sin 18 * sin 54 = sin (48-x) * sin 12 * sin 48.

Pode-se deduzir que sin 54 = (1+ sqrt(5))/4 e sin 18 = (sqrt(5)-1)/4. Logo,
sin 54 * sin 18 = 1/4. Assim, nossa equação fica

sin x / sin (48-x) = 4 * sin 12 * sin 48

usando Werner, temos que 2 *sin 12 * sin 48 = cos(48 -12) - cos (48 + 12) =
cos 36 - cos 60 = cos 36 - 1/2
Desse modo, nossa relação fica

sinx/ sin(48-x) = 2*cos 36 - 1 = 2*sin 18

Daí é muito trivial ver que 18 é solução. Esta solução é única pois f(x) =
sin x/ sin(48-x) é crescente para x entre 0 e 48 (graus).

Essa solução usa muitas relações trigonométricas não tão conhecidas
assim Essa foi uma das soluções dadas pelo prof. Sandro do Canal: A
hora do Bizu em seu último vídeo. Ele também dá uma solução sintética para
o problema. Enfim, vale a pena conferir.

On Fri, Dec 4, 2020 at 4:13 PM Armando Staib 
wrote:

> Não querendo polemizar, mas de acordo com o exercício, é, na minha
> opinião, impossível ser 30 o ângulo pedido  pq se fosse o triângulo DBC
> teria o lado oposto ao ângulo de 18 menor do que o lado oposto ao ângulo de
> 12.
>
> Se me enganei poderiam me mostrar, onde eu errei?
>
> Em sex., 4 de dez. de 2020 às 14:06, Claudio Buffara <
> claudio.buff...@gmail.com> escreveu:
>
>> Aliás, de posse da expressão para BAD e CAD, um exercício razoavelmente
>> fácil de programação (até em planilha), é descobrir para quais triângulos
>> isósceles com ângulos inteiros (em graus) e quais ângulos DBC e DCB
>> inteiros, BAD (e obviamente CAD) também são inteiros.
>>
>> Daí, um problema (não mais um exercício!) é descobrir o padrão por trás
>> destes triângulos especiais.
>>
>> On Fri, Dec 4, 2020 at 1:42 PM Claudio Buffara 
>> wrote:
>>
>>> Usando áreas - em particular, área(ABC) = (1/2)*AB*AC*sen(A) - você
>>> consegue, com alguma facilidade, expressar a tangente de DAC em termos de
>>> senos e cossenos dos ângulos dados.   Daí, é só calcular (com calculadora
>>> ou computador - eu uso Excel ou Wolfram Alpha).  E, de fato, AD divide BAC,
>>> que mede 48 graus, em dois ângulos: um medindo 30 e o outro 18 graus.
>>>
>>> O que não dá é - em 2020 - ficar manipulando aquelas fórmulas de
>>> prostaférese ou identidades trigonométricas obscuras envolvendo ângulos
>>> múltiplos de 3 graus. Isso é coisa do século 19...
>>>
>>> []s,
>>> Claudio.
>>>
>>> On Mon, Nov 30, 2020 at 7:28 PM Professor Vanderlei Nemitz <
>>> vanderma...@gmail.com> wrote:
>>>
 Boa noite!
 Alguém conhece uma saída para o seguinte problema?
 Muito obrigado!

 *Num triângulo isósceles ABC, AB = AC.*
 *Seja D um ponto interno tal que os ângulos DBC, DCB, DBA e DCA medem,
 respectivamente, 12°, 18°, 54° e 48°. *
 *Determine a medida do ângulo DAC.*


 
  Livre
 de vírus. www.avast.com
 .

 <#m_-9147473276317047412_m_6354815198100344298_m_-4747407596740689255_m_4608836649714424769_DAB4FAD8-2DD7-40BB-A1B8-4E2AA1F9FDF2>

>>>


[obm-l] Re: [obm-l] Re: [obm-l] Re: [obm-l] Re: [obm-l] Re: [obm-l] Re: [obm-l] Relação de girard

2020-11-16 Por tôpico Claudio Buffara
Sugestão: proponha pra eles o problema de determinar se é possível atribuir
sinais "+" ou "-" a cada um dos números:
1  2  3  4  5  6  7  8  9  10
de modo que a soma algébrica (com sinal) destes números seja igual a zero.
Isso é um desafio e é razoavelmente lúdico, apesar de envolver conceitos
que uma criança de 8 anos entenderia.

On Sat, Nov 14, 2020 at 4:22 PM Israel Meireles Chrisostomo <
israelmchrisost...@gmail.com> wrote:

> Desculpe é q eu queria propor algo q fosse lúdico, mais um desafio,
> voltada para jovens adolescentes, algo descompromissado, sem muitas
> complicações com formalidades
>
> Em qui, 12 de nov de 2020 09:10, Anderson Torres <
> torres.anderson...@gmail.com> escreveu:
>
>>
>>
>> Em sáb., 7 de nov. de 2020 às 16:44, Israel Meireles Chrisostomo <
>> israelmchrisost...@gmail.com> escreveu:
>>
>>>   o objetivo dessa proposta é recriar o ambiente vivido por Euler na
>>> época.
>>>
>>
>> E naquele tempo eles não usavam indução? Formalização é algo bem recente
>> na matemática.
>>
>> Sua exigência me parece algo tão surreal quanto exigir rigor na geometria
>> do tempo de Euclides.
>>
>>
>>
>>>
>>> Em sáb., 7 de nov. de 2020 às 15:10, Israel Meireles Chrisostomo <
>>> israelmchrisost...@gmail.com> escreveu:
>>>
 Na verdade eu estava elaborando um problema que dependia disso.O
 problema é esse aqui:

 Desafio do ano: resolver o problema da Basiléia sem usar derivadas,
 integrais, série de potências, produto infinito do seno ou cosseno, ou
 mesmo indução ou números complexos.

 Em sáb., 7 de nov. de 2020 às 15:07, Israel Meireles Chrisostomo <
 israelmchrisost...@gmail.com> escreveu:

> Na verdade eu estava elaborando um problema que dependia disso.O
> problema é esse aqui:
>
> Desafio do ano: resolver o problema da Basiléia sem usar derivadas,
> integrais, série de potências, produto infinito do seno ou cosseno, ou
> mesmo indução.
>
> Em sáb., 7 de nov. de 2020 às 14:47, Israel Meireles Chrisostomo <
> israelmchrisost...@gmail.com> escreveu:
>
>> conheço uma que usa o teorema de d'lambert
>>
>> Em sáb., 7 de nov. de 2020 às 12:50, Bernardo Freitas Paulo da Costa <
>> bernardo...@gmail.com> escreveu:
>>
>>> On Thu, Nov 5, 2020 at 9:26 PM Artur Costa Steiner
>>>  wrote:
>>> >
>>> > Para facilitar, suponhamos que o polinômio de grau n P seja
>>> mônico. Sejam z_1, , z_n suas n raízes não necessariamente 
>>> distintas.
>>> Para todo complexo z, temos que
>>> >
>>> > P(z)  = ( z - z_1) (z - z_n)
>>> >
>>> > Desenvolvendo e aplicando o chamado produto de Stevin, vc tem as
>>> relações de Girard.
>>>
>>> Eu não conhecia o produto de Stevin, mas de forma geral quando você
>>> usa "..." tem, muitas vezes, um argumento por indução que está
>>> subentendido.  Pode ser que o produto de Stevin "faça a indução pra
>>> você" (calculando os termos \sum \prod z_i que vão aparecer como
>>> coeficientes dos monômios z^k), mas é "quase" como se você estivesse
>>> empurrando a indução um andar abaixo ;-)
>>>
>>>
>>> Israel: qual a demonstração por indução que você conhece?  E porque
>>> você gostaria de outra??
>>>
>>> Abraços,
>>> --
>>> Bernardo Freitas Paulo da Costa
>>>
>>>
>>> =
>>> Instru�ões para entrar na lista, sair da lista e usar a lista em
>>> http://www.mat.puc-rio.br/~obmlistas/obm-l.html
>>>
>>> =
>>>
>>
>>
>> --
>> Israel Meireles Chrisostomo
>>
>
>
> --
> Israel Meireles Chrisostomo
>


 --
 Israel Meireles Chrisostomo

>>>
>>>
>>> --
>>> Israel Meireles Chrisostomo
>>>
>>


[obm-l] Re: [obm-l] Re: [obm-l] Re: [obm-l] Re: [obm-l] Re: [obm-l] Relação de girard

2020-11-14 Por tôpico Israel Meireles Chrisostomo
Desculpe é q eu queria propor algo q fosse lúdico, mais um desafio,
voltada para jovens adolescentes, algo descompromissado, sem muitas
complicações com formalidades

Em qui, 12 de nov de 2020 09:10, Anderson Torres <
torres.anderson...@gmail.com> escreveu:

>
>
> Em sáb., 7 de nov. de 2020 às 16:44, Israel Meireles Chrisostomo <
> israelmchrisost...@gmail.com> escreveu:
>
>>   o objetivo dessa proposta é recriar o ambiente vivido por Euler na
>> época.
>>
>
> E naquele tempo eles não usavam indução? Formalização é algo bem recente
> na matemática.
>
> Sua exigência me parece algo tão surreal quanto exigir rigor na geometria
> do tempo de Euclides.
>
>
>
>>
>> Em sáb., 7 de nov. de 2020 às 15:10, Israel Meireles Chrisostomo <
>> israelmchrisost...@gmail.com> escreveu:
>>
>>> Na verdade eu estava elaborando um problema que dependia disso.O
>>> problema é esse aqui:
>>>
>>> Desafio do ano: resolver o problema da Basiléia sem usar derivadas,
>>> integrais, série de potências, produto infinito do seno ou cosseno, ou
>>> mesmo indução ou números complexos.
>>>
>>> Em sáb., 7 de nov. de 2020 às 15:07, Israel Meireles Chrisostomo <
>>> israelmchrisost...@gmail.com> escreveu:
>>>
 Na verdade eu estava elaborando um problema que dependia disso.O
 problema é esse aqui:

 Desafio do ano: resolver o problema da Basiléia sem usar derivadas,
 integrais, série de potências, produto infinito do seno ou cosseno, ou
 mesmo indução.

 Em sáb., 7 de nov. de 2020 às 14:47, Israel Meireles Chrisostomo <
 israelmchrisost...@gmail.com> escreveu:

> conheço uma que usa o teorema de d'lambert
>
> Em sáb., 7 de nov. de 2020 às 12:50, Bernardo Freitas Paulo da Costa <
> bernardo...@gmail.com> escreveu:
>
>> On Thu, Nov 5, 2020 at 9:26 PM Artur Costa Steiner
>>  wrote:
>> >
>> > Para facilitar, suponhamos que o polinômio de grau n P seja mônico.
>> Sejam z_1, , z_n suas n raízes não necessariamente distintas. Para 
>> todo
>> complexo z, temos que
>> >
>> > P(z)  = ( z - z_1) (z - z_n)
>> >
>> > Desenvolvendo e aplicando o chamado produto de Stevin, vc tem as
>> relações de Girard.
>>
>> Eu não conhecia o produto de Stevin, mas de forma geral quando você
>> usa "..." tem, muitas vezes, um argumento por indução que está
>> subentendido.  Pode ser que o produto de Stevin "faça a indução pra
>> você" (calculando os termos \sum \prod z_i que vão aparecer como
>> coeficientes dos monômios z^k), mas é "quase" como se você estivesse
>> empurrando a indução um andar abaixo ;-)
>>
>>
>> Israel: qual a demonstração por indução que você conhece?  E porque
>> você gostaria de outra??
>>
>> Abraços,
>> --
>> Bernardo Freitas Paulo da Costa
>>
>>
>> =
>> Instru�ões para entrar na lista, sair da lista e usar a lista em
>> http://www.mat.puc-rio.br/~obmlistas/obm-l.html
>>
>> =
>>
>
>
> --
> Israel Meireles Chrisostomo
>


 --
 Israel Meireles Chrisostomo

>>>
>>>
>>> --
>>> Israel Meireles Chrisostomo
>>>
>>
>>
>> --
>> Israel Meireles Chrisostomo
>>
>


[obm-l] Re: [obm-l] Re: [obm-l] Re: [obm-l] Re: [obm-l] Relação de girard

2020-11-12 Por tôpico Anderson Torres
Em sáb., 7 de nov. de 2020 às 16:44, Israel Meireles Chrisostomo <
israelmchrisost...@gmail.com> escreveu:

>   o objetivo dessa proposta é recriar o ambiente vivido por Euler na
> época.
>

E naquele tempo eles não usavam indução? Formalização é algo bem recente na
matemática.

Sua exigência me parece algo tão surreal quanto exigir rigor na geometria
do tempo de Euclides.



>
> Em sáb., 7 de nov. de 2020 às 15:10, Israel Meireles Chrisostomo <
> israelmchrisost...@gmail.com> escreveu:
>
>> Na verdade eu estava elaborando um problema que dependia disso.O problema
>> é esse aqui:
>>
>> Desafio do ano: resolver o problema da Basiléia sem usar derivadas,
>> integrais, série de potências, produto infinito do seno ou cosseno, ou
>> mesmo indução ou números complexos.
>>
>> Em sáb., 7 de nov. de 2020 às 15:07, Israel Meireles Chrisostomo <
>> israelmchrisost...@gmail.com> escreveu:
>>
>>> Na verdade eu estava elaborando um problema que dependia disso.O
>>> problema é esse aqui:
>>>
>>> Desafio do ano: resolver o problema da Basiléia sem usar derivadas,
>>> integrais, série de potências, produto infinito do seno ou cosseno, ou
>>> mesmo indução.
>>>
>>> Em sáb., 7 de nov. de 2020 às 14:47, Israel Meireles Chrisostomo <
>>> israelmchrisost...@gmail.com> escreveu:
>>>
 conheço uma que usa o teorema de d'lambert

 Em sáb., 7 de nov. de 2020 às 12:50, Bernardo Freitas Paulo da Costa <
 bernardo...@gmail.com> escreveu:

> On Thu, Nov 5, 2020 at 9:26 PM Artur Costa Steiner
>  wrote:
> >
> > Para facilitar, suponhamos que o polinômio de grau n P seja mônico.
> Sejam z_1, , z_n suas n raízes não necessariamente distintas. Para 
> todo
> complexo z, temos que
> >
> > P(z)  = ( z - z_1) (z - z_n)
> >
> > Desenvolvendo e aplicando o chamado produto de Stevin, vc tem as
> relações de Girard.
>
> Eu não conhecia o produto de Stevin, mas de forma geral quando você
> usa "..." tem, muitas vezes, um argumento por indução que está
> subentendido.  Pode ser que o produto de Stevin "faça a indução pra
> você" (calculando os termos \sum \prod z_i que vão aparecer como
> coeficientes dos monômios z^k), mas é "quase" como se você estivesse
> empurrando a indução um andar abaixo ;-)
>
>
> Israel: qual a demonstração por indução que você conhece?  E porque
> você gostaria de outra??
>
> Abraços,
> --
> Bernardo Freitas Paulo da Costa
>
>
> =
> Instru�ões para entrar na lista, sair da lista e usar a lista em
> http://www.mat.puc-rio.br/~obmlistas/obm-l.html
>
> =
>


 --
 Israel Meireles Chrisostomo

>>>
>>>
>>> --
>>> Israel Meireles Chrisostomo
>>>
>>
>>
>> --
>> Israel Meireles Chrisostomo
>>
>
>
> --
> Israel Meireles Chrisostomo
>


[obm-l] Re: [obm-l] Re: [obm-l] Re: [obm-l] Relação de girard

2020-11-07 Por tôpico Israel Meireles Chrisostomo
  o objetivo dessa proposta é recriar o ambiente vivido por Euler na
época.

Em sáb., 7 de nov. de 2020 às 15:10, Israel Meireles Chrisostomo <
israelmchrisost...@gmail.com> escreveu:

> Na verdade eu estava elaborando um problema que dependia disso.O problema
> é esse aqui:
>
> Desafio do ano: resolver o problema da Basiléia sem usar derivadas,
> integrais, série de potências, produto infinito do seno ou cosseno, ou
> mesmo indução ou números complexos.
>
> Em sáb., 7 de nov. de 2020 às 15:07, Israel Meireles Chrisostomo <
> israelmchrisost...@gmail.com> escreveu:
>
>> Na verdade eu estava elaborando um problema que dependia disso.O problema
>> é esse aqui:
>>
>> Desafio do ano: resolver o problema da Basiléia sem usar derivadas,
>> integrais, série de potências, produto infinito do seno ou cosseno, ou
>> mesmo indução.
>>
>> Em sáb., 7 de nov. de 2020 às 14:47, Israel Meireles Chrisostomo <
>> israelmchrisost...@gmail.com> escreveu:
>>
>>> conheço uma que usa o teorema de d'lambert
>>>
>>> Em sáb., 7 de nov. de 2020 às 12:50, Bernardo Freitas Paulo da Costa <
>>> bernardo...@gmail.com> escreveu:
>>>
 On Thu, Nov 5, 2020 at 9:26 PM Artur Costa Steiner
  wrote:
 >
 > Para facilitar, suponhamos que o polinômio de grau n P seja mônico.
 Sejam z_1, , z_n suas n raízes não necessariamente distintas. Para todo
 complexo z, temos que
 >
 > P(z)  = ( z - z_1) (z - z_n)
 >
 > Desenvolvendo e aplicando o chamado produto de Stevin, vc tem as
 relações de Girard.

 Eu não conhecia o produto de Stevin, mas de forma geral quando você
 usa "..." tem, muitas vezes, um argumento por indução que está
 subentendido.  Pode ser que o produto de Stevin "faça a indução pra
 você" (calculando os termos \sum \prod z_i que vão aparecer como
 coeficientes dos monômios z^k), mas é "quase" como se você estivesse
 empurrando a indução um andar abaixo ;-)


 Israel: qual a demonstração por indução que você conhece?  E porque
 você gostaria de outra??

 Abraços,
 --
 Bernardo Freitas Paulo da Costa


 =
 Instru�ões para entrar na lista, sair da lista e usar a lista em
 http://www.mat.puc-rio.br/~obmlistas/obm-l.html

 =

>>>
>>>
>>> --
>>> Israel Meireles Chrisostomo
>>>
>>
>>
>> --
>> Israel Meireles Chrisostomo
>>
>
>
> --
> Israel Meireles Chrisostomo
>


-- 
Israel Meireles Chrisostomo


[obm-l] Re: [obm-l] Re: [obm-l] Re: [obm-l] Relação de girard

2020-11-07 Por tôpico Israel Meireles Chrisostomo
Na verdade eu estava elaborando um problema que dependia disso.O problema é
esse aqui:

Desafio do ano: resolver o problema da Basiléia sem usar derivadas,
integrais, série de potências, produto infinito do seno ou cosseno, ou
mesmo indução ou números complexos.

Em sáb., 7 de nov. de 2020 às 15:07, Israel Meireles Chrisostomo <
israelmchrisost...@gmail.com> escreveu:

> Na verdade eu estava elaborando um problema que dependia disso.O problema
> é esse aqui:
>
> Desafio do ano: resolver o problema da Basiléia sem usar derivadas,
> integrais, série de potências, produto infinito do seno ou cosseno, ou
> mesmo indução.
>
> Em sáb., 7 de nov. de 2020 às 14:47, Israel Meireles Chrisostomo <
> israelmchrisost...@gmail.com> escreveu:
>
>> conheço uma que usa o teorema de d'lambert
>>
>> Em sáb., 7 de nov. de 2020 às 12:50, Bernardo Freitas Paulo da Costa <
>> bernardo...@gmail.com> escreveu:
>>
>>> On Thu, Nov 5, 2020 at 9:26 PM Artur Costa Steiner
>>>  wrote:
>>> >
>>> > Para facilitar, suponhamos que o polinômio de grau n P seja mônico.
>>> Sejam z_1, , z_n suas n raízes não necessariamente distintas. Para todo
>>> complexo z, temos que
>>> >
>>> > P(z)  = ( z - z_1) (z - z_n)
>>> >
>>> > Desenvolvendo e aplicando o chamado produto de Stevin, vc tem as
>>> relações de Girard.
>>>
>>> Eu não conhecia o produto de Stevin, mas de forma geral quando você
>>> usa "..." tem, muitas vezes, um argumento por indução que está
>>> subentendido.  Pode ser que o produto de Stevin "faça a indução pra
>>> você" (calculando os termos \sum \prod z_i que vão aparecer como
>>> coeficientes dos monômios z^k), mas é "quase" como se você estivesse
>>> empurrando a indução um andar abaixo ;-)
>>>
>>>
>>> Israel: qual a demonstração por indução que você conhece?  E porque
>>> você gostaria de outra??
>>>
>>> Abraços,
>>> --
>>> Bernardo Freitas Paulo da Costa
>>>
>>> =
>>> Instru�ões para entrar na lista, sair da lista e usar a lista em
>>> http://www.mat.puc-rio.br/~obmlistas/obm-l.html
>>> =
>>>
>>
>>
>> --
>> Israel Meireles Chrisostomo
>>
>
>
> --
> Israel Meireles Chrisostomo
>


-- 
Israel Meireles Chrisostomo


[obm-l] Re: [obm-l] Re: [obm-l] Re: [obm-l] Relação de girard

2020-11-07 Por tôpico Israel Meireles Chrisostomo
Na verdade eu estava elaborando um problema que dependia disso.O problema é
esse aqui:

Desafio do ano: resolver o problema da Basiléia sem usar derivadas,
integrais, série de potências, produto infinito do seno ou cosseno, ou
mesmo indução.

Em sáb., 7 de nov. de 2020 às 14:47, Israel Meireles Chrisostomo <
israelmchrisost...@gmail.com> escreveu:

> conheço uma que usa o teorema de d'lambert
>
> Em sáb., 7 de nov. de 2020 às 12:50, Bernardo Freitas Paulo da Costa <
> bernardo...@gmail.com> escreveu:
>
>> On Thu, Nov 5, 2020 at 9:26 PM Artur Costa Steiner
>>  wrote:
>> >
>> > Para facilitar, suponhamos que o polinômio de grau n P seja mônico.
>> Sejam z_1, , z_n suas n raízes não necessariamente distintas. Para todo
>> complexo z, temos que
>> >
>> > P(z)  = ( z - z_1) (z - z_n)
>> >
>> > Desenvolvendo e aplicando o chamado produto de Stevin, vc tem as
>> relações de Girard.
>>
>> Eu não conhecia o produto de Stevin, mas de forma geral quando você
>> usa "..." tem, muitas vezes, um argumento por indução que está
>> subentendido.  Pode ser que o produto de Stevin "faça a indução pra
>> você" (calculando os termos \sum \prod z_i que vão aparecer como
>> coeficientes dos monômios z^k), mas é "quase" como se você estivesse
>> empurrando a indução um andar abaixo ;-)
>>
>>
>> Israel: qual a demonstração por indução que você conhece?  E porque
>> você gostaria de outra??
>>
>> Abraços,
>> --
>> Bernardo Freitas Paulo da Costa
>>
>> =
>> Instru�ões para entrar na lista, sair da lista e usar a lista em
>> http://www.mat.puc-rio.br/~obmlistas/obm-l.html
>> =
>>
>
>
> --
> Israel Meireles Chrisostomo
>


-- 
Israel Meireles Chrisostomo


[obm-l] Re: [obm-l] Re: [obm-l] Re: [obm-l] Relação de girard

2020-11-07 Por tôpico Israel Meireles Chrisostomo
conheço uma que usa o teorema de d'lambert

Em sáb., 7 de nov. de 2020 às 12:50, Bernardo Freitas Paulo da Costa <
bernardo...@gmail.com> escreveu:

> On Thu, Nov 5, 2020 at 9:26 PM Artur Costa Steiner
>  wrote:
> >
> > Para facilitar, suponhamos que o polinômio de grau n P seja mônico.
> Sejam z_1, , z_n suas n raízes não necessariamente distintas. Para todo
> complexo z, temos que
> >
> > P(z)  = ( z - z_1) (z - z_n)
> >
> > Desenvolvendo e aplicando o chamado produto de Stevin, vc tem as
> relações de Girard.
>
> Eu não conhecia o produto de Stevin, mas de forma geral quando você
> usa "..." tem, muitas vezes, um argumento por indução que está
> subentendido.  Pode ser que o produto de Stevin "faça a indução pra
> você" (calculando os termos \sum \prod z_i que vão aparecer como
> coeficientes dos monômios z^k), mas é "quase" como se você estivesse
> empurrando a indução um andar abaixo ;-)
>
>
> Israel: qual a demonstração por indução que você conhece?  E porque
> você gostaria de outra??
>
> Abraços,
> --
> Bernardo Freitas Paulo da Costa
>
> =
> Instru�ões para entrar na lista, sair da lista e usar a lista em
> http://www.mat.puc-rio.br/~obmlistas/obm-l.html
> =
>


-- 
Israel Meireles Chrisostomo


[obm-l] Re: [obm-l] Re: [obm-l] Re: [obm-l] Teoria dos Números

2020-10-26 Por tôpico Otávio Araújo
De nada mano.

Em seg, 26 de out de 2020 09:40, joao pedro b menezes <
joaopedrobmene...@gmail.com> escreveu:

> Muito obrigado pela ajuda! Entendi o exercício agora.
>
> Em dom, 25 de out de 2020 às 19:59, Otávio Araújo <
> otavio17.ara...@gmail.com> escreveu:
>
>> Vc resolve essa questão mostrando q p=n^2+n+1.  Se n=1 acabou. Se n>1,Já
>> que p divide n^3-1 e é primo, temos que p divide n-1 ou n^2+n+1. Não
>> podemos ter p dividindo n-1 pois n divide p-1 -> n<= p-1 n-1> Portanto p divide n^2+n+1. Faca n^2+n+1 = kp, k inteiro positivo. Temos que
>> kp=n^2+n+1 é congruente a 1 módulo n. Do enunciado temos p congruente a 1
>> módulo n,  mas p é congruente a 1 módulo n e é diferente de 1(pois é primo)
>> -> p>= n+1 e  k será congruente a 1 módulo n também. Suponha que k>1,
>> k>1 implica k>= n+1 daí kp>=(n+1)^2 > n^2+n+1, contradição. Portanto
>> k=1 e p=n^2+n+1.
>>
>> Em dom, 25 de out de 2020 17:37, joao pedro b menezes <
>> joaopedrobmene...@gmail.com> escreveu:
>>
>>> Olá, boa tarde.
>>> Estou com dúvida nesse exercício:
>>> " Sejam n um inteiro positivo maior que 1 e p um primo positivo tal que
>>> n divide p − 1 e p divide n 3 − 1. Mostre que 4p − 3 ´e um quadrado
>>> perfeito."
>>> Já agradeço pela ajuda e pelo tempo!
>>>
>>>


[obm-l] Re: [obm-l] Re: [obm-l] Re: [obm-l] Re: [obm-l] Ajuda em teoria dos números

2020-10-22 Por tôpico Pedro José
Boa tarde!
Na verdade: 2^a=64; a= 6 e y=12.

Em qui., 22 de out. de 2020 às 11:17, Pedro José 
escreveu:

> Bom dia!
> Recebi esse problema hoje: 615 + x^2 = 2^y., para x,y inteiros Não saberia
> fazer, como não soube resolver esse, acima. Mas devido a solução do colega
> Esdras, pensei:"já vi algo parecido".
> Basta restringir y aos pares.
> Se y é ímpar x^2=2 mod3, absurdo então y é par. Logo y=2a, com a inteiro.
> (2^a + x) (2^a-x)= 615= 1*615=3*205=5*123=15*41 e como a soma dos fatores
> necessita ser uma potência de 2, só serve para 123 e 5.
> Logo 2^y=64 e y=6 e x= 59 ou x=-59.
> Uma resolução levou a outra, não pelo talento nato, mas por aprendizado, o
> que é válido.
> Teve uma feita que estava tentando provar que o triângulo órtico, era o
> triângulo de menor perímetro que poderia ser inscrito em um triângulo
> acutângulo. Tentei por geometria analítica e só levando tinta. Tinha
> desistido. Quando me deparei com um problema que não consegui resolver, mas
> que tinha um caminho para a solução. Quando vi o rebatimento feito, pensei
> está resolvido. O curioso, é que, quando desisti, pesquisei na internet e
> não achei nada. Depois que consegui resolver, achei duas soluções, e
> infelizmente e como esperado, a minha não era novidade, era clássica.
> Obrigado, Esdras! Sem a sua solução, certamente, não teria resolvido essa
> última questão.
>
> Cordialmente,
> PJMS
>
> Em sex., 24 de jul. de 2020 às 12:19, Prof. Douglas Oliveira <
> profdouglaso.del...@gmail.com> escreveu:
>
>> Obrigado Claudio e Esdras, fatoração show
>>
>>
>> Em sex., 24 de jul. de 2020 às 11:12, Esdras Muniz <
>> esdrasmunizm...@gmail.com> escreveu:
>>
>>> Se for solução inteira positiva, acho que só tem 3 e 4. Vc supõe spdg x
>>> maior ou igual a y, vê que y=1 não tem solução e x=y tb não. Daí, x>y>1.
>>> Fatorando a expressão, fica: (xy-8-(x-y))(xy-8+(x-y))=15. Como
>>> (xy-8-(x-y))>(xy-8+(x-y))>-2. Temos que ou (xy-8-(x-y))=1 e (xy-8+(x-y))=15,
>>> o que não tem soluções inteiras positivas, ou (xy-8-(x-y))=3 e 
>>> (xy-8+(x-y))=5,
>>> cujas únicas soluções inteiras são x=4 e y=3.
>>>
>>> Em sex, 24 de jul de 2020 10:36, Claudio Buffara <
>>> claudio.buff...@gmail.com> escreveu:
>>>
 Pelo que entendi, a solução é a porção dessa curva algébrica situada no
 1o quadrante.
 Dá pra fazer isso no Wolfram Alpha, com o comando plot (x*y-7)^2 - x^2
 - y^2 = 0.

 []s,
 Claudio.

 On Fri, Jul 24, 2020 at 9:58 AM Prof. Douglas Oliveira <
 profdouglaso.del...@gmail.com> wrote:

> Preciso de ajuda para encontrar todas as soluções não negativas da
> equação
> (xy-7)^2=x^2+y^2.
>
> Desde já agradeço a ajuda
> Douglas Oliveira
>
> --
> Esta mensagem foi verificada pelo sistema de antivírus e
> acredita-se estar livre de perigo.


 --
 Esta mensagem foi verificada pelo sistema de antivírus e
 acredita-se estar livre de perigo.
>>>
>>>
>>> --
>>> Esta mensagem foi verificada pelo sistema de antivírus e
>>> acredita-se estar livre de perigo.
>>
>>
>> --
>> Esta mensagem foi verificada pelo sistema de antivírus e
>> acredita-se estar livre de perigo.
>
>


[obm-l] Re: [obm-l] Re: [obm-l] Re: [obm-l] Re: [obm-l] Ajuda em teoria dos números

2020-10-22 Por tôpico Pedro José
Bom dia!
Recebi esse problema hoje: 615 + x^2 = 2^y., para x,y inteiros Não saberia
fazer, como não soube resolver esse, acima. Mas devido a solução do colega
Esdras, pensei:"já vi algo parecido".
Basta restringir y aos pares.
Se y é ímpar x^2=2 mod3, absurdo então y é par. Logo y=2a, com a inteiro.
(2^a + x) (2^a-x)= 615= 1*615=3*205=5*123=15*41 e como a soma dos fatores
necessita ser uma potência de 2, só serve para 123 e 5.
Logo 2^y=64 e y=6 e x= 59 ou x=-59.
Uma resolução levou a outra, não pelo talento nato, mas por aprendizado, o
que é válido.
Teve uma feita que estava tentando provar que o triângulo órtico, era o
triângulo de menor perímetro que poderia ser inscrito em um triângulo
acutângulo. Tentei por geometria analítica e só levando tinta. Tinha
desistido. Quando me deparei com um problema que não consegui resolver, mas
que tinha um caminho para a solução. Quando vi o rebatimento feito, pensei
está resolvido. O curioso, é que, quando desisti, pesquisei na internet e
não achei nada. Depois que consegui resolver, achei duas soluções, e
infelizmente e como esperado, a minha não era novidade, era clássica.
Obrigado, Esdras! Sem a sua solução, certamente, não teria resolvido essa
última questão.

Cordialmente,
PJMS

Em sex., 24 de jul. de 2020 às 12:19, Prof. Douglas Oliveira <
profdouglaso.del...@gmail.com> escreveu:

> Obrigado Claudio e Esdras, fatoração show
>
>
> Em sex., 24 de jul. de 2020 às 11:12, Esdras Muniz <
> esdrasmunizm...@gmail.com> escreveu:
>
>> Se for solução inteira positiva, acho que só tem 3 e 4. Vc supõe spdg x
>> maior ou igual a y, vê que y=1 não tem solução e x=y tb não. Daí, x>y>1.
>> Fatorando a expressão, fica: (xy-8-(x-y))(xy-8+(x-y))=15. Como
>> (xy-8-(x-y))>(xy-8+(x-y))>-2. Temos que ou (xy-8-(x-y))=1 e (xy-8+(x-y))=15,
>> o que não tem soluções inteiras positivas, ou (xy-8-(x-y))=3 e 
>> (xy-8+(x-y))=5,
>> cujas únicas soluções inteiras são x=4 e y=3.
>>
>> Em sex, 24 de jul de 2020 10:36, Claudio Buffara <
>> claudio.buff...@gmail.com> escreveu:
>>
>>> Pelo que entendi, a solução é a porção dessa curva algébrica situada no
>>> 1o quadrante.
>>> Dá pra fazer isso no Wolfram Alpha, com o comando plot (x*y-7)^2 - x^2 -
>>> y^2 = 0.
>>>
>>> []s,
>>> Claudio.
>>>
>>> On Fri, Jul 24, 2020 at 9:58 AM Prof. Douglas Oliveira <
>>> profdouglaso.del...@gmail.com> wrote:
>>>
 Preciso de ajuda para encontrar todas as soluções não negativas da
 equação
 (xy-7)^2=x^2+y^2.

 Desde já agradeço a ajuda
 Douglas Oliveira

 --
 Esta mensagem foi verificada pelo sistema de antivírus e
 acredita-se estar livre de perigo.
>>>
>>>
>>> --
>>> Esta mensagem foi verificada pelo sistema de antivírus e
>>> acredita-se estar livre de perigo.
>>
>>
>> --
>> Esta mensagem foi verificada pelo sistema de antivírus e
>> acredita-se estar livre de perigo.
>
>
> --
> Esta mensagem foi verificada pelo sistema de antivírus e
> acredita-se estar livre de perigo.


[obm-l] Re: [obm-l] Re: [obm-l] Re: [obm-l] Sequência das médias ponderadas

2020-08-27 Por tôpico Artur Costa Steiner
Será que isso vale se (a_n) tiver termos negativos? Me parece que sim

Artur

Em qua, 26 de ago de 2020 21:55, Esdras Muniz 
escreveu:

> Dado e>0, existe n0 tq m>=n0 então a-e
> Sn= c+(am+...+an)/(p1+...+pn)
>
> Daí:
>
>
> c+a(pm+...+pn)/(p1+...+pn) -e
> Daí, fixando m e mandando n pro infinito, c vai pra zero e 
> (pm+...+pn)/(p1+...+pn)
> vai pra 1. Então o limite de Sn é a.
>
>
> Em qua, 26 de ago de 2020 20:19, Claudio Buffara <
> claudio.buff...@gmail.com> escreveu:
>
>> Acho que isso tá mal formulado.
>> Por exemplo,quanto é s_3?
>>
>> On Tue, Aug 25, 2020 at 3:49 PM Artur Costa Steiner <
>> artur.costa.stei...@gmail.com> wrote:
>>
>>> Isso me foi dado como verdadeiro, mas ainda não cheguei a uma conclusão.
>>>
>>> Sejam (a_ n) uma sequência de reais positivos e (s_n) a sequência das
>>> médias ponderadas de (a_n,) com relação aos pesos positivos (p_n).
>>> Suponhamos que lim p_n = p, 0 < p < oo, e que a sequência das médias
>>> aritméticas de (a_n) convirja para o real a. Então, s_n --> a.
>>>
>>> Abraços
>>> Artur
>>>
>>> --
>>> Esta mensagem foi verificada pelo sistema de antivírus e
>>> acredita-se estar livre de perigo.
>>
>>
>> --
>> Esta mensagem foi verificada pelo sistema de antivírus e
>> acredita-se estar livre de perigo.
>
>
> --
> Esta mensagem foi verificada pelo sistema de antivírus e
> acredita-se estar livre de perigo.

-- 
Esta mensagem foi verificada pelo sistema de antiv�rus e
 acredita-se estar livre de perigo.



[obm-l] Re: [obm-l] Re: [obm-l] Re: [obm-l] Re: [obm-l] Re: [obm-l] Re: [obm-l] Geometria plana com desigualdade de médias?

2020-08-27 Por tôpico Anderson Torres
Em qua., 26 de ago. de 2020 às 18:29, Pedro José  escreveu:
>
> Boa noite!
> Anderson,
> achei legal a sua visão. Mas não consegui evoluir com nada.
> Todavia, fiquei com uma dúvida. Como x+y é um dos ângulos do triângulo temos 
> a restrição 0 E entendo que tanto para cotg(x) + cot(y) , como para tg(x) + tg(y) ocorrerá 
> um mínimo em x=y=K/2, onde x+y=k,k sendo um constante.
> Não acompanhei a sua dedução d quando um é mínimo o outro é máximo.

Eu não fui muito claro.

Você converteu o problema em "calcule o valor mínimo de cot(x)+cot(y)
com x+y fixo". Isso é essencialmente o mesmo que resolver o problema
"calcule o valor mínimo de tan(a)+tan(b) com a+b fixo" - pois sabendo
resolver um é só usar a mesma solução para x=90-a, y=90-b.

>
> Saudações,
> PJMS
>
> Em qui., 20 de ago. de 2020 às 22:40, Anderson Torres 
>  escreveu:
>>
>> Em qui., 20 de ago. de 2020 às 22:03, Anderson Torres
>>  escreveu:
>> >
>> > Em ter., 18 de ago. de 2020 às 19:51, Pedro José  
>> > escreveu:
>> > >
>> > > Boa noite!
>> > > Cláudio,
>> > > não consegui nada geométrico.
>> > > O máximo que atingi foi:
>> > > a/ha + b/hb + c/hc= [cotg(A1) +cotg (A2)]  + [cotg(B1) +cotg (B2)] + 
>> > > co[tg(C1) +cotg (C2)] com A1 + A2 = A; B1 + B2 + B e C1 + C2 = C.
>> > > Para ser mínimo cada termo entre colchetes deve ser mínimo, o que ocorre 
>> > > quando A1 = A2; B1 = B2 e C1 = C2. Logo P seria o encontro das 
>> > > bissetrizes e logo I.
>> > > Onde: A1= PAB e A2=PAC; B1=PBA e B2=PBC; C1=PCA e C2=PCB.
>> >
>> > Acho que daqui poderia sair uma interpretação mais escamoteada.
>> > Afinal, trigonometria é uma espécie de "ponto de contato" entre a
>> > geometria analítica e a sintética, entre a nuvem de desenhos e a de
>> > números.
>> >
>> > Acredito que a solução aqui seria arranjar uma interpretação
>> > geométrica desses colchetes de co-tangentes. Acredito que possamos
>> > apelar para Ptolomeu em algum momento ou para um macete de
>> > semelhanças, pois as projeções de um ponto sobre duas retas criam um
>> > quadrilátero cíclico.
>>
>> Acrescentando mais coisas: se queremos minimizar cot(x) +cot(y) com
>> x+y fixo, isto é equivalente a minimizar tan(90-x)+tan(90-y) com
>> 90-x+90-y fixo. Ou como maximizar tan(x) + tan(y) com x+y fixo.
>>
>> Geometricamente, tangente é cateto oposto dividido por cateto
>> adjacente. Logo uma soma de tangentes com catetos adjacentes iguais
>> equivale a uma soma de catetos opostos! Assim sendo, nosso problema
>> pode ser pensado da seguinte forma:
>>
>> Dados um ponto A e uma reta d fixos, temos que construir duas retas x
>> e y, com ângulo 'alfa' entre elas, ambas passando por A e tais que a
>> distância entre os pontos X e Y, que elas geram ao intersectar d, seja
>> mínima.
>>
>> Daí fica fácil argumentar que a altura por A também tem que ser a
>> bissetriz por A.
>>
>> No fundo do fundo é uma forma de geometrizar a solução trigonométrica.
>> A trigonometria se torna apenas um atalho.
>>
>> Vou formalizar isso mais tarde, com desenhos e tudo.
>>
>>
>>
>> >
>> > Isso até me lembra o famoso artigo do Shine sobre geometria cearense
>> > VS geometria paulista:
>> > https://cyshine.webs.com/geometria-2005.pdf
>> >
>> >
>> > >
>> > > Saudações,
>> > > PJMS
>> > >
>> > > Em ter., 18 de ago. de 2020 às 11:34, Claudio Buffara 
>> > >  escreveu:
>> > >>
>> > >> Será que tem uma demonstração mais geométrica e menos algébrica disso? 
>> > >> E que torne o resultado mais intuitivo?
>> > >> É razoável que o ponto P não esteja muito próximo de qualquer dos 
>> > >> lados, pois neste caso, se P se aproximasse do lado a, por exemplo, 
>> > >> a/h_a cresceria e a expressão se afastaria do valor mínimo.
>> > >> Mas, com lados não necessariamente congruentes, não é óbvio, a priori, 
>> > >> que P deva ser equidistante dos três.
>> > >> De fato, seria razoável esperar que P estivesse mais próximo do maior 
>> > >> lado e conjecturar, por exemplo, que o P que minimiza a expressão é tal 
>> > >> que a/h_a = b/h_b = c/h_c.
>> > >> O fato de P ser o incentro não me parece a conjectura mais evidente 
>> > >> neste caso.
>> > >>
>> > >>
>> > >> On Sun, Aug 16, 2020 at 10:11 AM Matheus Secco  
>> > >> wrote:
>> > >>>
>> > >>> Olá, Vanderlei.
>> > >>> Por Cauchy-Schwarz, temos
>> > >>>
>> > >>> (a/ha + b/hb + c/hc) * (a*ha + b*hb + c*hc) >= (a+b+c)^2.  (#)
>> > >>>
>> > >>> Como (a*ha + b*hb + c*hc) = 2S, onde S é a área de ABC, segue que a 
>> > >>> expressão a/ha + b/hb + c/hc é pelo menos 2p^2/S, onde p é o 
>> > >>> semi-perimetro.
>> > >>>
>> > >>> Por outro lado, a igualdade em (#) ocorre se, e somente se, ha = hb = 
>> > >>> hc, ou seja, quando P é o incentro do triângulo
>> > >>>
>> > >>> Abraços,
>> > >>> Matheus
>> > >>>
>> > >>> Em dom, 16 de ago de 2020 08:59, Professor Vanderlei Nemitz 
>> > >>>  escreveu:
>> > 
>> >  Bom dia!
>> > 
>> >  Tentei utilizar alguma desigualdade de médias aqui, mas não tive 
>> >  êxito. Alguém ajuda?
>> >  Muito agradecido!
>> > 
>> >  Seja P um ponto no 

[obm-l] Re: [obm-l] Re: [obm-l] Re: [obm-l] Re: [obm-l] Re: [obm-l] Geometria plana com desigualdade de médias?

2020-08-26 Por tôpico Pedro José
Boa noite!
Anderson,
achei legal a sua visão. Mas não consegui evoluir com nada.
Todavia, fiquei com uma dúvida. Como x+y é um dos ângulos do triângulo
temos a restrição 0 escreveu:

> Em qui., 20 de ago. de 2020 às 22:03, Anderson Torres
>  escreveu:
> >
> > Em ter., 18 de ago. de 2020 às 19:51, Pedro José 
> escreveu:
> > >
> > > Boa noite!
> > > Cláudio,
> > > não consegui nada geométrico.
> > > O máximo que atingi foi:
> > > a/ha + b/hb + c/hc= [cotg(A1) +cotg (A2)]  + [cotg(B1) +cotg (B2)] +
> co[tg(C1) +cotg (C2)] com A1 + A2 = A; B1 + B2 + B e C1 + C2 = C.
> > > Para ser mínimo cada termo entre colchetes deve ser mínimo, o que
> ocorre quando A1 = A2; B1 = B2 e C1 = C2. Logo P seria o encontro das
> bissetrizes e logo I.
> > > Onde: A1= PAB e A2=PAC; B1=PBA e B2=PBC; C1=PCA e C2=PCB.
> >
> > Acho que daqui poderia sair uma interpretação mais escamoteada.
> > Afinal, trigonometria é uma espécie de "ponto de contato" entre a
> > geometria analítica e a sintética, entre a nuvem de desenhos e a de
> > números.
> >
> > Acredito que a solução aqui seria arranjar uma interpretação
> > geométrica desses colchetes de co-tangentes. Acredito que possamos
> > apelar para Ptolomeu em algum momento ou para um macete de
> > semelhanças, pois as projeções de um ponto sobre duas retas criam um
> > quadrilátero cíclico.
>
> Acrescentando mais coisas: se queremos minimizar cot(x) +cot(y) com
> x+y fixo, isto é equivalente a minimizar tan(90-x)+tan(90-y) com
> 90-x+90-y fixo. Ou como maximizar tan(x) + tan(y) com x+y fixo.
>
> Geometricamente, tangente é cateto oposto dividido por cateto
> adjacente. Logo uma soma de tangentes com catetos adjacentes iguais
> equivale a uma soma de catetos opostos! Assim sendo, nosso problema
> pode ser pensado da seguinte forma:
>
> Dados um ponto A e uma reta d fixos, temos que construir duas retas x
> e y, com ângulo 'alfa' entre elas, ambas passando por A e tais que a
> distância entre os pontos X e Y, que elas geram ao intersectar d, seja
> mínima.
>
> Daí fica fácil argumentar que a altura por A também tem que ser a
> bissetriz por A.
>
> No fundo do fundo é uma forma de geometrizar a solução trigonométrica.
> A trigonometria se torna apenas um atalho.
>
> Vou formalizar isso mais tarde, com desenhos e tudo.
>
>
>
> >
> > Isso até me lembra o famoso artigo do Shine sobre geometria cearense
> > VS geometria paulista:
> > https://cyshine.webs.com/geometria-2005.pdf
> >
> >
> > >
> > > Saudações,
> > > PJMS
> > >
> > > Em ter., 18 de ago. de 2020 às 11:34, Claudio Buffara <
> claudio.buff...@gmail.com> escreveu:
> > >>
> > >> Será que tem uma demonstração mais geométrica e menos algébrica
> disso? E que torne o resultado mais intuitivo?
> > >> É razoável que o ponto P não esteja muito próximo de qualquer dos
> lados, pois neste caso, se P se aproximasse do lado a, por exemplo, a/h_a
> cresceria e a expressão se afastaria do valor mínimo.
> > >> Mas, com lados não necessariamente congruentes, não é óbvio, a
> priori, que P deva ser equidistante dos três.
> > >> De fato, seria razoável esperar que P estivesse mais próximo do maior
> lado e conjecturar, por exemplo, que o P que minimiza a expressão é tal que
> a/h_a = b/h_b = c/h_c.
> > >> O fato de P ser o incentro não me parece a conjectura mais evidente
> neste caso.
> > >>
> > >>
> > >> On Sun, Aug 16, 2020 at 10:11 AM Matheus Secco <
> matheusse...@gmail.com> wrote:
> > >>>
> > >>> Olá, Vanderlei.
> > >>> Por Cauchy-Schwarz, temos
> > >>>
> > >>> (a/ha + b/hb + c/hc) * (a*ha + b*hb + c*hc) >= (a+b+c)^2.  (#)
> > >>>
> > >>> Como (a*ha + b*hb + c*hc) = 2S, onde S é a área de ABC, segue que a
> expressão a/ha + b/hb + c/hc é pelo menos 2p^2/S, onde p é o semi-perimetro.
> > >>>
> > >>> Por outro lado, a igualdade em (#) ocorre se, e somente se, ha = hb
> = hc, ou seja, quando P é o incentro do triângulo
> > >>>
> > >>> Abraços,
> > >>> Matheus
> > >>>
> > >>> Em dom, 16 de ago de 2020 08:59, Professor Vanderlei Nemitz <
> vanderma...@gmail.com> escreveu:
> > 
> >  Bom dia!
> > 
> >  Tentei utilizar alguma desigualdade de médias aqui, mas não tive
> êxito. Alguém ajuda?
> >  Muito agradecido!
> > 
> >  Seja P um ponto no interior de um triângulo e sejam ha, hb e hc as
> distâncias de P aos lados a, b e c, respectivamente. Mostre que o valor
> mínimo de (a/ha) + (b/hb) + (c/hc) ocorre quando P é o incentivo do
> triângulo ABC.
> > 
> >  --
> >  Esta mensagem foi verificada pelo sistema de antivírus e
> >  acredita-se estar livre de perigo.
> > >>>
> > >>>
> > >>> --
> > >>> Esta mensagem foi verificada pelo sistema de antivírus e
> > >>> acredita-se estar livre de perigo.
> > >>
> > >>
> > >> --
> > >> Esta mensagem foi verificada pelo sistema de antivírus e
> > >> acredita-se estar livre de perigo.
> > >
> > >
> > > --
> > > Esta mensagem foi verificada pelo sistema de antivírus e
> > > acredita-se estar livre de perigo.
>
> --
> Esta mensagem foi verificada pelo sistema de antivírus e
>  

Re: [obm-l] Re: [obm-l] Re: [obm-l] Re: [obm-l] Elipse e lugar g eométrico

2020-08-25 Por tôpico qedtexte

Tenho um arquivo com uma figura mostrando as
elipses. Posso mandar no privado pra quem quiser.

Lus
--
Esta mensagem foi verificada pelo sistema de antiv�rus e
acredita-se estar livre de perigo.



[obm-l] Re: [obm-l] Re: [obm-l] Re: [obm-l] Elipse e lugar geométrico

2020-08-24 Por tôpico Claudio Buffara
Tem um artigo do (saudoso) Morgado na RPM sobre este assunto. Está aqui:
http://www.rpm.org.br/cdrpm/43/5.htm

[]s,
Claudio.

On Sat, Aug 22, 2020 at 9:14 PM Professor Vanderlei Nemitz <
vanderma...@gmail.com> wrote:

> Demorei para responder, mas queria dizer que foi muito boa sua resolução,
> como sempre, Ralph!
> Eu desconhecia o fato de as coordenadas do incentro serem dadas daquela
> forma.
>
> Muitíssimo obrigado!
>
> Vanderlei
>
>
> 
>  Livre
> de vírus. www.avast.com
> .
> <#m_3828508563874758992_DAB4FAD8-2DD7-40BB-A1B8-4E2AA1F9FDF2>
>
> Em qui., 20 de ago. de 2020 às 00:37, Ralph Costa Teixeira <
> ralp...@gmail.com> escreveu:
>
>> As coordenadas do incentro sao a media ponderada das coordenadas dos
>> vertices, usando os lados como pesos. Ou seja, se escrevo P=(5cost,4sint),
>> F1=(-3,0), F2=(3,0) e Incentro=(x,y):
>>
>> x = (30cost + (-3)b + 3c) / 16
>> y = (24sint + 0 + 0) / 16
>>
>> onde b=d(P,F2) e c=d(P,F1). Note que b+c=eixo maior = 10.
>>
>> Mais especificamente:
>>
>> b^2=d(P,F2)^2=(5cost-3)^2+(4sint)^2=9(cost)^2-30cost+25=(3cost-5)^2, ou
>> seja, b=5-3cost, portanto c=5+3cost.
>>
>> Jogando na formula de x e y:
>>
>> x= 3cost ; y=3sint/2. Outra elipse, a saber, (x^2)/9+(y^2)/(9/4)=1
>> (talvez tirando os pontos onde tudo degenera, para ser chato).
>>
>> Abraço, Ralph.
>>
>>
>>
>> Hmm Assim:
>>
>> On Wed, Aug 19, 2020 at 11:58 PM Professor Vanderlei Nemitz <
>> vanderma...@gmail.com> wrote:
>>
>>> Oi!
>>> Venho com mais uma envolvendo incentro.
>>>
>>> *O ponto P pertence a uma elipse de focos F1 e F2 e de equação (x^2)/25
>>> + (y^2)/16 = 1. Determine o lugar geométrico do incentro do triângulo
>>> PF1F2.*
>>>
>>> Muito obrigado!
>>>
>>>
>>> 
>>>  Livre
>>> de vírus. www.avast.com
>>> .
>>>
>>> <#m_3828508563874758992_m_6041639077674691514_m_2344932968934913062_DAB4FAD8-2DD7-40BB-A1B8-4E2AA1F9FDF2>
>>>
>>> --
>>> Esta mensagem foi verificada pelo sistema de antivírus e
>>> acredita-se estar livre de perigo.
>>
>>
>> --
>> Esta mensagem foi verificada pelo sistema de antivírus e
>> acredita-se estar livre de perigo.
>
>
> --
> Esta mensagem foi verificada pelo sistema de antivírus e
> acredita-se estar livre de perigo.

-- 
Esta mensagem foi verificada pelo sistema de antiv�rus e
 acredita-se estar livre de perigo.



[obm-l] Re: [obm-l] Re: [obm-l] Re: [obm-l] Re: [obm-l] Geometria plana com desigualdade de médias?

2020-08-20 Por tôpico Anderson Torres
Em qui., 20 de ago. de 2020 às 22:03, Anderson Torres
 escreveu:
>
> Em ter., 18 de ago. de 2020 às 19:51, Pedro José  
> escreveu:
> >
> > Boa noite!
> > Cláudio,
> > não consegui nada geométrico.
> > O máximo que atingi foi:
> > a/ha + b/hb + c/hc= [cotg(A1) +cotg (A2)]  + [cotg(B1) +cotg (B2)] + 
> > co[tg(C1) +cotg (C2)] com A1 + A2 = A; B1 + B2 + B e C1 + C2 = C.
> > Para ser mínimo cada termo entre colchetes deve ser mínimo, o que ocorre 
> > quando A1 = A2; B1 = B2 e C1 = C2. Logo P seria o encontro das bissetrizes 
> > e logo I.
> > Onde: A1= PAB e A2=PAC; B1=PBA e B2=PBC; C1=PCA e C2=PCB.
>
> Acho que daqui poderia sair uma interpretação mais escamoteada.
> Afinal, trigonometria é uma espécie de "ponto de contato" entre a
> geometria analítica e a sintética, entre a nuvem de desenhos e a de
> números.
>
> Acredito que a solução aqui seria arranjar uma interpretação
> geométrica desses colchetes de co-tangentes. Acredito que possamos
> apelar para Ptolomeu em algum momento ou para um macete de
> semelhanças, pois as projeções de um ponto sobre duas retas criam um
> quadrilátero cíclico.

Acrescentando mais coisas: se queremos minimizar cot(x) +cot(y) com
x+y fixo, isto é equivalente a minimizar tan(90-x)+tan(90-y) com
90-x+90-y fixo. Ou como maximizar tan(x) + tan(y) com x+y fixo.

Geometricamente, tangente é cateto oposto dividido por cateto
adjacente. Logo uma soma de tangentes com catetos adjacentes iguais
equivale a uma soma de catetos opostos! Assim sendo, nosso problema
pode ser pensado da seguinte forma:

Dados um ponto A e uma reta d fixos, temos que construir duas retas x
e y, com ângulo 'alfa' entre elas, ambas passando por A e tais que a
distância entre os pontos X e Y, que elas geram ao intersectar d, seja
mínima.

Daí fica fácil argumentar que a altura por A também tem que ser a
bissetriz por A.

No fundo do fundo é uma forma de geometrizar a solução trigonométrica.
A trigonometria se torna apenas um atalho.

Vou formalizar isso mais tarde, com desenhos e tudo.



>
> Isso até me lembra o famoso artigo do Shine sobre geometria cearense
> VS geometria paulista:
> https://cyshine.webs.com/geometria-2005.pdf
>
>
> >
> > Saudações,
> > PJMS
> >
> > Em ter., 18 de ago. de 2020 às 11:34, Claudio Buffara 
> >  escreveu:
> >>
> >> Será que tem uma demonstração mais geométrica e menos algébrica disso? E 
> >> que torne o resultado mais intuitivo?
> >> É razoável que o ponto P não esteja muito próximo de qualquer dos lados, 
> >> pois neste caso, se P se aproximasse do lado a, por exemplo, a/h_a 
> >> cresceria e a expressão se afastaria do valor mínimo.
> >> Mas, com lados não necessariamente congruentes, não é óbvio, a priori, que 
> >> P deva ser equidistante dos três.
> >> De fato, seria razoável esperar que P estivesse mais próximo do maior lado 
> >> e conjecturar, por exemplo, que o P que minimiza a expressão é tal que 
> >> a/h_a = b/h_b = c/h_c.
> >> O fato de P ser o incentro não me parece a conjectura mais evidente neste 
> >> caso.
> >>
> >>
> >> On Sun, Aug 16, 2020 at 10:11 AM Matheus Secco  
> >> wrote:
> >>>
> >>> Olá, Vanderlei.
> >>> Por Cauchy-Schwarz, temos
> >>>
> >>> (a/ha + b/hb + c/hc) * (a*ha + b*hb + c*hc) >= (a+b+c)^2.  (#)
> >>>
> >>> Como (a*ha + b*hb + c*hc) = 2S, onde S é a área de ABC, segue que a 
> >>> expressão a/ha + b/hb + c/hc é pelo menos 2p^2/S, onde p é o 
> >>> semi-perimetro.
> >>>
> >>> Por outro lado, a igualdade em (#) ocorre se, e somente se, ha = hb = hc, 
> >>> ou seja, quando P é o incentro do triângulo
> >>>
> >>> Abraços,
> >>> Matheus
> >>>
> >>> Em dom, 16 de ago de 2020 08:59, Professor Vanderlei Nemitz 
> >>>  escreveu:
> 
>  Bom dia!
> 
>  Tentei utilizar alguma desigualdade de médias aqui, mas não tive êxito. 
>  Alguém ajuda?
>  Muito agradecido!
> 
>  Seja P um ponto no interior de um triângulo e sejam ha, hb e hc as 
>  distâncias de P aos lados a, b e c, respectivamente. Mostre que o valor 
>  mínimo de (a/ha) + (b/hb) + (c/hc) ocorre quando P é o incentivo do 
>  triângulo ABC.
> 
>  --
>  Esta mensagem foi verificada pelo sistema de antivírus e
>  acredita-se estar livre de perigo.
> >>>
> >>>
> >>> --
> >>> Esta mensagem foi verificada pelo sistema de antivírus e
> >>> acredita-se estar livre de perigo.
> >>
> >>
> >> --
> >> Esta mensagem foi verificada pelo sistema de antivírus e
> >> acredita-se estar livre de perigo.
> >
> >
> > --
> > Esta mensagem foi verificada pelo sistema de antivírus e
> > acredita-se estar livre de perigo.

-- 
Esta mensagem foi verificada pelo sistema de antiv�rus e
 acredita-se estar livre de perigo.


=
Instru��es para entrar na lista, sair da lista e usar a lista em
http://www.mat.puc-rio.br/~obmlistas/obm-l.html
=


[obm-l] Re: [obm-l] Re: [obm-l] Re: [obm-l] Re: [obm-l] Geometria plana com desigualdade de médias?

2020-08-20 Por tôpico Anderson Torres
Em ter., 18 de ago. de 2020 às 19:51, Pedro José  escreveu:
>
> Boa noite!
> Cláudio,
> não consegui nada geométrico.
> O máximo que atingi foi:
> a/ha + b/hb + c/hc= [cotg(A1) +cotg (A2)]  + [cotg(B1) +cotg (B2)] + 
> co[tg(C1) +cotg (C2)] com A1 + A2 = A; B1 + B2 + B e C1 + C2 = C.
> Para ser mínimo cada termo entre colchetes deve ser mínimo, o que ocorre 
> quando A1 = A2; B1 = B2 e C1 = C2. Logo P seria o encontro das bissetrizes e 
> logo I.
> Onde: A1= PAB e A2=PAC; B1=PBA e B2=PBC; C1=PCA e C2=PCB.

Acho que daqui poderia sair uma interpretação mais escamoteada.
Afinal, trigonometria é uma espécie de "ponto de contato" entre a
geometria analítica e a sintética, entre a nuvem de desenhos e a de
números.

Acredito que a solução aqui seria arranjar uma interpretação
geométrica desses colchetes de co-tangentes. Acredito que possamos
apelar para Ptolomeu em algum momento ou para um macete de
semelhanças, pois as projeções de um ponto sobre duas retas criam um
quadrilátero cíclico.

Isso até me lembra o famoso artigo do Shine sobre geometria cearense
VS geometria paulista:
https://cyshine.webs.com/geometria-2005.pdf


>
> Saudações,
> PJMS
>
> Em ter., 18 de ago. de 2020 às 11:34, Claudio Buffara 
>  escreveu:
>>
>> Será que tem uma demonstração mais geométrica e menos algébrica disso? E que 
>> torne o resultado mais intuitivo?
>> É razoável que o ponto P não esteja muito próximo de qualquer dos lados, 
>> pois neste caso, se P se aproximasse do lado a, por exemplo, a/h_a cresceria 
>> e a expressão se afastaria do valor mínimo.
>> Mas, com lados não necessariamente congruentes, não é óbvio, a priori, que P 
>> deva ser equidistante dos três.
>> De fato, seria razoável esperar que P estivesse mais próximo do maior lado e 
>> conjecturar, por exemplo, que o P que minimiza a expressão é tal que a/h_a = 
>> b/h_b = c/h_c.
>> O fato de P ser o incentro não me parece a conjectura mais evidente neste 
>> caso.
>>
>>
>> On Sun, Aug 16, 2020 at 10:11 AM Matheus Secco  
>> wrote:
>>>
>>> Olá, Vanderlei.
>>> Por Cauchy-Schwarz, temos
>>>
>>> (a/ha + b/hb + c/hc) * (a*ha + b*hb + c*hc) >= (a+b+c)^2.  (#)
>>>
>>> Como (a*ha + b*hb + c*hc) = 2S, onde S é a área de ABC, segue que a 
>>> expressão a/ha + b/hb + c/hc é pelo menos 2p^2/S, onde p é o semi-perimetro.
>>>
>>> Por outro lado, a igualdade em (#) ocorre se, e somente se, ha = hb = hc, 
>>> ou seja, quando P é o incentro do triângulo
>>>
>>> Abraços,
>>> Matheus
>>>
>>> Em dom, 16 de ago de 2020 08:59, Professor Vanderlei Nemitz 
>>>  escreveu:

 Bom dia!

 Tentei utilizar alguma desigualdade de médias aqui, mas não tive êxito. 
 Alguém ajuda?
 Muito agradecido!

 Seja P um ponto no interior de um triângulo e sejam ha, hb e hc as 
 distâncias de P aos lados a, b e c, respectivamente. Mostre que o valor 
 mínimo de (a/ha) + (b/hb) + (c/hc) ocorre quando P é o incentivo do 
 triângulo ABC.

 --
 Esta mensagem foi verificada pelo sistema de antivírus e
 acredita-se estar livre de perigo.
>>>
>>>
>>> --
>>> Esta mensagem foi verificada pelo sistema de antivírus e
>>> acredita-se estar livre de perigo.
>>
>>
>> --
>> Esta mensagem foi verificada pelo sistema de antivírus e
>> acredita-se estar livre de perigo.
>
>
> --
> Esta mensagem foi verificada pelo sistema de antivírus e
> acredita-se estar livre de perigo.

-- 
Esta mensagem foi verificada pelo sistema de antiv�rus e
 acredita-se estar livre de perigo.


=
Instru��es para entrar na lista, sair da lista e usar a lista em
http://www.mat.puc-rio.br/~obmlistas/obm-l.html
=


[obm-l] Re: [obm-l] Re: [obm-l] Re: [obm-l] Re: [obm-l] Re: [obm-l] Re: [obm-l] Re: [obm-l] polinômio irredutível

2020-08-20 Por tôpico Anderson Torres
Em seg., 17 de ago. de 2020 às 12:14, Claudio Buffara
 escreveu:
>
> Eu acho que o Eisenstein inventou este critério pra polinômios da forma x^n + 
> a ou, mais geralmente, pra polinômios ciclotômicos.
> Daí funciona bem.
>
> On Mon, Aug 17, 2020 at 11:02 AM Esdras Muniz  
> wrote:
>>
>> E se p=3, e p divide N^2+9, então p^2 divide N^2+9.
>>
>> Então o critério de Eisenstein realmente não é tão abrangente. Será que tem 
>> algum outro critério que cubra casos em que o de Eisenstein não cubra?
>>
>> Em seg, 17 de ago de 2020 09:46, Claudio Buffara  
>> escreveu:
>>>
>>> Boa! Se p <> 3 mas p divide 3N e 3N^2, então p divide N ==> p não divide 
>>> N^3 + 9.
>>>
>>> On Sun, Aug 16, 2020 at 10:51 PM Esdras Muniz  
>>> wrote:

 Tenta com x^3+9.

 Em dom, 16 de ago de 2020 15:24, Claudio Buffara 
  escreveu:
>
> f(x) em Z[x], bem entendido...
>
>
> On Sun, Aug 16, 2020 at 3:08 PM Claudio Buffara 
>  wrote:
>>
>> Que tal essa aqui?
>> Prove ou disprove que, dado um polinômio f(x), irredutível sobre Q, 
>> existe um inteiro N tal que a irredutibilidade de f pode ser provada 
>> pelo critério de Eisenstein aplicado a f(x+N).

Isso me parece uma daquelas questões ultra capciosas sobre "prove ou
disprove que existe um algoritmo que..."

Inclusive imagino que esta seja uma questão indecidível neste caso particular...

>>
>> On Sun, Aug 16, 2020 at 2:31 PM Matheus Secco  
>> wrote:
>>>
>>> O melhor jeito é pensar na contrapositiva (supondo que você esteja 
>>> falando sobre irredutibilidade em Z[x] ou até em Q[x]): se f(x) fatora 
>>> como g(x)*h(x), então f(x+a) fatora como g(x+a) *h(x+a) e é claro que 
>>> uma vez que g(x) e h(x) têm coeficientes inteiros, então g(x+a) e 
>>> h(x+a) também têm. A recíproca é essencialmente idêntica.
>>>
>>> Abraços
>>>
>>> Em dom, 16 de ago de 2020 14:11, Luís Lopes  
>>> escreveu:

 Sauda,c~oes,

 Como provar que um polinômio f(x) tendo como coeficientes números 
 inteiros
 é irredutível se e somente se f(x+a) é irredutível para algum  
 inteiro ?

 Luís




 --
 Esta mensagem foi verificada pelo sistema de antivírus e
 acredita-se estar livre de perigo.
>>>
>>>
>>> --
>>> Esta mensagem foi verificada pelo sistema de antivírus e
>>> acredita-se estar livre de perigo.
>
>
> --
> Esta mensagem foi verificada pelo sistema de antivírus e
> acredita-se estar livre de perigo.


 --
 Esta mensagem foi verificada pelo sistema de antivírus e
 acredita-se estar livre de perigo.
>>>
>>>
>>> --
>>> Esta mensagem foi verificada pelo sistema de antivírus e
>>> acredita-se estar livre de perigo.
>>
>>
>> --
>> Esta mensagem foi verificada pelo sistema de antivírus e
>> acredita-se estar livre de perigo.
>
>
> --
> Esta mensagem foi verificada pelo sistema de antivírus e
> acredita-se estar livre de perigo.

-- 
Esta mensagem foi verificada pelo sistema de antiv�rus e
 acredita-se estar livre de perigo.


=
Instru��es para entrar na lista, sair da lista e usar a lista em
http://www.mat.puc-rio.br/~obmlistas/obm-l.html
=


[obm-l] Re: [obm-l] Re: [obm-l] Re: [obm-l] Re: [obm-l] Geometria plana com desigualdade de médias?

2020-08-18 Por tôpico Claudio Buffara
Realmente, não era isso que eu estava procurando...  mas valeu! É outra
solução.


On Tue, Aug 18, 2020 at 7:51 PM Pedro José  wrote:

> Boa noite!
> Cláudio,
> não consegui nada geométrico.
> O máximo que atingi foi:
> a/ha + b/hb + c/hc= [cotg(A1) +cotg (A2)]  + [cotg(B1) +cotg (B2)] +
> co[tg(C1) +cotg (C2)] com A1 + A2 = A; B1 + B2 + B e C1 + C2 = C.
> Para ser mínimo cada termo entre colchetes deve ser mínimo, o que ocorre
> quando A1 = A2; B1 = B2 e C1 = C2. Logo P seria o encontro das bissetrizes
> e logo I.
> Onde: A1= PAB e A2=PAC; B1=PBA e B2=PBC; C1=PCA e C2=PCB.
>
> Saudações,
> PJMS
>
> Em ter., 18 de ago. de 2020 às 11:34, Claudio Buffara <
> claudio.buff...@gmail.com> escreveu:
>
>> Será que tem uma demonstração mais geométrica e menos algébrica disso? E
>> que torne o resultado mais intuitivo?
>> É razoável que o ponto P não esteja muito próximo de qualquer dos lados,
>> pois neste caso, se P se aproximasse do lado a, por exemplo,
>> a/h_a cresceria e a expressão se afastaria do valor mínimo.
>> Mas, com lados não necessariamente congruentes, não é óbvio, a priori,
>> que P deva ser equidistante dos três.
>> De fato, seria razoável esperar que P estivesse mais próximo do maior
>> lado e conjecturar, por exemplo, que o P que minimiza a expressão é tal que
>> a/h_a = b/h_b = c/h_c.
>> O fato de P ser o incentro não me parece a conjectura mais evidente neste
>> caso.
>>
>>
>> On Sun, Aug 16, 2020 at 10:11 AM Matheus Secco 
>> wrote:
>>
>>> Olá, Vanderlei.
>>> Por Cauchy-Schwarz, temos
>>>
>>> (a/ha + b/hb + c/hc) * (a*ha + b*hb + c*hc) >= (a+b+c)^2.  (#)
>>>
>>> Como (a*ha + b*hb + c*hc) = 2S, onde S é a área de ABC, segue que a
>>> expressão a/ha + b/hb + c/hc é pelo menos 2p^2/S, onde p é o
>>> semi-perimetro.
>>>
>>> Por outro lado, a igualdade em (#) ocorre se, e somente se, ha = hb =
>>> hc, ou seja, quando P é o incentro do triângulo
>>>
>>> Abraços,
>>> Matheus
>>>
>>> Em dom, 16 de ago de 2020 08:59, Professor Vanderlei Nemitz <
>>> vanderma...@gmail.com> escreveu:
>>>
 Bom dia!

 Tentei utilizar alguma desigualdade de médias aqui, mas não tive êxito.
 Alguém ajuda?
 Muito agradecido!

 Seja P um ponto no interior de um triângulo e sejam ha, hb e hc as
 distâncias de P aos lados a, b e c, respectivamente. Mostre que o valor
 mínimo de (a/ha) + (b/hb) + (c/hc) ocorre quando P é o incentivo do
 triângulo ABC.

 --
 Esta mensagem foi verificada pelo sistema de antivírus e
 acredita-se estar livre de perigo.
>>>
>>>
>>> --
>>> Esta mensagem foi verificada pelo sistema de antivírus e
>>> acredita-se estar livre de perigo.
>>
>>
>> --
>> Esta mensagem foi verificada pelo sistema de antivírus e
>> acredita-se estar livre de perigo.
>
>
> --
> Esta mensagem foi verificada pelo sistema de antivírus e
> acredita-se estar livre de perigo.

-- 
Esta mensagem foi verificada pelo sistema de antiv�rus e
 acredita-se estar livre de perigo.



[obm-l] Re: [obm-l] Re: [obm-l] Re: [obm-l] Geometria plana com desigualdade de médias?

2020-08-18 Por tôpico Pedro José
Boa noite!
Cláudio,
não consegui nada geométrico.
O máximo que atingi foi:
a/ha + b/hb + c/hc= [cotg(A1) +cotg (A2)]  + [cotg(B1) +cotg (B2)] +
co[tg(C1) +cotg (C2)] com A1 + A2 = A; B1 + B2 + B e C1 + C2 = C.
Para ser mínimo cada termo entre colchetes deve ser mínimo, o que ocorre
quando A1 = A2; B1 = B2 e C1 = C2. Logo P seria o encontro das bissetrizes
e logo I.
Onde: A1= PAB e A2=PAC; B1=PBA e B2=PBC; C1=PCA e C2=PCB.

Saudações,
PJMS

Em ter., 18 de ago. de 2020 às 11:34, Claudio Buffara <
claudio.buff...@gmail.com> escreveu:

> Será que tem uma demonstração mais geométrica e menos algébrica disso? E
> que torne o resultado mais intuitivo?
> É razoável que o ponto P não esteja muito próximo de qualquer dos lados,
> pois neste caso, se P se aproximasse do lado a, por exemplo,
> a/h_a cresceria e a expressão se afastaria do valor mínimo.
> Mas, com lados não necessariamente congruentes, não é óbvio, a priori, que
> P deva ser equidistante dos três.
> De fato, seria razoável esperar que P estivesse mais próximo do maior lado
> e conjecturar, por exemplo, que o P que minimiza a expressão é tal que
> a/h_a = b/h_b = c/h_c.
> O fato de P ser o incentro não me parece a conjectura mais evidente neste
> caso.
>
>
> On Sun, Aug 16, 2020 at 10:11 AM Matheus Secco 
> wrote:
>
>> Olá, Vanderlei.
>> Por Cauchy-Schwarz, temos
>>
>> (a/ha + b/hb + c/hc) * (a*ha + b*hb + c*hc) >= (a+b+c)^2.  (#)
>>
>> Como (a*ha + b*hb + c*hc) = 2S, onde S é a área de ABC, segue que a
>> expressão a/ha + b/hb + c/hc é pelo menos 2p^2/S, onde p é o
>> semi-perimetro.
>>
>> Por outro lado, a igualdade em (#) ocorre se, e somente se, ha = hb = hc,
>> ou seja, quando P é o incentro do triângulo
>>
>> Abraços,
>> Matheus
>>
>> Em dom, 16 de ago de 2020 08:59, Professor Vanderlei Nemitz <
>> vanderma...@gmail.com> escreveu:
>>
>>> Bom dia!
>>>
>>> Tentei utilizar alguma desigualdade de médias aqui, mas não tive êxito.
>>> Alguém ajuda?
>>> Muito agradecido!
>>>
>>> Seja P um ponto no interior de um triângulo e sejam ha, hb e hc as
>>> distâncias de P aos lados a, b e c, respectivamente. Mostre que o valor
>>> mínimo de (a/ha) + (b/hb) + (c/hc) ocorre quando P é o incentivo do
>>> triângulo ABC.
>>>
>>> --
>>> Esta mensagem foi verificada pelo sistema de antivírus e
>>> acredita-se estar livre de perigo.
>>
>>
>> --
>> Esta mensagem foi verificada pelo sistema de antivírus e
>> acredita-se estar livre de perigo.
>
>
> --
> Esta mensagem foi verificada pelo sistema de antivírus e
> acredita-se estar livre de perigo.

-- 
Esta mensagem foi verificada pelo sistema de antiv�rus e
 acredita-se estar livre de perigo.



Re: [obm-l] Re: [obm-l] Re: [obm-l] Re: [obm-l] Re: [obm-l] Re: [obm-l] Re: [obm-l] polinômio irredutível

2020-08-17 Por tôpico qedtexte

Sauda,c~oes,

Legal o estudo dox^3+9.

Sobre oEisenstein generalizado (teorema 3 em

http://yufeizhao.com/olympiad/intpoly.pdf;), tenho duas 
dvidas:





Theorem 3(Extended Eisenstein).Letf(x) =anxn+an1xn1++a1x+a0be a polynomial with integer coefficients 
such thatp|aifor 0i  k,pﰀ|/akandp2ﰀ|/a0. Thenf(x) has an irreducible factor of degree greater thank.


Quando k=n obtm-se o critrio tradicional.

i) quais as condies para os outros coeficientes a_(k+1), a_(k+2), 
. , a_n ?
p pode dividi-los ou no ?

ii) o grau do fator irredutvel   k ou = k ?

Lus














--
Esta mensagem foi verificada pelo sistema de antiv�rus e
acredita-se estar livre de perigo.



[obm-l] Re: [obm-l] Re: [obm-l] Re: [obm-l] Re: [obm-l] Re: [obm-l] Re: [obm-l] polinômio irredutível

2020-08-17 Por tôpico Claudio Buffara
Eu acho que o Eisenstein inventou este critério pra polinômios da forma
x^n + a ou, mais geralmente, pra polinômios ciclotômicos.
Daí funciona bem.

On Mon, Aug 17, 2020 at 11:02 AM Esdras Muniz 
wrote:

> E se p=3, e p divide N^2+9, então p^2 divide N^2+9.
>
> Então o critério de Eisenstein realmente não é tão abrangente. Será que
> tem algum outro critério que cubra casos em que o de Eisenstein não cubra?
>
> Em seg, 17 de ago de 2020 09:46, Claudio Buffara <
> claudio.buff...@gmail.com> escreveu:
>
>> Boa! Se p <> 3 mas p divide 3N e 3N^2, então p divide N ==> p não divide
>> N^3 + 9.
>>
>> On Sun, Aug 16, 2020 at 10:51 PM Esdras Muniz 
>> wrote:
>>
>>> Tenta com x^3+9.
>>>
>>> Em dom, 16 de ago de 2020 15:24, Claudio Buffara <
>>> claudio.buff...@gmail.com> escreveu:
>>>
 f(x) em Z[x], bem entendido...


 On Sun, Aug 16, 2020 at 3:08 PM Claudio Buffara <
 claudio.buff...@gmail.com> wrote:

> Que tal essa aqui?
> Prove ou disprove que, dado um polinômio f(x), irredutível sobre Q,
> existe um inteiro N tal que a irredutibilidade de f pode ser provada pelo
> critério de Eisenstein aplicado a f(x+N).
>
> On Sun, Aug 16, 2020 at 2:31 PM Matheus Secco 
> wrote:
>
>> O melhor jeito é pensar na contrapositiva (supondo que você esteja
>> falando sobre irredutibilidade em Z[x] ou até em Q[x]): se f(x) fatora 
>> como
>> g(x)*h(x), então f(x+a) fatora como g(x+a) *h(x+a) e é claro que uma vez
>> que g(x) e h(x) têm coeficientes inteiros, então g(x+a) e h(x+a) também
>> têm. A recíproca é essencialmente idêntica.
>>
>> Abraços
>>
>> Em dom, 16 de ago de 2020 14:11, Luís Lopes 
>> escreveu:
>>
>>> Sauda,c~oes,
>>>
>>> Como provar que um polinômio f(x) tendo como coeficientes números
>>> inteiros
>>> é irredutível se e somente se f(x+a) é irredutível para algum 
>>> inteiro ?
>>>
>>> Luís
>>>
>>>
>>>
>>>
>>> --
>>> Esta mensagem foi verificada pelo sistema de antivírus e
>>> acredita-se estar livre de perigo.
>>>
>>
>> --
>> Esta mensagem foi verificada pelo sistema de antivírus e
>> acredita-se estar livre de perigo.
>
>
 --
 Esta mensagem foi verificada pelo sistema de antivírus e
 acredita-se estar livre de perigo.
>>>
>>>
>>> --
>>> Esta mensagem foi verificada pelo sistema de antivírus e
>>> acredita-se estar livre de perigo.
>>
>>
>> --
>> Esta mensagem foi verificada pelo sistema de antivírus e
>> acredita-se estar livre de perigo.
>
>
> --
> Esta mensagem foi verificada pelo sistema de antivírus e
> acredita-se estar livre de perigo.

-- 
Esta mensagem foi verificada pelo sistema de antiv�rus e
 acredita-se estar livre de perigo.



[obm-l] Re: [obm-l] Re: [obm-l] Re: [obm-l] Re: [obm-l] Re: [obm-l] polinômio irredutível

2020-08-17 Por tôpico Esdras Muniz
E se p=3, e p divide N^2+9, então p^2 divide N^2+9.

Então o critério de Eisenstein realmente não é tão abrangente. Será que tem
algum outro critério que cubra casos em que o de Eisenstein não cubra?

Em seg, 17 de ago de 2020 09:46, Claudio Buffara 
escreveu:

> Boa! Se p <> 3 mas p divide 3N e 3N^2, então p divide N ==> p não divide
> N^3 + 9.
>
> On Sun, Aug 16, 2020 at 10:51 PM Esdras Muniz 
> wrote:
>
>> Tenta com x^3+9.
>>
>> Em dom, 16 de ago de 2020 15:24, Claudio Buffara <
>> claudio.buff...@gmail.com> escreveu:
>>
>>> f(x) em Z[x], bem entendido...
>>>
>>>
>>> On Sun, Aug 16, 2020 at 3:08 PM Claudio Buffara <
>>> claudio.buff...@gmail.com> wrote:
>>>
 Que tal essa aqui?
 Prove ou disprove que, dado um polinômio f(x), irredutível sobre Q,
 existe um inteiro N tal que a irredutibilidade de f pode ser provada pelo
 critério de Eisenstein aplicado a f(x+N).

 On Sun, Aug 16, 2020 at 2:31 PM Matheus Secco 
 wrote:

> O melhor jeito é pensar na contrapositiva (supondo que você esteja
> falando sobre irredutibilidade em Z[x] ou até em Q[x]): se f(x) fatora 
> como
> g(x)*h(x), então f(x+a) fatora como g(x+a) *h(x+a) e é claro que uma vez
> que g(x) e h(x) têm coeficientes inteiros, então g(x+a) e h(x+a) também
> têm. A recíproca é essencialmente idêntica.
>
> Abraços
>
> Em dom, 16 de ago de 2020 14:11, Luís Lopes 
> escreveu:
>
>> Sauda,c~oes,
>>
>> Como provar que um polinômio f(x) tendo como coeficientes números
>> inteiros
>> é irredutível se e somente se f(x+a) é irredutível para algum 
>> inteiro ?
>>
>> Luís
>>
>>
>>
>>
>> --
>> Esta mensagem foi verificada pelo sistema de antivírus e
>> acredita-se estar livre de perigo.
>>
>
> --
> Esta mensagem foi verificada pelo sistema de antivírus e
> acredita-se estar livre de perigo.


>>> --
>>> Esta mensagem foi verificada pelo sistema de antivírus e
>>> acredita-se estar livre de perigo.
>>
>>
>> --
>> Esta mensagem foi verificada pelo sistema de antivírus e
>> acredita-se estar livre de perigo.
>
>
> --
> Esta mensagem foi verificada pelo sistema de antivírus e
> acredita-se estar livre de perigo.

-- 
Esta mensagem foi verificada pelo sistema de antiv�rus e
 acredita-se estar livre de perigo.



[obm-l] Re: [obm-l] Re: [obm-l] Re: [obm-l] Re: [obm-l] polinômio irredutível

2020-08-17 Por tôpico Claudio Buffara
Boa! Se p <> 3 mas p divide 3N e 3N^2, então p divide N ==> p não divide
N^3 + 9.

On Sun, Aug 16, 2020 at 10:51 PM Esdras Muniz 
wrote:

> Tenta com x^3+9.
>
> Em dom, 16 de ago de 2020 15:24, Claudio Buffara <
> claudio.buff...@gmail.com> escreveu:
>
>> f(x) em Z[x], bem entendido...
>>
>>
>> On Sun, Aug 16, 2020 at 3:08 PM Claudio Buffara <
>> claudio.buff...@gmail.com> wrote:
>>
>>> Que tal essa aqui?
>>> Prove ou disprove que, dado um polinômio f(x), irredutível sobre Q,
>>> existe um inteiro N tal que a irredutibilidade de f pode ser provada pelo
>>> critério de Eisenstein aplicado a f(x+N).
>>>
>>> On Sun, Aug 16, 2020 at 2:31 PM Matheus Secco 
>>> wrote:
>>>
 O melhor jeito é pensar na contrapositiva (supondo que você esteja
 falando sobre irredutibilidade em Z[x] ou até em Q[x]): se f(x) fatora como
 g(x)*h(x), então f(x+a) fatora como g(x+a) *h(x+a) e é claro que uma vez
 que g(x) e h(x) têm coeficientes inteiros, então g(x+a) e h(x+a) também
 têm. A recíproca é essencialmente idêntica.

 Abraços

 Em dom, 16 de ago de 2020 14:11, Luís Lopes 
 escreveu:

> Sauda,c~oes,
>
> Como provar que um polinômio f(x) tendo como coeficientes números
> inteiros
> é irredutível se e somente se f(x+a) é irredutível para algum 
> inteiro ?
>
> Luís
>
>
>
>
> --
> Esta mensagem foi verificada pelo sistema de antivírus e
> acredita-se estar livre de perigo.
>

 --
 Esta mensagem foi verificada pelo sistema de antivírus e
 acredita-se estar livre de perigo.
>>>
>>>
>> --
>> Esta mensagem foi verificada pelo sistema de antivírus e
>> acredita-se estar livre de perigo.
>
>
> --
> Esta mensagem foi verificada pelo sistema de antivírus e
> acredita-se estar livre de perigo.

-- 
Esta mensagem foi verificada pelo sistema de antiv�rus e
 acredita-se estar livre de perigo.



[obm-l] Re: [obm-l] Re: [obm-l] Re: [obm-l] polinômio irredutível

2020-08-16 Por tôpico Esdras Muniz
Tenta com x^3+9.

Em dom, 16 de ago de 2020 15:24, Claudio Buffara 
escreveu:

> f(x) em Z[x], bem entendido...
>
>
> On Sun, Aug 16, 2020 at 3:08 PM Claudio Buffara 
> wrote:
>
>> Que tal essa aqui?
>> Prove ou disprove que, dado um polinômio f(x), irredutível sobre Q,
>> existe um inteiro N tal que a irredutibilidade de f pode ser provada pelo
>> critério de Eisenstein aplicado a f(x+N).
>>
>> On Sun, Aug 16, 2020 at 2:31 PM Matheus Secco 
>> wrote:
>>
>>> O melhor jeito é pensar na contrapositiva (supondo que você esteja
>>> falando sobre irredutibilidade em Z[x] ou até em Q[x]): se f(x) fatora como
>>> g(x)*h(x), então f(x+a) fatora como g(x+a) *h(x+a) e é claro que uma vez
>>> que g(x) e h(x) têm coeficientes inteiros, então g(x+a) e h(x+a) também
>>> têm. A recíproca é essencialmente idêntica.
>>>
>>> Abraços
>>>
>>> Em dom, 16 de ago de 2020 14:11, Luís Lopes 
>>> escreveu:
>>>
 Sauda,c~oes,

 Como provar que um polinômio f(x) tendo como coeficientes números
 inteiros
 é irredutível se e somente se f(x+a) é irredutível para algum 
 inteiro ?

 Luís




 --
 Esta mensagem foi verificada pelo sistema de antivírus e
 acredita-se estar livre de perigo.

>>>
>>> --
>>> Esta mensagem foi verificada pelo sistema de antivírus e
>>> acredita-se estar livre de perigo.
>>
>>
> --
> Esta mensagem foi verificada pelo sistema de antivírus e
> acredita-se estar livre de perigo.

-- 
Esta mensagem foi verificada pelo sistema de antiv�rus e
 acredita-se estar livre de perigo.



[obm-l] Re: [obm-l] Re: [obm-l] Re: [obm-l] Mostrar que está função não existe

2020-08-11 Por tôpico Esdras Muniz
Depois de ver essa solicitação genial, fiquei com vergonha de mandar a
minha.

Em ter, 11 de ago de 2020 01:37, Ralph Costa Teixeira 
escreveu:

> Acho que isso caiu numa IMO que eu fiz Ah, achei, 1987. Aqui tem uma
> resposta bem legal:
>
>
> https://math.stackexchange.com/questions/325504/imo-1987-function-such-that-ffn-n1987
>
>
> On Tue, Aug 11, 2020 at 12:50 AM  wrote:
>
>> É, fatou dizer que k é ímpar
>>
>> Artur
>>
>> Em 10 de ago de 2020 22:33, Ralph Costa Teixeira 
>> escreveu:
>>
>> K inteiro... ímpar? Porque tomando f(n)=n+k/2...
>>
>> On Mon, Aug 10, 2020, 22:05 Artur Costa Steiner <
>> artur.costa.stei...@gmail.com> wrote:
>>
>> Me pareceu que isto era simples, mas segui um caminho errado e ainda não
>> cheguei lá.
>>
>> Mostre que não existe f:N --> N, N os naturais com o 0, tal que f(f(n)) =
>> n + k, k > 0 inteiro.
>>
>> Obrigado
>>
>> Artur
>>
>> --
>> Esta mensagem foi verificada pelo sistema de antivírus e
>> acredita-se estar livre de perigo.
>>
>>
>> --
>> Esta mensagem foi verificada pelo sistema de antiv�rus e
>> acredita-se estar livre de perigo.
>>
>>
>>
>> --
>> Esta mensagem foi verificada pelo sistema de antivírus e
>> acredita-se estar livre de perigo.
>
>
> --
> Esta mensagem foi verificada pelo sistema de antivírus e
> acredita-se estar livre de perigo.

-- 
Esta mensagem foi verificada pelo sistema de antiv�rus e
 acredita-se estar livre de perigo.



[obm-l] Re: [obm-l] Re: [obm-l] Re: [obm-l] Álgebra

2020-08-05 Por tôpico Esdras Muniz
Acho que dá -2. Usa que (x+y)^2=xy e (x/y)^3=1.

Em qua, 5 de ago de 2020 20:07, Anderson Torres <
torres.anderson...@gmail.com> escreveu:

> Em ter., 14 de jul. de 2020 às 23:39, Pacini Bores
>  escreveu:
> >
> > A expressão pedida ao quadrado é igual a 4, sem usar complexos.
> >
> > Pacini
> >
> > Em 14/07/2020 21:50, marcone augusto araújo borges escreveu:
> >
> > Se x^2 +xy + y^2  = 0, com x,y <>0
> > Determinar (x/(x+y))^2019 + (y/(x+y))^2019, sem usar números complexos.
>
> Bem, é meio óbvio que x!=y e x!=-y, senão daria 0.
>
> Podemos supor sem perda de generalidade que x+y=1 (basta dividir x e y
> pela soma)
>
> Assim, temos x+y=1 e x^2+2xy+y^2=1, portanto xy=1.
>
> Assim x e y são zeros do polinômio P(x)=x^2-x+1, e x^2019+y^2019 seria
> calculável mediante uma recorrência.
>
>
> >
> > --
> > Esta mensagem foi verificada pelo sistema de antivírus e
> > acredita-se estar livre de perigo.
> >
> >
> >
> > --
> > Esta mensagem foi verificada pelo sistema de antivírus e
> > acredita-se estar livre de perigo.
>
> --
> Esta mensagem foi verificada pelo sistema de antivírus e
>  acredita-se estar livre de perigo.
>
>
> =
> Instru�ões para entrar na lista, sair da lista e usar a lista em
> http://www.mat.puc-rio.br/~obmlistas/obm-l.html
> =
>

-- 
Esta mensagem foi verificada pelo sistema de antiv�rus e
 acredita-se estar livre de perigo.



[obm-l] Re: [obm-l] Re: [obm-l] Re: [obm-l] Ajuda em teoria dos números

2020-07-24 Por tôpico Prof. Douglas Oliveira
Obrigado Claudio e Esdras, fatoração show


Em sex., 24 de jul. de 2020 às 11:12, Esdras Muniz <
esdrasmunizm...@gmail.com> escreveu:

> Se for solução inteira positiva, acho que só tem 3 e 4. Vc supõe spdg x
> maior ou igual a y, vê que y=1 não tem solução e x=y tb não. Daí, x>y>1.
> Fatorando a expressão, fica: (xy-8-(x-y))(xy-8+(x-y))=15. Como
> (xy-8-(x-y))>(xy-8+(x-y))>-2. Temos que ou (xy-8-(x-y))=1 e (xy-8+(x-y))=15,
> o que não tem soluções inteiras positivas, ou (xy-8-(x-y))=3 e (xy-8+(x-y))=5,
> cujas únicas soluções inteiras são x=4 e y=3.
>
> Em sex, 24 de jul de 2020 10:36, Claudio Buffara <
> claudio.buff...@gmail.com> escreveu:
>
>> Pelo que entendi, a solução é a porção dessa curva algébrica situada no
>> 1o quadrante.
>> Dá pra fazer isso no Wolfram Alpha, com o comando plot (x*y-7)^2 - x^2 -
>> y^2 = 0.
>>
>> []s,
>> Claudio.
>>
>> On Fri, Jul 24, 2020 at 9:58 AM Prof. Douglas Oliveira <
>> profdouglaso.del...@gmail.com> wrote:
>>
>>> Preciso de ajuda para encontrar todas as soluções não negativas da
>>> equação
>>> (xy-7)^2=x^2+y^2.
>>>
>>> Desde já agradeço a ajuda
>>> Douglas Oliveira
>>>
>>> --
>>> Esta mensagem foi verificada pelo sistema de antivírus e
>>> acredita-se estar livre de perigo.
>>
>>
>> --
>> Esta mensagem foi verificada pelo sistema de antivírus e
>> acredita-se estar livre de perigo.
>
>
> --
> Esta mensagem foi verificada pelo sistema de antivírus e
> acredita-se estar livre de perigo.

-- 
Esta mensagem foi verificada pelo sistema de antiv�rus e
 acredita-se estar livre de perigo.



[obm-l] Re: [obm-l] Re: [obm-l] Re: [obm-l] Conjuntos não enumeráveis

2020-06-15 Por tôpico Anderson Torres
Em seg., 15 de jun. de 2020 às 23:31, Israel Meireles Chrisostomo
 escreveu:
>
> usa a bijeção da tangente no intervalo 0 a pi sobre 2

Sim, e o que isso implica? Que a tangente mapeia esse intervalo nos
reais, logo ambos terão o mesmo tamanho - mas onde você demonstrou que
um desses não é enumerável? No máximo você demonstrou que um certo
conjunto tem bijeção com um subconjunto de si mesmo - que é meio que
uma definição de infinito.

>
> Em seg., 15 de jun. de 2020 às 21:38, Anderson Torres 
>  escreveu:
>>
>> Não entendi a última parte.
>>
>> Em dom., 14 de jun. de 2020 Ã s 18:24, Israel Meireles Chrisostomo
>>  escreveu:
>> >
>> >
>> > https://www.overleaf.com/read/cwxhsnctfxcf
>> > Nesse link eu demostro trigonmetricamente que o conjunto dos irracionais 
>> > é não enumerável.
>> > --
>> > Israel Meireles Chrisostomo
>> >
>> > --
>> > Esta mensagem foi verificada pelo sistema de antivírus e
>> > acredita-se estar livre de perigo.
>>
>> --
>> Esta mensagem foi verificada pelo sistema de antivírus e
>>  acredita-se estar livre de perigo.
>>
>>
>> =
>> Instruções para entrar na lista, sair da lista e usar a lista em
>> http://www.mat.puc-rio.br/~obmlistas/obm-l.html
>> =
>
>
>
> --
> Israel Meireles Chrisostomo
>
> --
> Esta mensagem foi verificada pelo sistema de antivírus e
> acredita-se estar livre de perigo.

-- 
Esta mensagem foi verificada pelo sistema de antiv�rus e
 acredita-se estar livre de perigo.


=
Instru��es para entrar na lista, sair da lista e usar a lista em
http://www.mat.puc-rio.br/~obmlistas/obm-l.html
=


[obm-l] Re: [obm-l] Re: [obm-l] Re: [obm-l] Re: [obm-l] Decrescimento de Funções Exponenciais

2020-05-13 Por tôpico Luiz Antonio Rodrigues
Olá, Ralph!
Tudo bem?
Muito obrigado!
Vou acessar os links!
Abraço!
Luiz


Em ter, 12 de mai de 2020 8:35 PM, Ralph Costa Teixeira 
escreveu:

> Bom, o assunto me parece ser "crescimento/decrescimento assintótico"...
> Não consigo pensar num texto para recomendar, mas olhe aqui:
> https://en.wikipedia.org/wiki/Big_O_notation
> E, em especial:
> https://en.wikipedia.org/wiki/Big_O_notation#Little-o_notation
>
> Abraço, Ralph.
>
> On Tue, May 12, 2020 at 7:09 PM Luiz Antonio Rodrigues <
> rodrigue...@gmail.com> wrote:
>
>> Olá, Ralph!
>> Tudo bem?
>> Sim, melhorou muito!
>> Muito obrigado!
>> Então, na função (5), nós temos uma incerteza...
>> Eu não havia percebido isso...
>> Muito interessante...
>> Vou ler mais sobre o assunto...
>> Você conhece algum bom livro que trate disso com mais profundidade?
>> Abraço!
>> Luiz
>>
>>
>> Em ter, 12 de mai de 2020 3:04 PM, Ralph Costa Teixeira <
>> ralp...@gmail.com> escreveu:
>>
>>> O assunto é delicado. Primeiro, precisamos de uma boa definição de
>>> "decresce mais rápido" (a gente diz que as exponenciais decrescem rápido,
>>> mas se você ler **ao pé da letra** isso é falso! A velocidade delas vai
>>> para 0 quando t vai para infinito... ou seja, elas decrescem mito
>>> devagar!?!?). Para esclarecer, suponho que queremos usar esta aqui:
>>>
>>> DEF. f(x) decresce (para 0) mais rápido (quando x vai para +Inf) do que
>>> g(x) quando lim f(x)/g(x) =0 (quando x vai para +Inf).
>>>
>>> Agora sim, você resolve tudo:
>>>
>>> 1) lim h(x)^2/h(x) = 0, portanto h^2 decresce mais rapido que h;
>>> 2) lim g(x)^2/h(x) = lim g(x)/h(x) . g(x) = 0.0=0, portanto g^2 decresce
>>> mais rapido que h;
>>> 3) lim f(x)*g(x)/h(x) = lim f(x) * (g(x)/h(x)) =0 (com f limitada),
>>> portanto fg decresce mais rapido que h;
>>> 4) lim sqrt(h)/h = lim 1/sqrt(h) =+Inf; assim, lim h/sqrt(h) = 0, ou
>>> seja, h decresce mais rapido que sqrt(h);
>>> 5) lim sqrt(g)/h = ??? Nao da para saber. Poderia ser g(x)=1/x^n e
>>> h(x)=1/x. Tomando n<2 ou n>2 podemos obter ambos comportamentos.
>>>
>>> Melhorou?
>>>
>>> Abraço, Ralph.
>>>
>>> On Tue, May 12, 2020 at 9:52 AM Luiz Antonio Rodrigues <
>>> rodrigue...@gmail.com> wrote:
>>>
 Olá, pessoal!

 Bom dia!

 Tudo bem?

 Estou tentando resolver um problema há uns 10 dias.

 Já tentei de tudo e estou com dúvidas.

 O problema é o seguinte:

 São dadas duas funções: h(x) e g(x).

 A função g(x) tende a zero mais rápido do que h(x), quando x tende a
 infinito.

 O problema pede que as seguintes funções sejam comparadas com h(x):


1.

(h(x))^2
2.

(g(x))^2
3.

f(x)*g(x)
4.

sqrt(h(x))
5.

sqrt(g(x))


 A pergunta é: quais dessas funções decrescem mais rápido do que h(x),
 quando x tende a infinito?

 Eu usei, entre outras, as seguintes funções:


 1/ln(x)

 1/x

 1/x^5

 1/e^x


 Utilizei a regra de L’Hospital e descobri que a única função que não
 decresce mais rápido do que h(x) é a (4).

 Também utilizei softwares gráficos e confirmei o meu resultado.

 Só sei que a resposta não está correta, mas ainda não sei qual seria a
 solução.

 Não consigo entender o motivo...

 Será que preciso achar um contra-exemplo?

 Alguém pode me ajudar?

 Muito obrigado!

 Abraços!

 Luiz

 --
 Esta mensagem foi verificada pelo sistema de antivírus e
 acredita-se estar livre de perigo.
>>>
>>>
>>> --
>>> Esta mensagem foi verificada pelo sistema de antivírus e
>>> acredita-se estar livre de perigo.
>>
>>
>> --
>> Esta mensagem foi verificada pelo sistema de antivírus e
>> acredita-se estar livre de perigo.
>
>
> --
> Esta mensagem foi verificada pelo sistema de antivírus e
> acredita-se estar livre de perigo.

-- 
Esta mensagem foi verificada pelo sistema de antiv�rus e
 acredita-se estar livre de perigo.



[obm-l] Re: [obm-l] Re: [obm-l] Re: [obm-l] Decrescimento de Funções Exponenciais

2020-05-12 Por tôpico Ralph Costa Teixeira
Bom, o assunto me parece ser "crescimento/decrescimento assintótico"...
Não consigo pensar num texto para recomendar, mas olhe aqui:
https://en.wikipedia.org/wiki/Big_O_notation
E, em especial:
https://en.wikipedia.org/wiki/Big_O_notation#Little-o_notation

Abraço, Ralph.

On Tue, May 12, 2020 at 7:09 PM Luiz Antonio Rodrigues <
rodrigue...@gmail.com> wrote:

> Olá, Ralph!
> Tudo bem?
> Sim, melhorou muito!
> Muito obrigado!
> Então, na função (5), nós temos uma incerteza...
> Eu não havia percebido isso...
> Muito interessante...
> Vou ler mais sobre o assunto...
> Você conhece algum bom livro que trate disso com mais profundidade?
> Abraço!
> Luiz
>
>
> Em ter, 12 de mai de 2020 3:04 PM, Ralph Costa Teixeira 
> escreveu:
>
>> O assunto é delicado. Primeiro, precisamos de uma boa definição de
>> "decresce mais rápido" (a gente diz que as exponenciais decrescem rápido,
>> mas se você ler **ao pé da letra** isso é falso! A velocidade delas vai
>> para 0 quando t vai para infinito... ou seja, elas decrescem mito
>> devagar!?!?). Para esclarecer, suponho que queremos usar esta aqui:
>>
>> DEF. f(x) decresce (para 0) mais rápido (quando x vai para +Inf) do que
>> g(x) quando lim f(x)/g(x) =0 (quando x vai para +Inf).
>>
>> Agora sim, você resolve tudo:
>>
>> 1) lim h(x)^2/h(x) = 0, portanto h^2 decresce mais rapido que h;
>> 2) lim g(x)^2/h(x) = lim g(x)/h(x) . g(x) = 0.0=0, portanto g^2 decresce
>> mais rapido que h;
>> 3) lim f(x)*g(x)/h(x) = lim f(x) * (g(x)/h(x)) =0 (com f limitada),
>> portanto fg decresce mais rapido que h;
>> 4) lim sqrt(h)/h = lim 1/sqrt(h) =+Inf; assim, lim h/sqrt(h) = 0, ou
>> seja, h decresce mais rapido que sqrt(h);
>> 5) lim sqrt(g)/h = ??? Nao da para saber. Poderia ser g(x)=1/x^n e
>> h(x)=1/x. Tomando n<2 ou n>2 podemos obter ambos comportamentos.
>>
>> Melhorou?
>>
>> Abraço, Ralph.
>>
>> On Tue, May 12, 2020 at 9:52 AM Luiz Antonio Rodrigues <
>> rodrigue...@gmail.com> wrote:
>>
>>> Olá, pessoal!
>>>
>>> Bom dia!
>>>
>>> Tudo bem?
>>>
>>> Estou tentando resolver um problema há uns 10 dias.
>>>
>>> Já tentei de tudo e estou com dúvidas.
>>>
>>> O problema é o seguinte:
>>>
>>> São dadas duas funções: h(x) e g(x).
>>>
>>> A função g(x) tende a zero mais rápido do que h(x), quando x tende a
>>> infinito.
>>>
>>> O problema pede que as seguintes funções sejam comparadas com h(x):
>>>
>>>
>>>1.
>>>
>>>(h(x))^2
>>>2.
>>>
>>>(g(x))^2
>>>3.
>>>
>>>f(x)*g(x)
>>>4.
>>>
>>>sqrt(h(x))
>>>5.
>>>
>>>sqrt(g(x))
>>>
>>>
>>> A pergunta é: quais dessas funções decrescem mais rápido do que h(x),
>>> quando x tende a infinito?
>>>
>>> Eu usei, entre outras, as seguintes funções:
>>>
>>>
>>> 1/ln(x)
>>>
>>> 1/x
>>>
>>> 1/x^5
>>>
>>> 1/e^x
>>>
>>>
>>> Utilizei a regra de L’Hospital e descobri que a única função que não
>>> decresce mais rápido do que h(x) é a (4).
>>>
>>> Também utilizei softwares gráficos e confirmei o meu resultado.
>>>
>>> Só sei que a resposta não está correta, mas ainda não sei qual seria a
>>> solução.
>>>
>>> Não consigo entender o motivo...
>>>
>>> Será que preciso achar um contra-exemplo?
>>>
>>> Alguém pode me ajudar?
>>>
>>> Muito obrigado!
>>>
>>> Abraços!
>>>
>>> Luiz
>>>
>>> --
>>> Esta mensagem foi verificada pelo sistema de antivírus e
>>> acredita-se estar livre de perigo.
>>
>>
>> --
>> Esta mensagem foi verificada pelo sistema de antivírus e
>> acredita-se estar livre de perigo.
>
>
> --
> Esta mensagem foi verificada pelo sistema de antivírus e
> acredita-se estar livre de perigo.

-- 
Esta mensagem foi verificada pelo sistema de antiv�rus e
 acredita-se estar livre de perigo.



[obm-l] Re: [obm-l] Re: [obm-l] Re: [obm-l] Re: [obm-l] Teoria dos números

2020-03-22 Por tôpico Israel Meireles Chrisostomo
O meu sonho tmbm é esse kk

Em dom., 22 de mar. de 2020 às 13:22, Israel Meireles Chrisostomo <
israelmchrisost...@gmail.com> escreveu:

> vc é engenheiro?
>
> Em dom., 22 de mar. de 2020 às 13:19, Israel Meireles Chrisostomo <
> israelmchrisost...@gmail.com> escreveu:
>
>> mas vc possui algum graduação ?
>>
>> Em dom., 22 de mar. de 2020 às 13:00, Pedro José 
>> escreveu:
>>
>>> Boa tarde!
>>> Perfeita a sua correção.
>>> Quanto ao questionamento, nem tenho formação em matemática, meu sonho é
>>> cursar no IMPA ao me aposentar. Sou pitaqueiro. Ouço um assunto que não
>>> conheço, tento aprendê-lo. Na verdade, gosto de matemática. Talvez seja ela
>>> o "Mundo das ideias", o mundo ideal, a qual Platão se referiu.
>>> Saudações,
>>> PJMS
>>>
>>> Em dom, 22 de mar de 2020 12:25, Israel Meireles Chrisostomo <
>>> israelmchrisost...@gmail.com> escreveu:
>>>
 Acho q tem uma ´pequena correção no seguinte passo "4k+1. Pegando os
 fatores (4n-1)^2 e (4n+1)^2, teremos que 2^6 |p(n) para qualquer n=4k+1."O
 correto seria "Para n=4k+1.Pegando os fatores (n-1)^2 e (n+1)^2"

 Em dom., 22 de mar. de 2020 às 10:14, Israel Meireles Chrisostomo <
 israelmchrisost...@gmail.com> escreveu:

> Primeiramente obrigado pela solução.Mas Pedro, tenho uma pergunta :   o
> sr. é professor de Matemática?
>
> Em dom., 22 de mar. de 2020 às 01:34, Pedro José 
> escreveu:
>
>> Bom dia!
>> Dei uma mancada.
>> O expoente de 3 é 3 e não 2.
>> Retornando às classes mod 3.
>> Ao último fator é côngruo à (n-1)*n
>> Para n=3k aparece outro fator e 3^3|p(n), n=3k.
>> n=3k+1, tenho (n-1)^2 e (n-1), 3^3|p(n), n=3k+1
>> n=3k+2, tenho(n-2)^2 é (n+1)^2, 3^3|p(n), n=3k+2,
>> Logo 3^3|p(n) para todo n inteiro.
>> D>=2^6*3^3*5. Mas D<=2^6*3^3*5, então D=8640
>> Desculpem-me pelo erro.
>> Saudações,
>> PJMS.
>>
>>
>>
>> Em sáb, 21 de mar de 2020 13:20, Pedro José 
>> escreveu:
>>
>>> Boa tarde!
>>> Nem carece método numérico.
>>> Para n=1 ou n=0 ou n=2 temos que qualquer inteiro divide o polinômio
>>> p(n)=(n-2)^2*(n-1)^2*n^2*(n+1)^2*(4n^2-4n-9)
>>>
>>> p(3)=8640
>>> p(4)=561600 então (p(3),p(4))=8640=2^6*3^3*5.
>>> Seja D o maior inteiro que divide p(n) para todo n inteiro, D<=8640
>>> Vamos pegar as classes de equivalência mod 4. Seja k um inteiro.
>>> Para 4k temos que n^2= 16k^2 e (n-2) é par logo (n-2)^2= 4s^2 com s
>>> inteiro. Logo 2^6 divide p(n) para qualquer n =4k.
>>> 4k+1. Pegando os fatores (4n-1)^2 e (4n+1)^2, teremos que 2^6 |p(n)
>>> para qualquer n=4k+1.
>>> 4k+2. Pegando (n-2)^2 e n^2, teremos que 2^6|p(n) para qualquer
>>> n=4k+2
>>> 4k+3, pegando os mesmos fatores de 4k+1, 2^6|p(n) para n=4k+3.
>>> Portanto 2^6|p(n) para qualquer inteiro
>>> Agora classes de equivalência mod 3
>>> 3k, pegando n^2, 3^2|p(n) para n=3k
>>> 3k+1, pegando (n-1)^2; 3^2| p(n), n=3k+1
>>> 3k+2, pegando (n-2)^2, 3^2| p(n), n=3k+2
>>> Daí 3^2|p(n) para qualquer n inteiro.
>>> Classes de equivalência mod 5.
>>> 5k, n^2 , 5 |p(n), n =5
>>> 5k +1, (n-1)^2, 5|p(n), n=5k+1
>>> 5k+2, (n-2)^2, 5|p(n), n=5k+2
>>> 5k+3, (4n^2-4n-9)=(100k^2-100k+15)
>>> 5|p(n), n=5k+3
>>> 5k+4, (n+1)^2, 5|p(n) , n=5k+4.
>>> Então 5|p(n) para todo inteiro
>>> D>=2^6*3^2×*5
>>> Mas D<=2^6*3^2*5, logo D=8640
>>>
>>> Saudações,
>>> PJMS
>>>
>>> Em sáb, 21 de mar de 2020 04:39, Pedro José 
>>> escreveu:
>>>
 Bom dia!
 Falta de novo, em seu questionamento, informar que n é inteiro ou
 natural e colocar a condição para qualquer valor de n. Chamando o 
 polinômio
 de p(n)
 Para n=0, 1 ou 2, qualquer inteiro divide.
 Faria mdc(p(3),p(4))= A1
 Se der "pequeno", com poucos fatores primos e expoentes pequenos.
 Paro em A1, se não.
 (p(5),A1)=A2 uso o mesmo critério de parar
 (p(6),A2)=A3 até parar em:
 Ai=(p(i+3),A(i-1)).
 Aí faço o polinômio módfi^xi, onde fi é um fator primo de Aí e xi
 seu expoente. verifico se para cada resíduon= 1, 2...fi^n-1 se P(n)=0 
 mod
 fi^si
 Se falhar diminuto xi em 1 e repito o teste para todos resíduos de
 fi^(xi-1)-1 até um dado xki em que todos os p(resíduos) foram 
 equivalente a
 zero módulo fi^xki ou quando fizer para o expoente 1  e não zerar para
 todos resíduos de fi, quando o fator será descartado.
 Depois repito para cada fator primo f e seu respectivo expoente.
 Ao final D = Produtório de cada fator fi elevado ao expoente xki
 que zerou p(n) mod fi^xki para todos os resíduos, descartando os fí em 
 que
 xji chegou a 1 e não atendeu ou considerando nesse caso xki=0.

 Mas resolveria por método numérico.
 Depois poste sua solução.

 

[obm-l] Re: [obm-l] Re: [obm-l] Re: [obm-l] Re: [obm-l] Teoria dos números

2020-03-22 Por tôpico Israel Meireles Chrisostomo
vc é engenheiro?

Em dom., 22 de mar. de 2020 às 13:19, Israel Meireles Chrisostomo <
israelmchrisost...@gmail.com> escreveu:

> mas vc possui algum graduação ?
>
> Em dom., 22 de mar. de 2020 às 13:00, Pedro José 
> escreveu:
>
>> Boa tarde!
>> Perfeita a sua correção.
>> Quanto ao questionamento, nem tenho formação em matemática, meu sonho é
>> cursar no IMPA ao me aposentar. Sou pitaqueiro. Ouço um assunto que não
>> conheço, tento aprendê-lo. Na verdade, gosto de matemática. Talvez seja ela
>> o "Mundo das ideias", o mundo ideal, a qual Platão se referiu.
>> Saudações,
>> PJMS
>>
>> Em dom, 22 de mar de 2020 12:25, Israel Meireles Chrisostomo <
>> israelmchrisost...@gmail.com> escreveu:
>>
>>> Acho q tem uma ´pequena correção no seguinte passo "4k+1. Pegando os
>>> fatores (4n-1)^2 e (4n+1)^2, teremos que 2^6 |p(n) para qualquer n=4k+1."O
>>> correto seria "Para n=4k+1.Pegando os fatores (n-1)^2 e (n+1)^2"
>>>
>>> Em dom., 22 de mar. de 2020 às 10:14, Israel Meireles Chrisostomo <
>>> israelmchrisost...@gmail.com> escreveu:
>>>
 Primeiramente obrigado pela solução.Mas Pedro, tenho uma pergunta :   o
 sr. é professor de Matemática?

 Em dom., 22 de mar. de 2020 às 01:34, Pedro José 
 escreveu:

> Bom dia!
> Dei uma mancada.
> O expoente de 3 é 3 e não 2.
> Retornando às classes mod 3.
> Ao último fator é côngruo à (n-1)*n
> Para n=3k aparece outro fator e 3^3|p(n), n=3k.
> n=3k+1, tenho (n-1)^2 e (n-1), 3^3|p(n), n=3k+1
> n=3k+2, tenho(n-2)^2 é (n+1)^2, 3^3|p(n), n=3k+2,
> Logo 3^3|p(n) para todo n inteiro.
> D>=2^6*3^3*5. Mas D<=2^6*3^3*5, então D=8640
> Desculpem-me pelo erro.
> Saudações,
> PJMS.
>
>
>
> Em sáb, 21 de mar de 2020 13:20, Pedro José 
> escreveu:
>
>> Boa tarde!
>> Nem carece método numérico.
>> Para n=1 ou n=0 ou n=2 temos que qualquer inteiro divide o polinômio
>> p(n)=(n-2)^2*(n-1)^2*n^2*(n+1)^2*(4n^2-4n-9)
>>
>> p(3)=8640
>> p(4)=561600 então (p(3),p(4))=8640=2^6*3^3*5.
>> Seja D o maior inteiro que divide p(n) para todo n inteiro, D<=8640
>> Vamos pegar as classes de equivalência mod 4. Seja k um inteiro.
>> Para 4k temos que n^2= 16k^2 e (n-2) é par logo (n-2)^2= 4s^2 com s
>> inteiro. Logo 2^6 divide p(n) para qualquer n =4k.
>> 4k+1. Pegando os fatores (4n-1)^2 e (4n+1)^2, teremos que 2^6 |p(n)
>> para qualquer n=4k+1.
>> 4k+2. Pegando (n-2)^2 e n^2, teremos que 2^6|p(n) para qualquer n=4k+2
>> 4k+3, pegando os mesmos fatores de 4k+1, 2^6|p(n) para n=4k+3.
>> Portanto 2^6|p(n) para qualquer inteiro
>> Agora classes de equivalência mod 3
>> 3k, pegando n^2, 3^2|p(n) para n=3k
>> 3k+1, pegando (n-1)^2; 3^2| p(n), n=3k+1
>> 3k+2, pegando (n-2)^2, 3^2| p(n), n=3k+2
>> Daí 3^2|p(n) para qualquer n inteiro.
>> Classes de equivalência mod 5.
>> 5k, n^2 , 5 |p(n), n =5
>> 5k +1, (n-1)^2, 5|p(n), n=5k+1
>> 5k+2, (n-2)^2, 5|p(n), n=5k+2
>> 5k+3, (4n^2-4n-9)=(100k^2-100k+15)
>> 5|p(n), n=5k+3
>> 5k+4, (n+1)^2, 5|p(n) , n=5k+4.
>> Então 5|p(n) para todo inteiro
>> D>=2^6*3^2×*5
>> Mas D<=2^6*3^2*5, logo D=8640
>>
>> Saudações,
>> PJMS
>>
>> Em sáb, 21 de mar de 2020 04:39, Pedro José 
>> escreveu:
>>
>>> Bom dia!
>>> Falta de novo, em seu questionamento, informar que n é inteiro ou
>>> natural e colocar a condição para qualquer valor de n. Chamando o 
>>> polinômio
>>> de p(n)
>>> Para n=0, 1 ou 2, qualquer inteiro divide.
>>> Faria mdc(p(3),p(4))= A1
>>> Se der "pequeno", com poucos fatores primos e expoentes pequenos.
>>> Paro em A1, se não.
>>> (p(5),A1)=A2 uso o mesmo critério de parar
>>> (p(6),A2)=A3 até parar em:
>>> Ai=(p(i+3),A(i-1)).
>>> Aí faço o polinômio módfi^xi, onde fi é um fator primo de Aí e xi
>>> seu expoente. verifico se para cada resíduon= 1, 2...fi^n-1 se P(n)=0 
>>> mod
>>> fi^si
>>> Se falhar diminuto xi em 1 e repito o teste para todos resíduos de
>>> fi^(xi-1)-1 até um dado xki em que todos os p(resíduos) foram 
>>> equivalente a
>>> zero módulo fi^xki ou quando fizer para o expoente 1  e não zerar para
>>> todos resíduos de fi, quando o fator será descartado.
>>> Depois repito para cada fator primo f e seu respectivo expoente.
>>> Ao final D = Produtório de cada fator fi elevado ao expoente xki que
>>> zerou p(n) mod fi^xki para todos os resíduos, descartando os fí em que 
>>> xji
>>> chegou a 1 e não atendeu ou considerando nesse caso xki=0.
>>>
>>> Mas resolveria por método numérico.
>>> Depois poste sua solução.
>>>
>>> Saudações,
>>> PJMS.
>>>
>>>
>>>
>>>
>>> Em sex, 20 de mar de 2020 12:42, Israel Meireles Chrisostomo <
>>> israelmchrisost...@gmail.com> escreveu:
>>>
 Qual o maior inteiro que divide (n - 2)^2 (n - 1)^2 n^2 (n + 1)^2
 (4 n^2 

[obm-l] Re: [obm-l] Re: [obm-l] Re: [obm-l] Re: [obm-l] Teoria dos números

2020-03-22 Por tôpico Israel Meireles Chrisostomo
mas vc possui algum graduação ?

Em dom., 22 de mar. de 2020 às 13:00, Pedro José 
escreveu:

> Boa tarde!
> Perfeita a sua correção.
> Quanto ao questionamento, nem tenho formação em matemática, meu sonho é
> cursar no IMPA ao me aposentar. Sou pitaqueiro. Ouço um assunto que não
> conheço, tento aprendê-lo. Na verdade, gosto de matemática. Talvez seja ela
> o "Mundo das ideias", o mundo ideal, a qual Platão se referiu.
> Saudações,
> PJMS
>
> Em dom, 22 de mar de 2020 12:25, Israel Meireles Chrisostomo <
> israelmchrisost...@gmail.com> escreveu:
>
>> Acho q tem uma ´pequena correção no seguinte passo "4k+1. Pegando os
>> fatores (4n-1)^2 e (4n+1)^2, teremos que 2^6 |p(n) para qualquer n=4k+1."O
>> correto seria "Para n=4k+1.Pegando os fatores (n-1)^2 e (n+1)^2"
>>
>> Em dom., 22 de mar. de 2020 às 10:14, Israel Meireles Chrisostomo <
>> israelmchrisost...@gmail.com> escreveu:
>>
>>> Primeiramente obrigado pela solução.Mas Pedro, tenho uma pergunta :   o
>>> sr. é professor de Matemática?
>>>
>>> Em dom., 22 de mar. de 2020 às 01:34, Pedro José 
>>> escreveu:
>>>
 Bom dia!
 Dei uma mancada.
 O expoente de 3 é 3 e não 2.
 Retornando às classes mod 3.
 Ao último fator é côngruo à (n-1)*n
 Para n=3k aparece outro fator e 3^3|p(n), n=3k.
 n=3k+1, tenho (n-1)^2 e (n-1), 3^3|p(n), n=3k+1
 n=3k+2, tenho(n-2)^2 é (n+1)^2, 3^3|p(n), n=3k+2,
 Logo 3^3|p(n) para todo n inteiro.
 D>=2^6*3^3*5. Mas D<=2^6*3^3*5, então D=8640
 Desculpem-me pelo erro.
 Saudações,
 PJMS.



 Em sáb, 21 de mar de 2020 13:20, Pedro José 
 escreveu:

> Boa tarde!
> Nem carece método numérico.
> Para n=1 ou n=0 ou n=2 temos que qualquer inteiro divide o polinômio
> p(n)=(n-2)^2*(n-1)^2*n^2*(n+1)^2*(4n^2-4n-9)
>
> p(3)=8640
> p(4)=561600 então (p(3),p(4))=8640=2^6*3^3*5.
> Seja D o maior inteiro que divide p(n) para todo n inteiro, D<=8640
> Vamos pegar as classes de equivalência mod 4. Seja k um inteiro.
> Para 4k temos que n^2= 16k^2 e (n-2) é par logo (n-2)^2= 4s^2 com s
> inteiro. Logo 2^6 divide p(n) para qualquer n =4k.
> 4k+1. Pegando os fatores (4n-1)^2 e (4n+1)^2, teremos que 2^6 |p(n)
> para qualquer n=4k+1.
> 4k+2. Pegando (n-2)^2 e n^2, teremos que 2^6|p(n) para qualquer n=4k+2
> 4k+3, pegando os mesmos fatores de 4k+1, 2^6|p(n) para n=4k+3.
> Portanto 2^6|p(n) para qualquer inteiro
> Agora classes de equivalência mod 3
> 3k, pegando n^2, 3^2|p(n) para n=3k
> 3k+1, pegando (n-1)^2; 3^2| p(n), n=3k+1
> 3k+2, pegando (n-2)^2, 3^2| p(n), n=3k+2
> Daí 3^2|p(n) para qualquer n inteiro.
> Classes de equivalência mod 5.
> 5k, n^2 , 5 |p(n), n =5
> 5k +1, (n-1)^2, 5|p(n), n=5k+1
> 5k+2, (n-2)^2, 5|p(n), n=5k+2
> 5k+3, (4n^2-4n-9)=(100k^2-100k+15)
> 5|p(n), n=5k+3
> 5k+4, (n+1)^2, 5|p(n) , n=5k+4.
> Então 5|p(n) para todo inteiro
> D>=2^6*3^2×*5
> Mas D<=2^6*3^2*5, logo D=8640
>
> Saudações,
> PJMS
>
> Em sáb, 21 de mar de 2020 04:39, Pedro José 
> escreveu:
>
>> Bom dia!
>> Falta de novo, em seu questionamento, informar que n é inteiro ou
>> natural e colocar a condição para qualquer valor de n. Chamando o 
>> polinômio
>> de p(n)
>> Para n=0, 1 ou 2, qualquer inteiro divide.
>> Faria mdc(p(3),p(4))= A1
>> Se der "pequeno", com poucos fatores primos e expoentes pequenos.
>> Paro em A1, se não.
>> (p(5),A1)=A2 uso o mesmo critério de parar
>> (p(6),A2)=A3 até parar em:
>> Ai=(p(i+3),A(i-1)).
>> Aí faço o polinômio módfi^xi, onde fi é um fator primo de Aí e xi seu
>> expoente. verifico se para cada resíduon= 1, 2...fi^n-1 se P(n)=0 mod 
>> fi^si
>> Se falhar diminuto xi em 1 e repito o teste para todos resíduos de
>> fi^(xi-1)-1 até um dado xki em que todos os p(resíduos) foram 
>> equivalente a
>> zero módulo fi^xki ou quando fizer para o expoente 1  e não zerar para
>> todos resíduos de fi, quando o fator será descartado.
>> Depois repito para cada fator primo f e seu respectivo expoente.
>> Ao final D = Produtório de cada fator fi elevado ao expoente xki que
>> zerou p(n) mod fi^xki para todos os resíduos, descartando os fí em que 
>> xji
>> chegou a 1 e não atendeu ou considerando nesse caso xki=0.
>>
>> Mas resolveria por método numérico.
>> Depois poste sua solução.
>>
>> Saudações,
>> PJMS.
>>
>>
>>
>>
>> Em sex, 20 de mar de 2020 12:42, Israel Meireles Chrisostomo <
>> israelmchrisost...@gmail.com> escreveu:
>>
>>> Qual o maior inteiro que divide (n - 2)^2 (n - 1)^2 n^2 (n + 1)^2 (4
>>> n^2 - 4 n - 9))?
>>> Eu sei resolver esse problema com meu algoritmo, porém gostaria de
>>> saber como os colegas o resolveriam.
>>> --
>>> Israel Meireles Chrisostomo
>>>
>>> --
>>> Esta mensagem foi verificada pelo sistema de antivírus e

[obm-l] Re: [obm-l] Re: [obm-l] Re: [obm-l] Teoria dos números

2020-03-22 Por tôpico Pedro José
Boa tarde!
Perfeita a sua correção.
Quanto ao questionamento, nem tenho formação em matemática, meu sonho é
cursar no IMPA ao me aposentar. Sou pitaqueiro. Ouço um assunto que não
conheço, tento aprendê-lo. Na verdade, gosto de matemática. Talvez seja ela
o "Mundo das ideias", o mundo ideal, a qual Platão se referiu.
Saudações,
PJMS

Em dom, 22 de mar de 2020 12:25, Israel Meireles Chrisostomo <
israelmchrisost...@gmail.com> escreveu:

> Acho q tem uma ´pequena correção no seguinte passo "4k+1. Pegando os
> fatores (4n-1)^2 e (4n+1)^2, teremos que 2^6 |p(n) para qualquer n=4k+1."O
> correto seria "Para n=4k+1.Pegando os fatores (n-1)^2 e (n+1)^2"
>
> Em dom., 22 de mar. de 2020 às 10:14, Israel Meireles Chrisostomo <
> israelmchrisost...@gmail.com> escreveu:
>
>> Primeiramente obrigado pela solução.Mas Pedro, tenho uma pergunta :   o
>> sr. é professor de Matemática?
>>
>> Em dom., 22 de mar. de 2020 às 01:34, Pedro José 
>> escreveu:
>>
>>> Bom dia!
>>> Dei uma mancada.
>>> O expoente de 3 é 3 e não 2.
>>> Retornando às classes mod 3.
>>> Ao último fator é côngruo à (n-1)*n
>>> Para n=3k aparece outro fator e 3^3|p(n), n=3k.
>>> n=3k+1, tenho (n-1)^2 e (n-1), 3^3|p(n), n=3k+1
>>> n=3k+2, tenho(n-2)^2 é (n+1)^2, 3^3|p(n), n=3k+2,
>>> Logo 3^3|p(n) para todo n inteiro.
>>> D>=2^6*3^3*5. Mas D<=2^6*3^3*5, então D=8640
>>> Desculpem-me pelo erro.
>>> Saudações,
>>> PJMS.
>>>
>>>
>>>
>>> Em sáb, 21 de mar de 2020 13:20, Pedro José 
>>> escreveu:
>>>
 Boa tarde!
 Nem carece método numérico.
 Para n=1 ou n=0 ou n=2 temos que qualquer inteiro divide o polinômio
 p(n)=(n-2)^2*(n-1)^2*n^2*(n+1)^2*(4n^2-4n-9)

 p(3)=8640
 p(4)=561600 então (p(3),p(4))=8640=2^6*3^3*5.
 Seja D o maior inteiro que divide p(n) para todo n inteiro, D<=8640
 Vamos pegar as classes de equivalência mod 4. Seja k um inteiro.
 Para 4k temos que n^2= 16k^2 e (n-2) é par logo (n-2)^2= 4s^2 com s
 inteiro. Logo 2^6 divide p(n) para qualquer n =4k.
 4k+1. Pegando os fatores (4n-1)^2 e (4n+1)^2, teremos que 2^6 |p(n)
 para qualquer n=4k+1.
 4k+2. Pegando (n-2)^2 e n^2, teremos que 2^6|p(n) para qualquer n=4k+2
 4k+3, pegando os mesmos fatores de 4k+1, 2^6|p(n) para n=4k+3.
 Portanto 2^6|p(n) para qualquer inteiro
 Agora classes de equivalência mod 3
 3k, pegando n^2, 3^2|p(n) para n=3k
 3k+1, pegando (n-1)^2; 3^2| p(n), n=3k+1
 3k+2, pegando (n-2)^2, 3^2| p(n), n=3k+2
 Daí 3^2|p(n) para qualquer n inteiro.
 Classes de equivalência mod 5.
 5k, n^2 , 5 |p(n), n =5
 5k +1, (n-1)^2, 5|p(n), n=5k+1
 5k+2, (n-2)^2, 5|p(n), n=5k+2
 5k+3, (4n^2-4n-9)=(100k^2-100k+15)
 5|p(n), n=5k+3
 5k+4, (n+1)^2, 5|p(n) , n=5k+4.
 Então 5|p(n) para todo inteiro
 D>=2^6*3^2×*5
 Mas D<=2^6*3^2*5, logo D=8640

 Saudações,
 PJMS

 Em sáb, 21 de mar de 2020 04:39, Pedro José 
 escreveu:

> Bom dia!
> Falta de novo, em seu questionamento, informar que n é inteiro ou
> natural e colocar a condição para qualquer valor de n. Chamando o 
> polinômio
> de p(n)
> Para n=0, 1 ou 2, qualquer inteiro divide.
> Faria mdc(p(3),p(4))= A1
> Se der "pequeno", com poucos fatores primos e expoentes pequenos. Paro
> em A1, se não.
> (p(5),A1)=A2 uso o mesmo critério de parar
> (p(6),A2)=A3 até parar em:
> Ai=(p(i+3),A(i-1)).
> Aí faço o polinômio módfi^xi, onde fi é um fator primo de Aí e xi seu
> expoente. verifico se para cada resíduon= 1, 2...fi^n-1 se P(n)=0 mod 
> fi^si
> Se falhar diminuto xi em 1 e repito o teste para todos resíduos de
> fi^(xi-1)-1 até um dado xki em que todos os p(resíduos) foram equivalente 
> a
> zero módulo fi^xki ou quando fizer para o expoente 1  e não zerar para
> todos resíduos de fi, quando o fator será descartado.
> Depois repito para cada fator primo f e seu respectivo expoente.
> Ao final D = Produtório de cada fator fi elevado ao expoente xki que
> zerou p(n) mod fi^xki para todos os resíduos, descartando os fí em que xji
> chegou a 1 e não atendeu ou considerando nesse caso xki=0.
>
> Mas resolveria por método numérico.
> Depois poste sua solução.
>
> Saudações,
> PJMS.
>
>
>
>
> Em sex, 20 de mar de 2020 12:42, Israel Meireles Chrisostomo <
> israelmchrisost...@gmail.com> escreveu:
>
>> Qual o maior inteiro que divide (n - 2)^2 (n - 1)^2 n^2 (n + 1)^2 (4
>> n^2 - 4 n - 9))?
>> Eu sei resolver esse problema com meu algoritmo, porém gostaria de
>> saber como os colegas o resolveriam.
>> --
>> Israel Meireles Chrisostomo
>>
>> --
>> Esta mensagem foi verificada pelo sistema de antivírus e
>> acredita-se estar livre de perigo.
>
>
>>> --
>>> Esta mensagem foi verificada pelo sistema de antivírus e
>>> acredita-se estar livre de perigo.
>>
>>
>>
>> --
>> Israel Meireles Chrisostomo
>>
>
>
> --
> Israel Meireles 

[obm-l] Re: [obm-l] Re: [obm-l] Re: [obm-l] Re: [obm-l] Re: [obm-l] Re: [obm-l] Teoria dos números

2020-03-17 Por tôpico Pedro José
Boa noite!
Aí, como dizia minha falecida vó, são outros quinhentos.
Como nas propostas anteriores n era natural. Vamos seguir nessa linha, se
não for reformule o problema.
Seja f(n)=   n (427 - 90n - 70n^2 + 45n^3 + 18n^4)
f(0)=0 qualquer natural divide, portanto, é indiferente.
f(1)= 330
f(2)= 1230
É fácil verificar que mdc(330,1230)=30 então D<=30, onde D é o máximo
inteiro que divide f(n) para todo n natural.
f(n) = 7n +5n^4 + 8 n^5 mod 10.
f(0)=0 mod10
f(1)= 20 = 0 mod10
f(2)= 350= 0 mod10
f(3)= 2370 = 0 mod10
f(4)= 9500 = 0 mod10
f(5)= 28160 = 0 mod10
f(6)=68730 = 0 mod10
f(7)=146510 = 0 mod10
f(8)=282680 = 0 mod10
f(9)=505260 = 0 mod10
logo 10 | f(n) para qualquer n natural.

f(n) = n -n^3 mod 3
f(0) = 0 mod 3
f(1) = 0 mod 3
f(2)= -6 = 0 mod 3
logo 3| f(n) para todo n natural
então D = 30.

Saudações,
PJMS



Em ter., 17 de mar. de 2020 às 11:57, Israel Meireles Chrisostomo <
israelmchrisost...@gmail.com> escreveu:

> Sim é isso q eu quis dizer
>
> Em ter, 17 de mar de 2020 11:12, Carlos Gustavo Tamm de Araujo Moreira <
> g...@impa.br> escreveu:
>
>> Acho que a pergunta deve ser qual é o maior inteiro positivo que divide
>> essa expressão para todo valor de n ao mesmo tempo.
>>
>> On Tue, Mar 17, 2020 at 6:58 AM Pedro José  wrote:
>>
>>> Bom dia!
>>> Se você considerar a expressão n(427-90n-70n^2+45n^3+18n^4)
>>> D=|n(427-90n-70n^2+45n^3+18n^4)|
>>> Por exemplo, n=1
>>> D=330.
>>> Agora se liberar n para variar D tende a oo.
>>>
>>> Se n for raiz da expressão, também tende a oi, pois qualquer inteiro
>>> divide 0.
>>>
>>>
>>> Em seg, 16 de mar de 2020 22:16, Israel Meireles Chrisostomo <
>>> israelmchrisost...@gmail.com> escreveu:
>>>
 não entendi

 Em seg., 16 de mar. de 2020 às 22:01, Pedro José 
 escreveu:

> Para um dado n é o módulo do valor da expressão.
>
> Em seg, 16 de mar de 2020 21:49, Pedro José 
> escreveu:
>
>> Boa noite!
>> O módulo dessa expressão tende a oo. Não existe máximo.
>> Saudações,
>> PJMS
>>
>> Em seg, 16 de mar de 2020 20:36, Israel Meireles Chrisostomo <
>> israelmchrisost...@gmail.com> escreveu:
>>
>>> Qual é o maior inteiro que divide  n (427 - 90n - 70n^2 + 45n^3 +
>>> 18n^4)?
>>>
>>> --
>>> Israel Meireles Chrisostomo
>>>
>>> --
>>> Esta mensagem foi verificada pelo sistema de antivírus e
>>> acredita-se estar livre de perigo.
>>
>>
> --
> Esta mensagem foi verificada pelo sistema de antivírus e
> acredita-se estar livre de perigo.



 --
 Israel Meireles Chrisostomo

 --
 Esta mensagem foi verificada pelo sistema de antivírus e
 acredita-se estar livre de perigo.
>>>
>>>
>>> --
>>> Esta mensagem foi verificada pelo sistema de antivírus e
>>> acredita-se estar livre de perigo.
>>
>>
>> --
>> Esta mensagem foi verificada pelo sistema de antivírus e
>> acredita-se estar livre de perigo.
>
>
> --
> Esta mensagem foi verificada pelo sistema de antivírus e
> acredita-se estar livre de perigo.

-- 
Esta mensagem foi verificada pelo sistema de antiv�rus e
 acredita-se estar livre de perigo.



[obm-l] Re: [obm-l] Re: [obm-l] Re: [obm-l] Re: [obm-l] Re: [obm-l] Teoria dos números

2020-03-17 Por tôpico Israel Meireles Chrisostomo
Sim é isso q eu quis dizer

Em ter, 17 de mar de 2020 11:12, Carlos Gustavo Tamm de Araujo Moreira <
g...@impa.br> escreveu:

> Acho que a pergunta deve ser qual é o maior inteiro positivo que divide
> essa expressão para todo valor de n ao mesmo tempo.
>
> On Tue, Mar 17, 2020 at 6:58 AM Pedro José  wrote:
>
>> Bom dia!
>> Se você considerar a expressão n(427-90n-70n^2+45n^3+18n^4)
>> D=|n(427-90n-70n^2+45n^3+18n^4)|
>> Por exemplo, n=1
>> D=330.
>> Agora se liberar n para variar D tende a oo.
>>
>> Se n for raiz da expressão, também tende a oi, pois qualquer inteiro
>> divide 0.
>>
>>
>> Em seg, 16 de mar de 2020 22:16, Israel Meireles Chrisostomo <
>> israelmchrisost...@gmail.com> escreveu:
>>
>>> não entendi
>>>
>>> Em seg., 16 de mar. de 2020 às 22:01, Pedro José 
>>> escreveu:
>>>
 Para um dado n é o módulo do valor da expressão.

 Em seg, 16 de mar de 2020 21:49, Pedro José 
 escreveu:

> Boa noite!
> O módulo dessa expressão tende a oo. Não existe máximo.
> Saudações,
> PJMS
>
> Em seg, 16 de mar de 2020 20:36, Israel Meireles Chrisostomo <
> israelmchrisost...@gmail.com> escreveu:
>
>> Qual é o maior inteiro que divide  n (427 - 90n - 70n^2 + 45n^3 +
>> 18n^4)?
>>
>> --
>> Israel Meireles Chrisostomo
>>
>> --
>> Esta mensagem foi verificada pelo sistema de antivírus e
>> acredita-se estar livre de perigo.
>
>
 --
 Esta mensagem foi verificada pelo sistema de antivírus e
 acredita-se estar livre de perigo.
>>>
>>>
>>>
>>> --
>>> Israel Meireles Chrisostomo
>>>
>>> --
>>> Esta mensagem foi verificada pelo sistema de antivírus e
>>> acredita-se estar livre de perigo.
>>
>>
>> --
>> Esta mensagem foi verificada pelo sistema de antivírus e
>> acredita-se estar livre de perigo.
>
>
> --
> Esta mensagem foi verificada pelo sistema de antivírus e
> acredita-se estar livre de perigo.

-- 
Esta mensagem foi verificada pelo sistema de antiv�rus e
 acredita-se estar livre de perigo.



[obm-l] Re: [obm-l] Re: [obm-l] Re: [obm-l] Re: [obm-l] Teoria dos números

2020-03-17 Por tôpico Carlos Gustavo Tamm de Araujo Moreira
Acho que a pergunta deve ser qual é o maior inteiro positivo que divide
essa expressão para todo valor de n ao mesmo tempo.

On Tue, Mar 17, 2020 at 6:58 AM Pedro José  wrote:

> Bom dia!
> Se você considerar a expressão n(427-90n-70n^2+45n^3+18n^4)
> D=|n(427-90n-70n^2+45n^3+18n^4)|
> Por exemplo, n=1
> D=330.
> Agora se liberar n para variar D tende a oo.
>
> Se n for raiz da expressão, também tende a oi, pois qualquer inteiro
> divide 0.
>
>
> Em seg, 16 de mar de 2020 22:16, Israel Meireles Chrisostomo <
> israelmchrisost...@gmail.com> escreveu:
>
>> não entendi
>>
>> Em seg., 16 de mar. de 2020 às 22:01, Pedro José 
>> escreveu:
>>
>>> Para um dado n é o módulo do valor da expressão.
>>>
>>> Em seg, 16 de mar de 2020 21:49, Pedro José 
>>> escreveu:
>>>
 Boa noite!
 O módulo dessa expressão tende a oo. Não existe máximo.
 Saudações,
 PJMS

 Em seg, 16 de mar de 2020 20:36, Israel Meireles Chrisostomo <
 israelmchrisost...@gmail.com> escreveu:

> Qual é o maior inteiro que divide  n (427 - 90n - 70n^2 + 45n^3 +
> 18n^4)?
>
> --
> Israel Meireles Chrisostomo
>
> --
> Esta mensagem foi verificada pelo sistema de antivírus e
> acredita-se estar livre de perigo.


>>> --
>>> Esta mensagem foi verificada pelo sistema de antivírus e
>>> acredita-se estar livre de perigo.
>>
>>
>>
>> --
>> Israel Meireles Chrisostomo
>>
>> --
>> Esta mensagem foi verificada pelo sistema de antivírus e
>> acredita-se estar livre de perigo.
>
>
> --
> Esta mensagem foi verificada pelo sistema de antivírus e
> acredita-se estar livre de perigo.

-- 
Esta mensagem foi verificada pelo sistema de antiv�rus e
 acredita-se estar livre de perigo.



[obm-l] Re: [obm-l] Re: [obm-l] Re: [obm-l] Teoria dos números

2020-03-17 Por tôpico Pedro José
Bom dia!
Se você considerar a expressão n(427-90n-70n^2+45n^3+18n^4)
D=|n(427-90n-70n^2+45n^3+18n^4)|
Por exemplo, n=1
D=330.
Agora se liberar n para variar D tende a oo.

Se n for raiz da expressão, também tende a oi, pois qualquer inteiro divide
0.


Em seg, 16 de mar de 2020 22:16, Israel Meireles Chrisostomo <
israelmchrisost...@gmail.com> escreveu:

> não entendi
>
> Em seg., 16 de mar. de 2020 às 22:01, Pedro José 
> escreveu:
>
>> Para um dado n é o módulo do valor da expressão.
>>
>> Em seg, 16 de mar de 2020 21:49, Pedro José 
>> escreveu:
>>
>>> Boa noite!
>>> O módulo dessa expressão tende a oo. Não existe máximo.
>>> Saudações,
>>> PJMS
>>>
>>> Em seg, 16 de mar de 2020 20:36, Israel Meireles Chrisostomo <
>>> israelmchrisost...@gmail.com> escreveu:
>>>
 Qual é o maior inteiro que divide  n (427 - 90n - 70n^2 + 45n^3 +
 18n^4)?

 --
 Israel Meireles Chrisostomo

 --
 Esta mensagem foi verificada pelo sistema de antivírus e
 acredita-se estar livre de perigo.
>>>
>>>
>> --
>> Esta mensagem foi verificada pelo sistema de antivírus e
>> acredita-se estar livre de perigo.
>
>
>
> --
> Israel Meireles Chrisostomo
>
> --
> Esta mensagem foi verificada pelo sistema de antivírus e
> acredita-se estar livre de perigo.

-- 
Esta mensagem foi verificada pelo sistema de antiv�rus e
 acredita-se estar livre de perigo.



[obm-l] Re: [obm-l] Re: [obm-l] Re: [obm-l] Ajuda com dízima

2020-03-09 Por tôpico Pedro José
Boa noite!
Errata da nota anterior independente de m e não de m, supondo (m,n)=1 e m/n
não inteiro.
Outro ponto não há necessidade a verificação de se o proposto vale para
quando n for múltiplo de 2 ou de 10, pois a ordem m mod n só existe se
(10,n)=1. Foi bobagem só ter aventado a possibilidade.
não coloquei como cheguei a conclusão de que era ordem 10 mod n, pois achei
bem intuitivo. Mas na hora que fui mostrar, achei complicado o que julgara
fácil. Mas quanto a isso estou seguro.
Para (n,m)=1 e (n,10)=1 e n/m não inteiro.
Se m>n pode-se representar por uma fração q j/n com q, j e n inteiros e
(j,n)=1 pois m=qn+j e se d<>1 divide n e j então d|m pois m é uma Z
combinação linear de j e n. Absurdo pois(m,n)=1 por hipótese.
Então sem perda de generalidade podemos só trabalhar para o caso m=2, está correta.

Saudações,
PJMS


Em dom, 8 de mar de 2020 16:09, Pedro José  escreveu:

> Boa tarde!
> Douglas,
> Não creio, no meu entendimento 3^2003 é o número de algarismos da dízima
> pois, é a ordem 10 módulo 3^2005.
> 1/3^2005 tem uma montoeira de algarismos zeros no início do período o que
> não acontece em 3^2005.
> O número de algarismos do período de uma dízima m/n, pelo menos quando n
> não  é múltiplo dos primos 2 e 5 é ord10mod n e independe de n. Nao
> verifiquei se vale sem a restriçao.
> Por exemplo o período de 1/7 é 142857 e ord 10 mod 7 = 6.
> Se aquele fosse o período da dízima bastaria fazer n =[log10 (3^2003)+1]
> onde colchetes representam parte inteira..
> Minha dúvida está na prova por absurdo, que ord 10 mod 3^n= 3^(n-2).
>
> Saudações,
> PJMS
>
>
>
> Em dom, 8 de mar de 2020 11:31, Prof. Douglas Oliveira <
> profdouglaso.del...@gmail.com> escreveu:
>
>> 3^2003 é o período certo??, o número de dígitos disso que seria a
>> pergunta.
>> 
>>
>> Douglas oliveira
>>
>> Em dom, 8 de mar de 2020 11:13, Prof. Douglas Oliveira <
>> profdouglaso.del...@gmail.com> escreveu:
>>
>>> Olá Pedro, primeiramente muito obrigado pela sua solução, eu dei uma
>>> olhada rápida e acredito estar correta. Estarei olhando com mais calma,
>>> assim que tiver um tempinho.
>>>
>>> Douglas Oliveira.
>>>
>>> Em dom, 8 de mar de 2020 11:05, Pedro José 
>>> escreveu:
>>>
 Bom dia!
 Não compreendi o porquê dessa questão ter sido vilipendiada. Não sou
 matemático, sou pitaqueiro, ouço falar em inteiros de Gauss vou atrás, de
 espaço fibrado idem, equações de Pell idem..., o que não consigo aprender
 fica para o futuro. Quando me aposentar  cursar uma faculdade de
 matemática. Portanto, nem tudo que resolvo me dá segurança. Reforço, alguém
 poderia me informar se está correto?
 Saudações,
 PJMS.

 Em ter, 3 de mar de 2020 12:03, Pedro José 
 escreveu:

> Boa tarde!
> Não me senti muito seguro na resposta. Está correto?
>
> Saudações,
> PJMS
>
> Em seg., 2 de mar. de 2020 às 23:27, Pedro José 
> escreveu:
>
>> Boa noite!
>> Creio ter conseguido.
>> Seja k o número de algarismos do período de 1/3^2005. Como (3,10)=1
>> então k é a ordem 10 mod 3^2005.
>> 3^(n-2)|| 3^(n-2); (|| significa divide exatamente) e 3^2||10-1 então
>> pelo lema de Hensel 3^n||10^(3^(n-2))-1 para n>=2.(i)
>> Então 10^(3^(n-2))= 1 mod 3^n logo ord 10 mod 3^n | 3^(n-2) Se
>> x<>3^(n-2) absurdo; pois, teria que ser 3^k com k> e por (i) 3^(k+2)||10^(3^k)-1 e k+2> ord 10 mod 3^2005 =3^2003
>> 3^2003 algarismos
>> Saudações,
>> PJMS
>>
>> Em sáb, 29 de fev de 2020 16:13, Pedro José 
>> escreveu:
>>
>>> Boa tarde!
>>> 3^2005 e não 10^2005.
>>>
>>> Em sex, 28 de fev de 2020 16:06, Pedro José 
>>> escreveu:
>>>
 Boa tarde!
 Questão complicada.
 Como (3^2005; 10) =1, o número de dígitos x deve ser a ordem de 10
 mod 10^2005. Portanto x | 2*3^2004.
 Se 10 fosse uma raiz primitiva de 3^2005 aí daria x=2.3^2004. Mas
 parece que não...
 Achar essa ordem é muito difícil, pelo menos para mim.
 O que achei empiricamente foi a conjectura: ord 10 mod 3^n =
 3^(n-2) para n>=2.
 Será que sai por indução, aí seriam 3^2003 algarismos. Caso a
 conjectura esteja correta.

 Saudações,
 PJMS

 Em qui., 20 de fev. de 2020 às 18:12, Prof. Douglas Oliveira <
 profdouglaso.del...@gmail.com> escreveu:

> Qual o número de dígitos do período de 1/(3^2005) ?
>
>
> Saudações
> Douglas Oliveira
>
> --
> Esta mensagem foi verificada pelo sistema de antivírus e
> acredita-se estar livre de perigo.


 --
 Esta mensagem foi verificada pelo sistema de antivírus e
 acredita-se estar livre de perigo.
>>>
>>>
>> --
>> Esta mensagem foi verificada pelo sistema de antivírus e
>> acredita-se estar livre de perigo.
>
>

-- 
Esta mensagem foi verificada pelo sistema de antiv�rus e
 

[obm-l] Re: [obm-l] Re: [obm-l] Re: [obm-l] Ajuda com dízima

2020-03-08 Por tôpico Pedro José
Boa tarde!
Douglas,
Não creio, no meu entendimento 3^2003 é o número de algarismos da dízima
pois, é a ordem 10 módulo 3^2005.
1/3^2005 tem uma montoeira de algarismos zeros no início do período o que
não acontece em 3^2005.
O número de algarismos do período de uma dízima m/n, pelo menos quando n
não  é múltiplo dos primos 2 e 5 é ord10mod n e independe de n. Nao
verifiquei se vale sem a restriçao.
Por exemplo o período de 1/7 é 142857 e ord 10 mod 7 = 6.
Se aquele fosse o período da dízima bastaria fazer n =[log10 (3^2003)+1]
onde colchetes representam parte inteira..
Minha dúvida está na prova por absurdo, que ord 10 mod 3^n= 3^(n-2).

Saudações,
PJMS



Em dom, 8 de mar de 2020 11:31, Prof. Douglas Oliveira <
profdouglaso.del...@gmail.com> escreveu:

> 3^2003 é o período certo??, o número de dígitos disso que seria a pergunta.
> 
>
> Douglas oliveira
>
> Em dom, 8 de mar de 2020 11:13, Prof. Douglas Oliveira <
> profdouglaso.del...@gmail.com> escreveu:
>
>> Olá Pedro, primeiramente muito obrigado pela sua solução, eu dei uma
>> olhada rápida e acredito estar correta. Estarei olhando com mais calma,
>> assim que tiver um tempinho.
>>
>> Douglas Oliveira.
>>
>> Em dom, 8 de mar de 2020 11:05, Pedro José 
>> escreveu:
>>
>>> Bom dia!
>>> Não compreendi o porquê dessa questão ter sido vilipendiada. Não sou
>>> matemático, sou pitaqueiro, ouço falar em inteiros de Gauss vou atrás, de
>>> espaço fibrado idem, equações de Pell idem..., o que não consigo aprender
>>> fica para o futuro. Quando me aposentar  cursar uma faculdade de
>>> matemática. Portanto, nem tudo que resolvo me dá segurança. Reforço, alguém
>>> poderia me informar se está correto?
>>> Saudações,
>>> PJMS.
>>>
>>> Em ter, 3 de mar de 2020 12:03, Pedro José 
>>> escreveu:
>>>
 Boa tarde!
 Não me senti muito seguro na resposta. Está correto?

 Saudações,
 PJMS

 Em seg., 2 de mar. de 2020 às 23:27, Pedro José 
 escreveu:

> Boa noite!
> Creio ter conseguido.
> Seja k o número de algarismos do período de 1/3^2005. Como (3,10)=1
> então k é a ordem 10 mod 3^2005.
> 3^(n-2)|| 3^(n-2); (|| significa divide exatamente) e 3^2||10-1 então
> pelo lema de Hensel 3^n||10^(3^(n-2))-1 para n>=2.(i)
> Então 10^(3^(n-2))= 1 mod 3^n logo ord 10 mod 3^n | 3^(n-2) Se
> x<>3^(n-2) absurdo; pois, teria que ser 3^k com k e por (i) 3^(k+2)||10^(3^k)-1 e k+2 ord 10 mod 3^2005 =3^2003
> 3^2003 algarismos
> Saudações,
> PJMS
>
> Em sáb, 29 de fev de 2020 16:13, Pedro José 
> escreveu:
>
>> Boa tarde!
>> 3^2005 e não 10^2005.
>>
>> Em sex, 28 de fev de 2020 16:06, Pedro José 
>> escreveu:
>>
>>> Boa tarde!
>>> Questão complicada.
>>> Como (3^2005; 10) =1, o número de dígitos x deve ser a ordem de 10
>>> mod 10^2005. Portanto x | 2*3^2004.
>>> Se 10 fosse uma raiz primitiva de 3^2005 aí daria x=2.3^2004. Mas
>>> parece que não...
>>> Achar essa ordem é muito difícil, pelo menos para mim.
>>> O que achei empiricamente foi a conjectura: ord 10 mod 3^n = 3^(n-2)
>>> para n>=2.
>>> Será que sai por indução, aí seriam 3^2003 algarismos. Caso a
>>> conjectura esteja correta.
>>>
>>> Saudações,
>>> PJMS
>>>
>>> Em qui., 20 de fev. de 2020 às 18:12, Prof. Douglas Oliveira <
>>> profdouglaso.del...@gmail.com> escreveu:
>>>
 Qual o número de dígitos do período de 1/(3^2005) ?


 Saudações
 Douglas Oliveira

 --
 Esta mensagem foi verificada pelo sistema de antivírus e
 acredita-se estar livre de perigo.
>>>
>>>
>>> --
>>> Esta mensagem foi verificada pelo sistema de antivírus e
>>> acredita-se estar livre de perigo.
>>
>>
> --
> Esta mensagem foi verificada pelo sistema de antivírus e
> acredita-se estar livre de perigo.

-- 
Esta mensagem foi verificada pelo sistema de antiv�rus e
 acredita-se estar livre de perigo.



[obm-l] Re: [obm-l] Re: [obm-l] Re: [obm-l] Re: [obm-l] Cálculo do Volume de um Sólido

2020-02-13 Por tôpico Luiz Antonio Rodrigues
Olá, Ralph!
Tudo bem?
Eu gostei muito da maneira que você indicou na segunda opção de resolução.
Olhamos o plano xy "por cima" e calculamos a integral "empilhando" os
trapézios em relação ao eixo z.
Muito obrigado pela resposta!
Abraços!
Luiz

Em qua, 12 de fev de 2020 2:27 PM, Ralph Teixeira 
escreveu:

> Vamos fixar um z (entre 0 e 2) para desenhar a seção horizontal. Como
> x+y=z^2 e x+y=2z são duas retas paralelas, a seção horizontal é um trapézio
> mais ou menos assim:
>
> |\
> | \
> |  \
> |   \
> |\
>  \\
>   \\
>
> As retas inclinadas são x+y=z^2, e x+y=2z. A reta vertical é o eixo y
> entre z^2 e z, e a horizontal é o eixo x entre z^2 e z.
>
> Então o problema na sua integral é que nem sempre o x vai de z^2-y até
> 2z-y, depende do valor de y! Trace uma vareta horizontal atravessando o
> trapézio:
>
> -- Na parte "de baixo" do trapézio, a vareta fura o trapézio nas duas
> retas inclinadas, ou seja, ali temos o x variando de z^2-y até 2z-y que nem
> você falou. Mas isso é só na parte de baixo, ou seja, apenas quando 0 -- Na parte de cima, a vareta fura na reta vertrical e na inclinada, isto
> é, 0
> Ou seja, para resolver isso com uma integral tripla dxdydz, vai ter que
> dividi-la em duas:
>
> Int (0 a 2) Int (0 a z^2) Int (z^2-y a 2z-y) dx dy dz +
> + Int (0 a 2) Int (z^2 a z) Int (0 a 2z-y) dx dy dz
>
> ---///---
>
> Outra opção (equivalente ao que o Buffara fez, mas subtraindo no plano
> antes de integrar para achar o volume): o trapézio pode ser pensado como a
> diferença de dois triângulos retângulos isósceles com vértice na origem --
> um grande tem cateto 2z, o pequeno z^2. Então a área de cada trapézio é:
>
> [(2z)^2 - (z^2)^2]/2 = 2z^2-z^4/2
>
> que, naturalmente, varia com z. Note que, como era de se esperar, a área
> dá 0 em z=0 e z=2.
>
> Agora é só integrar essa área para 0<=z<=2. Ou seja:
>
> Volume = 2.2^3/3 - 2^5/10 = 16/3 - 16/5 = 32/15.
>
> Abraço, Ralph.
>
>
> On Wed, Feb 12, 2020 at 9:29 AM Pedro José  wrote:
>
>> Bom dia!
>> Alguém poderia me ajudar e mostrar onde errei os limites? Resolvendo por
>> integral tripla, usando f(x,y,z)=1.
>>
>> Grato,
>> PJMS
>>
>> Em ter, 11 de fev de 2020 13:11, Pedro José 
>> escreveu:
>>
>>> Boa tarde!
>>>
>>> Resolvi por método numérico usando, pelo menos penso eu, os mesmos
>>> limites e encontrei 2,1329, muito próximo da resposta. Gostaria que alguém
>>> me ajudasse onde errei na integral tripla.
>>> Usei z^2-y e 2z-y como os limites para integral em dx. Em seguida, z^2 e
>>> 2z para dy e finalmente 0 e 2 para dz.
>>> Onde está o erro?
>>> Grato,
>>> PJMS
>>>
>>> Em ter, 11 de fev de 2020 12:49, Claudio Buffara <
>>> claudio.buff...@gmail.com> escreveu:
>>>
 O sólido é a região do 1o octante (todas as coordenadas positivas)
 compreendida entre os planos x-z e y-z, acima do plano z = (x+y)/2 e abaixo
 da z = raiz(x+y).
 A superfície e o plano se intersectam numa reta:
 raiz(x+y) = (x+y)/2 ==> x+y = (x+y)^2/4 ==> x+y = 4, contida no plano z
 = 2.

 Assim, o volume pode ser dado pela diferença entre duas integrais
 duplas, calculadas sobre o domínio D, no plano x-y, dado por x > 0, y > 0 e
 x+y = 4.
 Volume = Integral(D) raiz(x+y)*dA - Integral(D) (x+y)/2*dA.

 Usando coordenadas cartesianas, a primeira integral fica:
 Integral(x=0...4)Integral(y=0...4-x)*raiz(x+y)*dy*dx
 = Integral(0...4) (2/3)*(4^(3/2) - x^(3/2))*dx
 = Integral(0...4) (16/3 - (2/3)*x^(3/2))
 = 64/3 - (4/15)*4^(5/2)
 = 64/3 - 128/15
 = 64/5

 A segunda integral é:
 Integral(x=0...4)Integral(y=0...4-x) (x+y)/2*dy*dx
 = Integral(x=0...4) (1/2)*(x*(4-x) + (4-x)^2/2)*dx
 = Integral(0...4) (4 - x^2/4)*dx
 = 32/3

 Logo, o volume é 64/5 - 32/3 = 32/15  (se não errei nenhuma conta...)

 []s,
 Claudio.


 On Mon, Feb 3, 2020 at 8:55 PM Luiz Antonio Rodrigues <
 rodrigue...@gmail.com> wrote:

> Olá, pessoal!
> Tudo bem?
> Estou tentando resolver o seguinte problema:
>
> Ache o volume da região tridimensional definida por:
>
> z^2
> Sendo que:
> x>0 e y>0 e z>0
>
> Com o auxílio de um software eu consegui visualizar o sólido em
> questão.
> Eu calculei o volume do sólido girando em torno do eixo z e dividindo
> o resultado por 4.
> A resposta que eu obtive foi (16*pi)/15, que não está correta.
> Já refiz os cálculos muitas vezes e chego sempre na mesma resposta.
> Alguém pode me ajudar?
> Muito obrigado e um abraço!
>
> --
> Esta mensagem foi verificada pelo sistema de antivírus e
> acredita-se estar livre de perigo.


 --
 Esta mensagem foi verificada pelo sistema de antivírus e
 acredita-se estar livre de perigo.
>>>
>>>
>> --
>> Esta mensagem foi verificada pelo sistema de antivírus e
>> acredita-se estar livre de perigo.
>
>
> --
> Esta mensagem foi verificada pelo sistema de antivírus e
> acredita-se estar livre de 

[obm-l] Re: [obm-l] Re: [obm-l] Re: [obm-l] Cálculo do Volume de um Sólido

2020-02-12 Por tôpico Ralph Teixeira
Vamos fixar um z (entre 0 e 2) para desenhar a seção horizontal. Como
x+y=z^2 e x+y=2z são duas retas paralelas, a seção horizontal é um trapézio
mais ou menos assim:

|\
| \
|  \
|   \
|\
 \\
  \\

As retas inclinadas são x+y=z^2, e x+y=2z. A reta vertical é o eixo y entre
z^2 e z, e a horizontal é o eixo x entre z^2 e z.

Então o problema na sua integral é que nem sempre o x vai de z^2-y até
2z-y, depende do valor de y! Trace uma vareta horizontal atravessando o
trapézio:

-- Na parte "de baixo" do trapézio, a vareta fura o trapézio nas duas retas
inclinadas, ou seja, ali temos o x variando de z^2-y até 2z-y que nem você
falou. Mas isso é só na parte de baixo, ou seja, apenas quando 0 wrote:

> Bom dia!
> Alguém poderia me ajudar e mostrar onde errei os limites? Resolvendo por
> integral tripla, usando f(x,y,z)=1.
>
> Grato,
> PJMS
>
> Em ter, 11 de fev de 2020 13:11, Pedro José 
> escreveu:
>
>> Boa tarde!
>>
>> Resolvi por método numérico usando, pelo menos penso eu, os mesmos
>> limites e encontrei 2,1329, muito próximo da resposta. Gostaria que alguém
>> me ajudasse onde errei na integral tripla.
>> Usei z^2-y e 2z-y como os limites para integral em dx. Em seguida, z^2 e
>> 2z para dy e finalmente 0 e 2 para dz.
>> Onde está o erro?
>> Grato,
>> PJMS
>>
>> Em ter, 11 de fev de 2020 12:49, Claudio Buffara <
>> claudio.buff...@gmail.com> escreveu:
>>
>>> O sólido é a região do 1o octante (todas as coordenadas positivas)
>>> compreendida entre os planos x-z e y-z, acima do plano z = (x+y)/2 e abaixo
>>> da z = raiz(x+y).
>>> A superfície e o plano se intersectam numa reta:
>>> raiz(x+y) = (x+y)/2 ==> x+y = (x+y)^2/4 ==> x+y = 4, contida no plano z
>>> = 2.
>>>
>>> Assim, o volume pode ser dado pela diferença entre duas integrais
>>> duplas, calculadas sobre o domínio D, no plano x-y, dado por x > 0, y > 0 e
>>> x+y = 4.
>>> Volume = Integral(D) raiz(x+y)*dA - Integral(D) (x+y)/2*dA.
>>>
>>> Usando coordenadas cartesianas, a primeira integral fica:
>>> Integral(x=0...4)Integral(y=0...4-x)*raiz(x+y)*dy*dx
>>> = Integral(0...4) (2/3)*(4^(3/2) - x^(3/2))*dx
>>> = Integral(0...4) (16/3 - (2/3)*x^(3/2))
>>> = 64/3 - (4/15)*4^(5/2)
>>> = 64/3 - 128/15
>>> = 64/5
>>>
>>> A segunda integral é:
>>> Integral(x=0...4)Integral(y=0...4-x) (x+y)/2*dy*dx
>>> = Integral(x=0...4) (1/2)*(x*(4-x) + (4-x)^2/2)*dx
>>> = Integral(0...4) (4 - x^2/4)*dx
>>> = 32/3
>>>
>>> Logo, o volume é 64/5 - 32/3 = 32/15  (se não errei nenhuma conta...)
>>>
>>> []s,
>>> Claudio.
>>>
>>>
>>> On Mon, Feb 3, 2020 at 8:55 PM Luiz Antonio Rodrigues <
>>> rodrigue...@gmail.com> wrote:
>>>
 Olá, pessoal!
 Tudo bem?
 Estou tentando resolver o seguinte problema:

 Ache o volume da região tridimensional definida por:

 z^2>>>
 Sendo que:
 x>0 e y>0 e z>0

 Com o auxílio de um software eu consegui visualizar o sólido em questão.
 Eu calculei o volume do sólido girando em torno do eixo z e dividindo o
 resultado por 4.
 A resposta que eu obtive foi (16*pi)/15, que não está correta.
 Já refiz os cálculos muitas vezes e chego sempre na mesma resposta.
 Alguém pode me ajudar?
 Muito obrigado e um abraço!

 --
 Esta mensagem foi verificada pelo sistema de antivírus e
 acredita-se estar livre de perigo.
>>>
>>>
>>> --
>>> Esta mensagem foi verificada pelo sistema de antivírus e
>>> acredita-se estar livre de perigo.
>>
>>
> --
> Esta mensagem foi verificada pelo sistema de antivírus e
> acredita-se estar livre de perigo.

-- 
Esta mensagem foi verificada pelo sistema de antiv�rus e
 acredita-se estar livre de perigo.



[obm-l] Re: [obm-l] Re: [obm-l] Re: [obm-l] Re: [obm-l] Cálculo do Volume de um Sólido

2020-02-12 Por tôpico Rodrigo Ângelo
Luiz Antonio,

Creio que os livros de Cálculo cubram integrais iteradas. Eu estudei pelo
livro do James Stewart, mas dê uma olhada no livro que você já está
acostumado que deve ter esse conteúdo.

Mas, basicamente, quando você tem algo do tipo

[image: image.png]

Você primeiro integra f(x,y,z) de x0 até x1. O resultado vai ser uma função
g(y,z).
Então integra g(y,z) de y0 até y1, e o resultado vai ser uma h(z).
Então integra h(z) de z0 até z1 e obtém um número.

(ou seja, vai integrando de dentro pra fora)

Outra coisa: esse processo é o mesmo independente da quantidade de
integrais que você tiver: integral dupla, tripla, quadrupla, etc é sempre
de dentro pra fora.


Uma aplicação de integrais iteradas é justamente o cálculo de volume. Se
f(x,y,z) = 1, então a integral iterada em dxdydz vai ser o volume do sólido
definido pelos limites de integração (volume da região de integração).

Um detalhe é que os limites de integração podem ser em função das variáveis
mais externas (no caso da imagem, os limites de x podem depender de y e z,
e os limites de y podem depender de z, mas os limites de z devem ser fixos)


Atenciosamente,
Rodrigo de Castro Ângelo


Em qua., 12 de fev. de 2020 às 12:14, Luiz Antonio Rodrigues <
rodrigue...@gmail.com> escreveu:

> Olá, Claudio!
> Olá, Pedro!
> Tudo bem?
> Muito obrigado pela resposta!
> Eu estava tentando resolver o problema "empilhando" secções do plano xy,
> mas demorei para perceber que eram trapézios.
> Isso não deixa de ser uma forma de integração.
> Vocês podem me indicar um bom material para eu aprender a trabalhar com as
> integrais duplas e triplas?
> Percebi que pelas integrais duplas é bem mais fácil.
> Não tenho muito conhecimento para utilizar as integrais triplas.
> Abraços!
> Luiz
>
>
>
> Em qua, 12 de fev de 2020 9:29 AM, Pedro José 
> escreveu:
>
>> Bom dia!
>> Alguém poderia me ajudar e mostrar onde errei os limites? Resolvendo por
>> integral tripla, usando f(x,y,z)=1.
>>
>> Grato,
>> PJMS
>>
>> Em ter, 11 de fev de 2020 13:11, Pedro José 
>> escreveu:
>>
>>> Boa tarde!
>>>
>>> Resolvi por método numérico usando, pelo menos penso eu, os mesmos
>>> limites e encontrei 2,1329, muito próximo da resposta. Gostaria que alguém
>>> me ajudasse onde errei na integral tripla.
>>> Usei z^2-y e 2z-y como os limites para integral em dx. Em seguida, z^2 e
>>> 2z para dy e finalmente 0 e 2 para dz.
>>> Onde está o erro?
>>> Grato,
>>> PJMS
>>>
>>> Em ter, 11 de fev de 2020 12:49, Claudio Buffara <
>>> claudio.buff...@gmail.com> escreveu:
>>>
 O sólido é a região do 1o octante (todas as coordenadas positivas)
 compreendida entre os planos x-z e y-z, acima do plano z = (x+y)/2 e abaixo
 da z = raiz(x+y).
 A superfície e o plano se intersectam numa reta:
 raiz(x+y) = (x+y)/2 ==> x+y = (x+y)^2/4 ==> x+y = 4, contida no plano z
 = 2.

 Assim, o volume pode ser dado pela diferença entre duas integrais
 duplas, calculadas sobre o domínio D, no plano x-y, dado por x > 0, y > 0 e
 x+y = 4.
 Volume = Integral(D) raiz(x+y)*dA - Integral(D) (x+y)/2*dA.

 Usando coordenadas cartesianas, a primeira integral fica:
 Integral(x=0...4)Integral(y=0...4-x)*raiz(x+y)*dy*dx
 = Integral(0...4) (2/3)*(4^(3/2) - x^(3/2))*dx
 = Integral(0...4) (16/3 - (2/3)*x^(3/2))
 = 64/3 - (4/15)*4^(5/2)
 = 64/3 - 128/15
 = 64/5

 A segunda integral é:
 Integral(x=0...4)Integral(y=0...4-x) (x+y)/2*dy*dx
 = Integral(x=0...4) (1/2)*(x*(4-x) + (4-x)^2/2)*dx
 = Integral(0...4) (4 - x^2/4)*dx
 = 32/3

 Logo, o volume é 64/5 - 32/3 = 32/15  (se não errei nenhuma conta...)

 []s,
 Claudio.


 On Mon, Feb 3, 2020 at 8:55 PM Luiz Antonio Rodrigues <
 rodrigue...@gmail.com> wrote:

> Olá, pessoal!
> Tudo bem?
> Estou tentando resolver o seguinte problema:
>
> Ache o volume da região tridimensional definida por:
>
> z^2
> Sendo que:
> x>0 e y>0 e z>0
>
> Com o auxílio de um software eu consegui visualizar o sólido em
> questão.
> Eu calculei o volume do sólido girando em torno do eixo z e dividindo
> o resultado por 4.
> A resposta que eu obtive foi (16*pi)/15, que não está correta.
> Já refiz os cálculos muitas vezes e chego sempre na mesma resposta.
> Alguém pode me ajudar?
> Muito obrigado e um abraço!
>
> --
> Esta mensagem foi verificada pelo sistema de antivírus e
> acredita-se estar livre de perigo.


 --
 Esta mensagem foi verificada pelo sistema de antivírus e
 acredita-se estar livre de perigo.
>>>
>>>
>> --
>> Esta mensagem foi verificada pelo sistema de antivírus e
>> acredita-se estar livre de perigo.
>
>
> --
> Esta mensagem foi verificada pelo sistema de antivírus e
> acredita-se estar livre de perigo.

-- 
Esta mensagem foi verificada pelo sistema de antiv�rus e
 acredita-se estar livre de perigo.



[obm-l] Re: [obm-l] Re: [obm-l] Re: [obm-l] Cálculo do Volume de um Sólido

2020-02-12 Por tôpico Luiz Antonio Rodrigues
Olá, Claudio!
Olá, Pedro!
Tudo bem?
Muito obrigado pela resposta!
Eu estava tentando resolver o problema "empilhando" secções do plano xy,
mas demorei para perceber que eram trapézios.
Isso não deixa de ser uma forma de integração.
Vocês podem me indicar um bom material para eu aprender a trabalhar com as
integrais duplas e triplas?
Percebi que pelas integrais duplas é bem mais fácil.
Não tenho muito conhecimento para utilizar as integrais triplas.
Abraços!
Luiz



Em qua, 12 de fev de 2020 9:29 AM, Pedro José 
escreveu:

> Bom dia!
> Alguém poderia me ajudar e mostrar onde errei os limites? Resolvendo por
> integral tripla, usando f(x,y,z)=1.
>
> Grato,
> PJMS
>
> Em ter, 11 de fev de 2020 13:11, Pedro José 
> escreveu:
>
>> Boa tarde!
>>
>> Resolvi por método numérico usando, pelo menos penso eu, os mesmos
>> limites e encontrei 2,1329, muito próximo da resposta. Gostaria que alguém
>> me ajudasse onde errei na integral tripla.
>> Usei z^2-y e 2z-y como os limites para integral em dx. Em seguida, z^2 e
>> 2z para dy e finalmente 0 e 2 para dz.
>> Onde está o erro?
>> Grato,
>> PJMS
>>
>> Em ter, 11 de fev de 2020 12:49, Claudio Buffara <
>> claudio.buff...@gmail.com> escreveu:
>>
>>> O sólido é a região do 1o octante (todas as coordenadas positivas)
>>> compreendida entre os planos x-z e y-z, acima do plano z = (x+y)/2 e abaixo
>>> da z = raiz(x+y).
>>> A superfície e o plano se intersectam numa reta:
>>> raiz(x+y) = (x+y)/2 ==> x+y = (x+y)^2/4 ==> x+y = 4, contida no plano z
>>> = 2.
>>>
>>> Assim, o volume pode ser dado pela diferença entre duas integrais
>>> duplas, calculadas sobre o domínio D, no plano x-y, dado por x > 0, y > 0 e
>>> x+y = 4.
>>> Volume = Integral(D) raiz(x+y)*dA - Integral(D) (x+y)/2*dA.
>>>
>>> Usando coordenadas cartesianas, a primeira integral fica:
>>> Integral(x=0...4)Integral(y=0...4-x)*raiz(x+y)*dy*dx
>>> = Integral(0...4) (2/3)*(4^(3/2) - x^(3/2))*dx
>>> = Integral(0...4) (16/3 - (2/3)*x^(3/2))
>>> = 64/3 - (4/15)*4^(5/2)
>>> = 64/3 - 128/15
>>> = 64/5
>>>
>>> A segunda integral é:
>>> Integral(x=0...4)Integral(y=0...4-x) (x+y)/2*dy*dx
>>> = Integral(x=0...4) (1/2)*(x*(4-x) + (4-x)^2/2)*dx
>>> = Integral(0...4) (4 - x^2/4)*dx
>>> = 32/3
>>>
>>> Logo, o volume é 64/5 - 32/3 = 32/15  (se não errei nenhuma conta...)
>>>
>>> []s,
>>> Claudio.
>>>
>>>
>>> On Mon, Feb 3, 2020 at 8:55 PM Luiz Antonio Rodrigues <
>>> rodrigue...@gmail.com> wrote:
>>>
 Olá, pessoal!
 Tudo bem?
 Estou tentando resolver o seguinte problema:

 Ache o volume da região tridimensional definida por:

 z^2>>>
 Sendo que:
 x>0 e y>0 e z>0

 Com o auxílio de um software eu consegui visualizar o sólido em questão.
 Eu calculei o volume do sólido girando em torno do eixo z e dividindo o
 resultado por 4.
 A resposta que eu obtive foi (16*pi)/15, que não está correta.
 Já refiz os cálculos muitas vezes e chego sempre na mesma resposta.
 Alguém pode me ajudar?
 Muito obrigado e um abraço!

 --
 Esta mensagem foi verificada pelo sistema de antivírus e
 acredita-se estar livre de perigo.
>>>
>>>
>>> --
>>> Esta mensagem foi verificada pelo sistema de antivírus e
>>> acredita-se estar livre de perigo.
>>
>>
> --
> Esta mensagem foi verificada pelo sistema de antivírus e
> acredita-se estar livre de perigo.

-- 
Esta mensagem foi verificada pelo sistema de antiv�rus e
 acredita-se estar livre de perigo.



[obm-l] Re: [obm-l] Re: [obm-l] Re: [obm-l] Re: [obm-l] Re: [obm-l] Re: [obm-l] Re: [obm-l] Análise complexa - mostrar que f é sobrejetora

2020-02-11 Por tôpico Artur Costa Steiner
Em

>
> > Talvez seja uma tradução um tanto infeliz de entire function, do Inglês.
> No Inglês, entire em nada lembra integer.
>
> Em geral, eu chuto que um termo matemático usado antes do século XX
> não vem do inglês; a França e a Alemanha eram os grandes centros
> praticamente até a segunda guerra.
>
> > Mas será que é possível provar o teorema sem invocar Picard?
>
> Boa pergunta.  Será que o resultado é equivalente a Picard?  Acho
> pouco provável, mas talvez valha a pena tentar...
> --
> Bernardo Freitas Paulo da Costa
>
> A minha prova é:


Como a função identicamente nula é a única simultaneamente constante e
> ímpar, f não é constante. Logo, se for polinomial, pelo T. Fundamental da
> Álgebra, f é sobrejetora.
>

Suponhamos  agora que f não seja polinomial. Sendo inteira, se não for
sobrejetora, por Picard em seu conjunto imagem falta precisamente um
complexo w. Como f é ímpar e definida em todo o C, f(0) = 0, de modo que f
assume 0 e, portanto, w <> 0 e -w <> w. Logo, existe z com f(z) = -w. Como
f é ímpar e definida em -z, segue-se que f(-z) = -f(z) = w, contradizendo o
fato de que f não assume w. Logo, f é sobrejetora.

Artur

>
>
>
>

-- 
Esta mensagem foi verificada pelo sistema de antiv�rus e
 acredita-se estar livre de perigo.



[obm-l] Re: [obm-l] Re: [obm-l] Re: [obm-l] Re: [obm-l] Re: [obm-l] Re: [obm-l] Análise complexa - mostrar que f é sobrejetora

2020-02-11 Por tôpico Bernardo Freitas Paulo da Costa
On Mon, Feb 10, 2020 at 10:12 PM Artur Costa Steiner
 wrote:
>
> Em seg, 10 de fev de 2020 21:13, Pedro Angelo  
> escreveu:
>>
>> Aparentemente, errei hehe. Achei engraçada essa explicação: funções
>> holomorfas não-inteiras também têm "série de potências inteiras" em
>> torno de cada ponto. Por que só as inteiras levam o nome?
>
> Acho que inteira é no sentido de global, completa.

De fato.  A primeira evidência vem do próprio Picard: "Nous donnerons,
avec M. Weierstrass, le nom de fonctions entières d'une variable
complexe z aux fonctions uniformes et continues dans toute l'étendue
du plan; ce seront, par suite, des fonctions représentées par une
série, toujours convergente, ordonnée suivant les puissances
croissantes de la variable." (Mémoire sur les fonctions entières,
1880, justamente onde ele demonstra os "teoremas de Picard",
http://www.numdam.org/article/ASENS_1880_2_9__145_0.pdf).  Depois, tem
que ler em alemão alguém falando da história do Weierstrass (não achei
o livro / artigo onde ele usa esta notação pela primeira vez).  Eu
achei o Felix Klein, em
https://books.google.co.uk/books?id=XtunBgAAQBAJ=PA286=PA286=weierstrass+ganze+funktion=bl=E5OhNVM3WW=ACfU3U2ABjdB68sIwWisSpPCLJZf_8KdTQ=en=X=2ahUKEwjYteu1lsnnAhXioVwKHWlDAGYQ6AEwAHoECAcQAQ#v=onepage=weierstrass%20ganze%20funktion=false,
e de fato ele usa a mesma terminologia do Picard: ganzen Ebene (o
plano inteiro) e Potenzreihe (série de potências).

> Talvez seja uma tradução um tanto infeliz de entire function, do Inglês. No 
> Inglês, entire em nada lembra integer.

Em geral, eu chuto que um termo matemático usado antes do século XX
não vem do inglês; a França e a Alemanha eram os grandes centros
praticamente até a segunda guerra.

> Mas será que é possível provar o teorema sem invocar Picard?

Boa pergunta.  Será que o resultado é equivalente a Picard?  Acho
pouco provável, mas talvez valha a pena tentar...
--
Bernardo Freitas Paulo da Costa

-- 
Esta mensagem foi verificada pelo sistema de antiv�rus e
 acredita-se estar livre de perigo.


=
Instru��es para entrar na lista, sair da lista e usar a lista em
http://www.mat.puc-rio.br/~obmlistas/obm-l.html
=


[obm-l] Re: [obm-l] Re: [obm-l] Re: [obm-l] Re: [obm-l] Re: [obm-l] Análise complexa - mostrar que f é sobrejetora

2020-02-10 Por tôpico Artur Costa Steiner
Em seg, 10 de fev de 2020 21:13, Pedro Angelo 
escreveu:

> Aparentemente, errei hehe. Achei engraçada essa explicação: funções
> holomorfas não-inteiras também têm "série de potências inteiras" em
> torno de cada ponto. Por que só as inteiras levam o nome?
>

Acho que inteira é no sentido de global, completa. Talvez seja uma tradução
um tanto infeliz de entire function, do Inglês. No Inglês, entire em nada
lembra integer.

Mas será que é possível provar o teorema sem invocar Picard?

Artur

>
> Le lun. 10 févr. 2020 à 20:52, Bernardo Freitas Paulo da Costa
>  a écrit :
> >
> > On Mon, Feb 10, 2020 at 8:16 PM Artur Costa Steiner
> >  wrote:
> > > O adjetivo inteira, em análise complexa,  não tem nada a ver com o que
> ele sugere. Acho uma terminologia infeliz, mas é consagrada.
> >
> > Um chute: em francês, o termo "série inteira" (por oposição a série
> > fracionária) se refere às séries de potências (inteiras) da variável z
> > (por oposição às "séries de Puiseux" onde há expoentes fracionários).
> > E as funções inteiras têm expansão, convergente, como série de
> > potências (inteiras) da variável z, f(z) = \sum_{n=0}^\infty a_n z^n.
> >
> > Abraços,
> > --
> > Bernardo Freitas Paulo da Costa
> >
> > --
> > Esta mensagem foi verificada pelo sistema de antivírus e
> >  acredita-se estar livre de perigo.
> >
> >
> > =
> > Instru�ões para entrar na lista, sair da lista e usar a lista em
> > http://www.mat.puc-rio.br/~obmlistas/obm-l.html
> > =
>
> --
> Esta mensagem foi verificada pelo sistema de antivírus e
>  acredita-se estar livre de perigo.
>
>
> =
> Instru�ões para entrar na lista, sair da lista e usar a lista em
> http://www.mat.puc-rio.br/~obmlistas/obm-l.html
> =
>

-- 
Esta mensagem foi verificada pelo sistema de antiv�rus e
 acredita-se estar livre de perigo.



[obm-l] Re: [obm-l] Re: [obm-l] Re: [obm-l] Re: [obm-l] Análise complexa - mostrar que f é sobrejetora

2020-02-10 Por tôpico Pedro Angelo
Aparentemente, errei hehe. Achei engraçada essa explicação: funções
holomorfas não-inteiras também têm "série de potências inteiras" em
torno de cada ponto. Por que só as inteiras levam o nome?

Le lun. 10 févr. 2020 à 20:52, Bernardo Freitas Paulo da Costa
 a écrit :
>
> On Mon, Feb 10, 2020 at 8:16 PM Artur Costa Steiner
>  wrote:
> > O adjetivo inteira, em análise complexa,  não tem nada a ver com o que ele 
> > sugere. Acho uma terminologia infeliz, mas é consagrada.
>
> Um chute: em francês, o termo "série inteira" (por oposição a série
> fracionária) se refere às séries de potências (inteiras) da variável z
> (por oposição às "séries de Puiseux" onde há expoentes fracionários).
> E as funções inteiras têm expansão, convergente, como série de
> potências (inteiras) da variável z, f(z) = \sum_{n=0}^\infty a_n z^n.
>
> Abraços,
> --
> Bernardo Freitas Paulo da Costa
>
> --
> Esta mensagem foi verificada pelo sistema de antivírus e
>  acredita-se estar livre de perigo.
>
>
> =
> Instru�ões para entrar na lista, sair da lista e usar a lista em
> http://www.mat.puc-rio.br/~obmlistas/obm-l.html
> =

-- 
Esta mensagem foi verificada pelo sistema de antiv�rus e
 acredita-se estar livre de perigo.


=
Instru��es para entrar na lista, sair da lista e usar a lista em
http://www.mat.puc-rio.br/~obmlistas/obm-l.html
=


[obm-l] Re: [obm-l] Re: [obm-l] Re: [obm-l] Análise complexa - mostrar que f é sobrejetora

2020-02-10 Por tôpico Pedro Angelo
Eu gosto de pensar o "inteira" como significando que a série de
potências f(z) = a_0 + a_1 z + ... converge no plano *inteiro*.

Le lun. 10 févr. 2020 à 20:16, Artur Costa Steiner
 a écrit :
>
>
>
> Em seg, 10 de fev de 2020 17:28, Anderson Torres 
>  escreveu:
>>
>> Em dom., 9 de fev. de 2020 às 21:50, Artur Costa Steiner
>>  escreveu:
>> >
>> > Nunca vi este curioso fato ser citado em lugar nenhum.É fácil de provar 
>> > recorrendo-se ao teorema de Picard. Será que há uma prova simples (ou uma 
>> > qualquer) que não recorra a este teorema?
>> >
>> > Se a não identicamente nula f for inteira e ímpar, então f é sobrejetora.
>> >
>>
>> O que é função inteira?
>>
>> Se f é uma função definida em um aberto V do plano complexo C, dizemos que f 
>> é holomorfa em V se f for diferenciável em cada elemento de V.
>
>
> Se V = C, dizemos que f é inteira. Assim, uma função de C em C é inteira 
> se for diferenciável em todo o C. É holomorfa em C.
>
> O adjetivo inteira, em análise complexa,  não tem nada a ver com o que ele 
> sugere. Acho uma terminologia infeliz, mas é consagrada.
>
> Artur
>
>
>
>
> --
> Esta mensagem foi verificada pelo sistema de antivírus e
> acredita-se estar livre de perigo.

-- 
Esta mensagem foi verificada pelo sistema de antiv�rus e
 acredita-se estar livre de perigo.


=
Instru��es para entrar na lista, sair da lista e usar a lista em
http://www.mat.puc-rio.br/~obmlistas/obm-l.html
=


[obm-l] Re: [obm-l] Re: [obm-l] Re: [obm-l] Análise complexa - mostrar que f é sobrejetora

2020-02-10 Por tôpico Bernardo Freitas Paulo da Costa
On Mon, Feb 10, 2020 at 8:16 PM Artur Costa Steiner
 wrote:
> O adjetivo inteira, em análise complexa,  não tem nada a ver com o que ele 
> sugere. Acho uma terminologia infeliz, mas é consagrada.

Um chute: em francês, o termo "série inteira" (por oposição a série
fracionária) se refere às séries de potências (inteiras) da variável z
(por oposição às "séries de Puiseux" onde há expoentes fracionários).
E as funções inteiras têm expansão, convergente, como série de
potências (inteiras) da variável z, f(z) = \sum_{n=0}^\infty a_n z^n.

Abraços,
-- 
Bernardo Freitas Paulo da Costa

-- 
Esta mensagem foi verificada pelo sistema de antiv�rus e
 acredita-se estar livre de perigo.


=
Instru��es para entrar na lista, sair da lista e usar a lista em
http://www.mat.puc-rio.br/~obmlistas/obm-l.html
=


[obm-l] Re: [obm-l] Re: [obm-l] Re: [obm-l] Re: [obm-l] Re: [obm-l] Cálculo do Volume de um Sólido

2020-02-10 Por tôpico Luiz Antonio Rodrigues
Olá, Pedro!
Vou pensar na questão novamente e ver se consigo chegar na resposta.
Eu escreverei para dizer se consegui.
Muito obrigado!
Abraços!
Luiz


Em seg, 10 de fev de 2020 7:19 PM, Pedro José 
escreveu:

> Boa noite!
> Não sei onde errei está dando exatamente a metade 16/15.
> Saudações,
> PJMS
>
> Em seg, 10 de fev de 2020 15:46, Luiz Antonio Rodrigues <
> rodrigue...@gmail.com> escreveu:
>
>> Olá, Pedro!
>> Tudo bem?
>> Obrigado pela resposta!
>> A resposta realmente não tem pi: é 32/15.
>> Eu percebi ontem que o meu erro foi fazer uma rotação em torno do eixo z.
>> Se seccionarmos a figura no plano xy teremos um trapézio.
>> Vou pensar na sua sugestão e tentar fazer tudo de novo.
>> Muito obrigado!
>> Abraços!
>> Luiz
>>
>>
>>
>> Em seg, 10 de fev de 2020 1:38 PM, Pedro José 
>> escreveu:
>>
>>> Boa tarde!
>>> Como no caso você tem a resposta, facilitaria se a expusesse.
>>> Para evitar que postemos soluções erradas.
>>>
>>> Saudações,
>>> PJMS
>>>
>>> Em qui., 6 de fev. de 2020 às 07:41, Anderson Torres <
>>> torres.anderson...@gmail.com> escreveu:
>>>
 Em seg., 3 de fev. de 2020 às 20:55, Luiz Antonio Rodrigues
  escreveu:
 >
 > Olá, pessoal!
 > Tudo bem?
 > Estou tentando resolver o seguinte problema:
 >
 > Ache o volume da região tridimensional definida por:
 >
 > z^2>>> >
 > Sendo que:
 > x>0 e y>0 e z>0
 >
 > Com o auxílio de um software eu consegui visualizar o sólido em
 questão.
 > Eu calculei o volume do sólido girando em torno do eixo z e dividindo
 o resultado por 4.
 > A resposta que eu obtive foi (16*pi)/15, que não está correta.
 > Já refiz os cálculos muitas vezes e chego sempre na mesma resposta.
 > Alguém pode me ajudar?

 Tem como cê enviar as contas e o desenho que cê fez?

 > Muito obrigado e um abraço!
 >
 > --
 > Esta mensagem foi verificada pelo sistema de antivírus e
 > acredita-se estar livre de perigo.

 --
 Esta mensagem foi verificada pelo sistema de antivírus e
  acredita-se estar livre de perigo.



 =
 Instru�ões para entrar na lista, sair da lista e usar a lista em
 http://www.mat.puc-rio.br/~obmlistas/obm-l.html

 =

>>>
>>> --
>>> Esta mensagem foi verificada pelo sistema de antivírus e
>>> acredita-se estar livre de perigo.
>>
>>
>> --
>> Esta mensagem foi verificada pelo sistema de antivírus e
>> acredita-se estar livre de perigo.
>
>
> --
> Esta mensagem foi verificada pelo sistema de antivírus e
> acredita-se estar livre de perigo.

-- 
Esta mensagem foi verificada pelo sistema de antiv�rus e
 acredita-se estar livre de perigo.



[obm-l] Re: [obm-l] Re: [obm-l] Re: [obm-l] Re: [obm-l] Cálculo do Volume de um Sólido

2020-02-10 Por tôpico Pedro José
Boa noite!
Não sei onde errei está dando exatamente a metade 16/15.
Saudações,
PJMS

Em seg, 10 de fev de 2020 15:46, Luiz Antonio Rodrigues <
rodrigue...@gmail.com> escreveu:

> Olá, Pedro!
> Tudo bem?
> Obrigado pela resposta!
> A resposta realmente não tem pi: é 32/15.
> Eu percebi ontem que o meu erro foi fazer uma rotação em torno do eixo z.
> Se seccionarmos a figura no plano xy teremos um trapézio.
> Vou pensar na sua sugestão e tentar fazer tudo de novo.
> Muito obrigado!
> Abraços!
> Luiz
>
>
>
> Em seg, 10 de fev de 2020 1:38 PM, Pedro José 
> escreveu:
>
>> Boa tarde!
>> Como no caso você tem a resposta, facilitaria se a expusesse.
>> Para evitar que postemos soluções erradas.
>>
>> Saudações,
>> PJMS
>>
>> Em qui., 6 de fev. de 2020 às 07:41, Anderson Torres <
>> torres.anderson...@gmail.com> escreveu:
>>
>>> Em seg., 3 de fev. de 2020 às 20:55, Luiz Antonio Rodrigues
>>>  escreveu:
>>> >
>>> > Olá, pessoal!
>>> > Tudo bem?
>>> > Estou tentando resolver o seguinte problema:
>>> >
>>> > Ache o volume da região tridimensional definida por:
>>> >
>>> > z^2>> >
>>> > Sendo que:
>>> > x>0 e y>0 e z>0
>>> >
>>> > Com o auxílio de um software eu consegui visualizar o sólido em
>>> questão.
>>> > Eu calculei o volume do sólido girando em torno do eixo z e dividindo
>>> o resultado por 4.
>>> > A resposta que eu obtive foi (16*pi)/15, que não está correta.
>>> > Já refiz os cálculos muitas vezes e chego sempre na mesma resposta.
>>> > Alguém pode me ajudar?
>>>
>>> Tem como cê enviar as contas e o desenho que cê fez?
>>>
>>> > Muito obrigado e um abraço!
>>> >
>>> > --
>>> > Esta mensagem foi verificada pelo sistema de antivírus e
>>> > acredita-se estar livre de perigo.
>>>
>>> --
>>> Esta mensagem foi verificada pelo sistema de antivírus e
>>>  acredita-se estar livre de perigo.
>>>
>>>
>>> =
>>> Instru�ões para entrar na lista, sair da lista e usar a lista em
>>> http://www.mat.puc-rio.br/~obmlistas/obm-l.html
>>> =
>>>
>>
>> --
>> Esta mensagem foi verificada pelo sistema de antivírus e
>> acredita-se estar livre de perigo.
>
>
> --
> Esta mensagem foi verificada pelo sistema de antivírus e
> acredita-se estar livre de perigo.

-- 
Esta mensagem foi verificada pelo sistema de antiv�rus e
 acredita-se estar livre de perigo.



[obm-l] Re: [obm-l] Re: [obm-l] Re: [obm-l] Cálculo do Volume de um Sólido

2020-02-10 Por tôpico Luiz Antonio Rodrigues
Olá, Pedro!
Tudo bem?
Obrigado pela resposta!
A resposta realmente não tem pi: é 32/15.
Eu percebi ontem que o meu erro foi fazer uma rotação em torno do eixo z.
Se seccionarmos a figura no plano xy teremos um trapézio.
Vou pensar na sua sugestão e tentar fazer tudo de novo.
Muito obrigado!
Abraços!
Luiz



Em seg, 10 de fev de 2020 1:38 PM, Pedro José 
escreveu:

> Boa tarde!
> Como no caso você tem a resposta, facilitaria se a expusesse.
> Para evitar que postemos soluções erradas.
>
> Saudações,
> PJMS
>
> Em qui., 6 de fev. de 2020 às 07:41, Anderson Torres <
> torres.anderson...@gmail.com> escreveu:
>
>> Em seg., 3 de fev. de 2020 às 20:55, Luiz Antonio Rodrigues
>>  escreveu:
>> >
>> > Olá, pessoal!
>> > Tudo bem?
>> > Estou tentando resolver o seguinte problema:
>> >
>> > Ache o volume da região tridimensional definida por:
>> >
>> > z^2> >
>> > Sendo que:
>> > x>0 e y>0 e z>0
>> >
>> > Com o auxílio de um software eu consegui visualizar o sólido em questão.
>> > Eu calculei o volume do sólido girando em torno do eixo z e dividindo o
>> resultado por 4.
>> > A resposta que eu obtive foi (16*pi)/15, que não está correta.
>> > Já refiz os cálculos muitas vezes e chego sempre na mesma resposta.
>> > Alguém pode me ajudar?
>>
>> Tem como cê enviar as contas e o desenho que cê fez?
>>
>> > Muito obrigado e um abraço!
>> >
>> > --
>> > Esta mensagem foi verificada pelo sistema de antivírus e
>> > acredita-se estar livre de perigo.
>>
>> --
>> Esta mensagem foi verificada pelo sistema de antivírus e
>>  acredita-se estar livre de perigo.
>>
>>
>> =
>> Instru�ões para entrar na lista, sair da lista e usar a lista em
>> http://www.mat.puc-rio.br/~obmlistas/obm-l.html
>> =
>>
>
> --
> Esta mensagem foi verificada pelo sistema de antivírus e
> acredita-se estar livre de perigo.

-- 
Esta mensagem foi verificada pelo sistema de antiv�rus e
 acredita-se estar livre de perigo.



[obm-l] Re: [obm-l] Re: [obm-l] Re: [obm-l] Soma de raízes cúbicas de cossenos

2020-01-24 Por tôpico saulo nilson
NAO PRECISAVA ENCONTRAR COS5, COS 30=COS3*10, DAÍ ENCONTRA O COS10, DEPOIS
É SÓ SUBSTITUIR.

On Fri, Jan 24, 2020 at 10:23 AM Vanderlei Nemitz 
wrote:

> Como?
>
> Não entendi a ideia...
>
>
> Em sex, 24 de jan de 2020 02:37, saulo nilson 
> escreveu:
>
>> COS 15=COS 30/2
>> COS 15=COS(3*5)
>> DAÍ ENCONTRA O VALOR DE COS5 =COS10/2
>> DAÍ ENCONTRA O VALOR DE COS 10
>>
>> S= F(COS 10) QUE ENCONTRA O VALOR
>>
>> On Sun, Jan 19, 2020 at 8:41 AM Vanderlei Nemitz 
>> wrote:
>>
>>> Bom dia, pessoal!
>>>
>>> Pensei em resolver a seguinte questão associando cos 40°, cos 80° e cos
>>> 160° às raízes da equação cos(3x) = -1/2 e utilizando o arco triplo,
>>> recaindo em uma equação de grau 3. Porém, fica difícil determinar o produto
>>> de 2 em 2 das raízes cúbicas. Alguém conhece uma solução melhor?
>>> Muito obrigado!
>>>
>>> S = (cos 40°)^(1/3) + (cos 80°)^(1/3) +  (cos 160°)^(1/3)
>>>
>>> (Soma das raízes cúbicas de cos 40°, cos 80° e cos 160°)
>>>
>>> --
>>> Esta mensagem foi verificada pelo sistema de antivírus e
>>> acredita-se estar livre de perigo.
>>
>>
>> --
>> Esta mensagem foi verificada pelo sistema de antivírus e
>> acredita-se estar livre de perigo.
>
>
> --
> Esta mensagem foi verificada pelo sistema de antivírus e
> acredita-se estar livre de perigo.

-- 
Esta mensagem foi verificada pelo sistema de antiv�rus e
 acredita-se estar livre de perigo.



[obm-l] Re: [obm-l] Re: [obm-l] Re: [obm-l] Questão OBM - U

2020-01-23 Por tôpico Ralph Teixeira
Seja ABCD o quadrilatero convexo, e seja P o encontro das diagonais.

No triangulo APB, temos AP+PB>AB. Escreva as desigualdades analogas para os
triangulos BPC, CPD e DPA. Somando-as, voce vai obter que

2(AC+BD)>perimetro=8

Ou seja, o infimo tem que ser pelo menos 4.

Agora, para chegar no infimo, voce vai ter que "degenerar" os triangulos...
Entao considere um quadrilatero do tipo ABCB (ou seja, tome D=B), com,
digamos, AC=BC=2. Note que o perimetro eh 8, enquando AC=4 e BB=0, ou seja,
a soma das diagonais eh 4.

Mas alguns diriam que isso nao eh um quadrilatero convexo (bom, depende da
sua definicao de quadrilatero!)... Entao se "quadrilateros" nao incluem
casos degenrados, para fazer isso ficar rigoroso, voce teria que tomar um
quadrilatero convexo QUASE degenerado de perimetro 8 (um losango serve, a
conta fica facil), e mostrar que a soma das diagonais fica tao perto de 4
quanto voce queira.

Abraco, Ralph.

On Thu, Jan 23, 2020 at 7:24 AM gilberto azevedo 
wrote:

> Tentei com o retângulo e o quadrado, porém não obtive a resposta...  O
> gabarito é 4.
>
> Em sáb, 11 de jan de 2020 12:03, Bernardo Freitas Paulo da Costa <
> bernardo...@gmail.com> escreveu:
>
>> On Sat, Jan 11, 2020 at 11:24 AM gilberto azevedo 
>> wrote:
>> >
>> > Qual o ínfimo sobre todos os quadriláteros convexos com perímetro 8
>> da soma dos comprimentos de suas diagonais ?
>>
>> Quais são os quadriláteros que você tentaria?
>> --
>> Bernardo Freitas Paulo da Costa
>>
>> --
>> Esta mensagem foi verificada pelo sistema de antivírus e
>>  acredita-se estar livre de perigo.
>>
>>
>> =
>> Instruções para entrar na lista, sair da lista e usar a lista em
>> http://www.mat.puc-rio.br/~obmlistas/obm-l.html
>> =
>>
>
> --
> Esta mensagem foi verificada pelo sistema de antivírus e
> acredita-se estar livre de perigo.

-- 
Esta mensagem foi verificada pelo sistema de antiv�rus e
 acredita-se estar livre de perigo.



[obm-l] Re: [obm-l] Re: [obm-l] Re: [obm-l] Re: [obm-l] Re: [obm-l] Re: [obm-l] Questão OBM - U

2020-01-23 Por tôpico Esdras Muniz
É fácil ver que esse ínfimo tem que ser no mínimo 4, basta fazer
desigualdade triângulos com os triângulos que têm dois vértices comuns com
o quadrilátero e o terceiro sendo a interseção das diagonais. E por esse
argumento do Caio, vemos que é 4 mesmo.

Em qui, 23 de jan de 2020 08:59, Caio Costa  escreveu:

> Minimiza-se a soma das diagonais ao tomar-se um losango degenerado, com
> uma diagonal valendo 4 e outra valendo 0.
>
> Em qui, 23 de jan de 2020 08:34, gilberto azevedo 
> escreveu:
>
>> Pensei em minimizar √(a² + (4-a)²)
>> 4 - a, devido ao fato do perímetro ser 8.
>> No caso obtenho o mínimo sendo 2√2, quando o retângulo é um quadrado de
>> lado 2.
>> A soma das diagonais seria no caso 4√2, e não bate com o gabarito.
>>
>> Em qui, 23 de jan de 2020 08:20, Bernardo Freitas Paulo da Costa <
>> bernardo...@gmail.com> escreveu:
>>
>>> On Thu, Jan 23, 2020 at 7:24 AM gilberto azevedo 
>>> wrote:
>>> >> On Sat, Jan 11, 2020 at 11:24 AM gilberto azevedo <
>>> gil159...@gmail.com> wrote:
>>> >> >
>>> >> > Qual o ínfimo sobre todos os quadriláteros convexos com
>>> perímetro 8 da soma dos comprimentos de suas diagonais ?
>>> >
>>> > Tentei com o retângulo e o quadrado, porém não obtive a resposta...  O
>>> gabarito é 4.
>>>
>>> Qual (ou quais?) retângulo(s) você testou??  Que resposta você obteve?
>>> --
>>> Bernardo Freitas Paulo da Costa
>>>
>>> --
>>> Esta mensagem foi verificada pelo sistema de antivírus e
>>>  acredita-se estar livre de perigo.
>>>
>>>
>>> =
>>> Instru�ões para entrar na lista, sair da lista e usar a lista em
>>> http://www.mat.puc-rio.br/~obmlistas/obm-l.html
>>> =
>>>
>>
>> --
>> Esta mensagem foi verificada pelo sistema de antivírus e
>> acredita-se estar livre de perigo.
>
>
> --
> Esta mensagem foi verificada pelo sistema de antivírus e
> acredita-se estar livre de perigo.

-- 
Esta mensagem foi verificada pelo sistema de antiv�rus e
 acredita-se estar livre de perigo.



[obm-l] Re: [obm-l] Re: [obm-l] Re: [obm-l] Re: [obm-l] Re: [obm-l] Questão OBM - U

2020-01-23 Por tôpico Caio Costa
Minimiza-se a soma das diagonais ao tomar-se um losango degenerado, com uma
diagonal valendo 4 e outra valendo 0.

Em qui, 23 de jan de 2020 08:34, gilberto azevedo 
escreveu:

> Pensei em minimizar √(a² + (4-a)²)
> 4 - a, devido ao fato do perímetro ser 8.
> No caso obtenho o mínimo sendo 2√2, quando o retângulo é um quadrado de
> lado 2.
> A soma das diagonais seria no caso 4√2, e não bate com o gabarito.
>
> Em qui, 23 de jan de 2020 08:20, Bernardo Freitas Paulo da Costa <
> bernardo...@gmail.com> escreveu:
>
>> On Thu, Jan 23, 2020 at 7:24 AM gilberto azevedo 
>> wrote:
>> >> On Sat, Jan 11, 2020 at 11:24 AM gilberto azevedo 
>> wrote:
>> >> >
>> >> > Qual o ínfimo sobre todos os quadriláteros convexos com perímetro
>> 8 da soma dos comprimentos de suas diagonais ?
>> >
>> > Tentei com o retângulo e o quadrado, porém não obtive a resposta...  O
>> gabarito é 4.
>>
>> Qual (ou quais?) retângulo(s) você testou??  Que resposta você obteve?
>> --
>> Bernardo Freitas Paulo da Costa
>>
>> --
>> Esta mensagem foi verificada pelo sistema de antivírus e
>>  acredita-se estar livre de perigo.
>>
>>
>> =
>> Instru�ões para entrar na lista, sair da lista e usar a lista em
>> http://www.mat.puc-rio.br/~obmlistas/obm-l.html
>> =
>>
>
> --
> Esta mensagem foi verificada pelo sistema de antivírus e
> acredita-se estar livre de perigo.

-- 
Esta mensagem foi verificada pelo sistema de antiv�rus e
 acredita-se estar livre de perigo.



[obm-l] Re: [obm-l] Re: [obm-l] Re: [obm-l] Re: [obm-l] Questão OBM - U

2020-01-23 Por tôpico gilberto azevedo
Pensei em minimizar √(a² + (4-a)²)
4 - a, devido ao fato do perímetro ser 8.
No caso obtenho o mínimo sendo 2√2, quando o retângulo é um quadrado de
lado 2.
A soma das diagonais seria no caso 4√2, e não bate com o gabarito.

Em qui, 23 de jan de 2020 08:20, Bernardo Freitas Paulo da Costa <
bernardo...@gmail.com> escreveu:

> On Thu, Jan 23, 2020 at 7:24 AM gilberto azevedo 
> wrote:
> >> On Sat, Jan 11, 2020 at 11:24 AM gilberto azevedo 
> wrote:
> >> >
> >> > Qual o ínfimo sobre todos os quadriláteros convexos com perímetro
> 8 da soma dos comprimentos de suas diagonais ?
> >
> > Tentei com o retângulo e o quadrado, porém não obtive a resposta...  O
> gabarito é 4.
>
> Qual (ou quais?) retângulo(s) você testou??  Que resposta você obteve?
> --
> Bernardo Freitas Paulo da Costa
>
> --
> Esta mensagem foi verificada pelo sistema de antivírus e
>  acredita-se estar livre de perigo.
>
>
> =
> Instru�ões para entrar na lista, sair da lista e usar a lista em
> http://www.mat.puc-rio.br/~obmlistas/obm-l.html
> =
>

-- 
Esta mensagem foi verificada pelo sistema de antiv�rus e
 acredita-se estar livre de perigo.



[obm-l] Re: [obm-l] Re: [obm-l] Re: [obm-l] Questão OBM - U

2020-01-23 Por tôpico Bernardo Freitas Paulo da Costa
On Thu, Jan 23, 2020 at 7:24 AM gilberto azevedo  wrote:
>> On Sat, Jan 11, 2020 at 11:24 AM gilberto azevedo  
>> wrote:
>> >
>> > Qual o ínfimo sobre todos os quadriláteros convexos com perímetro 8 da 
>> > soma dos comprimentos de suas diagonais ?
>
> Tentei com o retângulo e o quadrado, porém não obtive a resposta...  O 
> gabarito é 4.

Qual (ou quais?) retângulo(s) você testou??  Que resposta você obteve?
-- 
Bernardo Freitas Paulo da Costa

-- 
Esta mensagem foi verificada pelo sistema de antiv�rus e
 acredita-se estar livre de perigo.


=
Instru��es para entrar na lista, sair da lista e usar a lista em
http://www.mat.puc-rio.br/~obmlistas/obm-l.html
=


[obm-l] Re: [obm-l] Re: [obm-l] Re: [obm-l] Re: [obm-l] Função Desconhecida

2020-01-15 Por tôpico Luiz Antonio Rodrigues
Olá, Anderson!
Bom dia!
Visitei o site que você indicou.
É muito bom!
Muito obrigado!
Abs

Em qua, 15 de jan de 2020 8:11 AM, Anderson Torres <
torres.anderson...@gmail.com> escreveu:

> Em sex., 20 de dez. de 2019 às 18:24, Luiz Antonio Rodrigues
>  escreveu:
> >
> > Olá, Esdras!
> > Eu de novo!
> > Você, ou alguém do grupo, pode me indicar um bom material relacionado às
> funções transcendentes?
> > É um assunto que me interessa bastante!
> > Abraços!
> > Luiz
> >
> > Em sex, 20 de dez de 2019 4:38 PM, Esdras Muniz <
> esdrasmunizm...@gmail.com> escreveu:
> >>
> >> Acho que essa função é trancendente.
>
> Pelo que eu sei, existe um algoritmo (sim, um programa de computador)
> que verifica se uma funçao é ou não passível de "integração
> bonitinha".
>
> Sempre que a dúvida bater, use esse site:
>
> https://www.integral-calculator.com/
>
> >>
> >> Em sex, 20 de dez de 2019 14:42, Luiz Antonio Rodrigues <
> rodrigue...@gmail.com> escreveu:
> >>>
> >>> Olá, pessoal!
> >>> Tudo bem?
> >>> Estou tentando, há alguns dias, resolver o seguinte problema:
> >>>
> >>> Preciso descobrir uma função f(x) cuja derivada é sen(x^3). Sabe-se
> que f(0)=2.
> >>>
> >>> Utilizei um software e mesmo assim não cheguei numa resposta para esta
> integral...
> >>> Alguém sabe se esta função é de algum tipo "especial"?
> >>> Muito obrigado!
> >>> Luiz
> >>>
> >>> --
> >>> Esta mensagem foi verificada pelo sistema de antivírus e
> >>> acredita-se estar livre de perigo.
> >>
> >>
> >> --
> >> Esta mensagem foi verificada pelo sistema de antivírus e
> >> acredita-se estar livre de perigo.
> >
> >
> > --
> > Esta mensagem foi verificada pelo sistema de antivírus e
> > acredita-se estar livre de perigo.
>
> --
> Esta mensagem foi verificada pelo sistema de antivírus e
>  acredita-se estar livre de perigo.
>
>
> =
> Instru�ões para entrar na lista, sair da lista e usar a lista em
> http://www.mat.puc-rio.br/~obmlistas/obm-l.html
> =
>

-- 
Esta mensagem foi verificada pelo sistema de antiv�rus e
 acredita-se estar livre de perigo.



[obm-l] Re: [obm-l] Re: [obm-l] Re: [obm-l] Função Desconhecida

2020-01-15 Por tôpico Anderson Torres
Em sex., 20 de dez. de 2019 às 18:24, Luiz Antonio Rodrigues
 escreveu:
>
> Olá, Esdras!
> Eu de novo!
> Você, ou alguém do grupo, pode me indicar um bom material relacionado às 
> funções transcendentes?
> É um assunto que me interessa bastante!
> Abraços!
> Luiz
>
> Em sex, 20 de dez de 2019 4:38 PM, Esdras Muniz  
> escreveu:
>>
>> Acho que essa função é trancendente.

Pelo que eu sei, existe um algoritmo (sim, um programa de computador)
que verifica se uma funçao é ou não passível de "integração
bonitinha".

Sempre que a dúvida bater, use esse site:

https://www.integral-calculator.com/

>>
>> Em sex, 20 de dez de 2019 14:42, Luiz Antonio Rodrigues 
>>  escreveu:
>>>
>>> Olá, pessoal!
>>> Tudo bem?
>>> Estou tentando, há alguns dias, resolver o seguinte problema:
>>>
>>> Preciso descobrir uma função f(x) cuja derivada é sen(x^3). Sabe-se que 
>>> f(0)=2.
>>>
>>> Utilizei um software e mesmo assim não cheguei numa resposta para esta 
>>> integral...
>>> Alguém sabe se esta função é de algum tipo "especial"?
>>> Muito obrigado!
>>> Luiz
>>>
>>> --
>>> Esta mensagem foi verificada pelo sistema de antivírus e
>>> acredita-se estar livre de perigo.
>>
>>
>> --
>> Esta mensagem foi verificada pelo sistema de antivírus e
>> acredita-se estar livre de perigo.
>
>
> --
> Esta mensagem foi verificada pelo sistema de antivírus e
> acredita-se estar livre de perigo.

-- 
Esta mensagem foi verificada pelo sistema de antiv�rus e
 acredita-se estar livre de perigo.


=
Instru��es para entrar na lista, sair da lista e usar a lista em
http://www.mat.puc-rio.br/~obmlistas/obm-l.html
=


[obm-l] Re: [obm-l] Re: [obm-l] Re: [obm-l] Máximo

2020-01-13 Por tôpico gilberto azevedo
Excelente, foi de grande ajuda. Muito obrigado !

Em dom, 12 de jan de 2020 20:42, Pedro Cardoso 
escreveu:

> O problema é resolvível no contexto do ensino médio porque uma das
> equações vão ser retas.
> Talvez tenha um jeito ainda mais fácil de resolver, mas essa foi a solução
> que encontrei:
>
> Por √x ser crescente, o máximo de
> √(16a² + 4b² - 16ab - 12a + 6b + 9)
> é a raíz do máximo de
> 16a² + 4b² - 16ab - 12a + 6b + 9.
> Seja
> 16a² + 4b² - 16ab - 12a + 6b + 9  = p
> Então
> (4a-2b)² - 3(4a-2b) + 9 = p
> (4a-2b)² - 3(4a-2b) + 9/4 = p - 27/4
> (4a-2b - 3/2)² = p -27/4
> O que dá duas retas:
> 2a-b = (√(p -27/4) + 3/2 )/2 e
> 2a-b = (-√(p -27/4) + 3/2 )/2
> E queremos encontrar o maior valor de p tal que pelo menos uma dessas
> retas toque a equação dada.
>
> Para simplificar, recomendo por
> k =  (√(p -27/4) + 3/2 )/2   e
> k' =   (-√(p -27/4) + 3/2 )/2
>
> Dessa forma, se encontrarmos o maior valor de k tal que 2a-b = k toca a
> equação dada, podemos encontrar também o maior valor de p tal que a
> primeira reta toca a equação dada.
> Analogamente, se encontrarmos o *menor* valor de *k'* tal que 2a - b = k'
> toca a equação, encontramos o maior valor de p tal que a *segunda* reta
> toca a equação dada. Aqui fica invertido pois tem um "-" na raíz com o p.
>
> A partir daqui já é possível resolver substituindo 2a-b = k restrição e
> encontrando o vértice de uma equação do segundo grau, mas para continuar,
> vou aplicar a substituição
>
> a=x+y
> b=x-y
>
> temos que
> 17(a²+b²) -30ab - 16 = 0 fica
> 34(x²+y²)-30(x²-y²)-16 = 0
> 4x² + 64y² - 16 = 0
> x² + 16y² - 4 = 0
>
> E 2a - b = k fica
> 2(x + y) - (x - y) = k
> x + 3y = k
> x = k - 3y
>
> Substituindo, temos
> (k-3y)² + 16y² - 4 = 0
> k² - 6ky + 25y² - 4 = 0
> 25y² - 6ky + k²-4 = 0
> Essa quadrática em y tem discriminante
> Δ = 36k² - 25(4k² - 16)
> Δ = 36k² - 100k² + 400
> Δ = 400 - 64k²
>
> Quando  Δ>0, a reta toca a equação em mais de 1 ponto. Quando  Δ=0, a reta
> é tangente à equação (isso ocorrerá quando k for máximo ou mínimo), e
> quando  Δ<0, a reta não toca a equação.
>
> Pondo  Δ=0, temos
> 25 - 4k² = 0
> k =  5/2  e
> k = -5/2
>
> Assim, encontramos o maior valor para k:  k = √(25/4)
> e o menor valor para k':  k' = -√(25/4). Isso nos dá as equações
>
> 5 = √(p -27/4) + 3/2e
> -5 = -√(p -27/4) + 3/2
> Que não são difíceis de resolver e dão, respectivamente
> p = 19   e
> p = 49
>
> Como disse no começo, o resultado é a raíz desse máximo, ou seja, √49 =
> *7*.
> Como o resultado foi muito bonito, suspeito que há um jeito muito mais
> simples de resolver, mas não o encontrei.
> A saber, esse máximo ocorre quando
> a = -1,9
> b = -1,3
>
> Espero que tenha sido útil
> Pedro Cardoso
>
> Em dom., 12 de jan. de 2020 às 18:09, Claudio Buffara <
> claudio.buff...@gmail.com> escreveu:
>
>> Oi, Gilberto:
>>
>> Que mal eu pergunte, de onde veio este problema?
>> E por que um aluno de EM teria que resolver um problema desses (e sem
>> usar cálculo)?
>>
>> []s,
>> Claudio.
>>
>>
>> On Sun, Jan 12, 2020 at 6:33 PM gilberto azevedo 
>> wrote:
>>
>>> Se a e b são números que satisfazem a equação :
>>> 17(a²+b²) - 30ab - 16 = 0
>>> Determinar o máximo de :
>>> √(16a² + 4b² - 16ab - 12a + 6b + 9)
>>> Sem utilizar lagrange e nada que envolva ensino superior . Não sei
>>> oq utilizar, se a sacada é enxergar uma fatoração... Enfim aceitando
>>> ideias.
>>>
>>> --
>>> Esta mensagem foi verificada pelo sistema de antivírus e
>>> acredita-se estar livre de perigo.
>>
>>
>> --
>> Esta mensagem foi verificada pelo sistema de antivírus e
>> acredita-se estar livre de perigo.
>
>
> --
> Esta mensagem foi verificada pelo sistema de antivírus e
> acredita-se estar livre de perigo.

-- 
Esta mensagem foi verificada pelo sistema de antiv�rus e
 acredita-se estar livre de perigo.



[obm-l] Re: [obm-l] Re: [obm-l] Re: [obm-l] Potência

2020-01-11 Por tôpico Ernesto Rodrigues
Temos 4^6 = 4096 = -4 (mod 100). 2^222 = 4^111 = 4^3*4^108 = 4^3*(-4)^18 =
4^3*4^18 = 4^3*(-4)^3 = -4^6 = -(-4) = 4 (mod 100)

Em sáb, 11 de jan de 2020 11:30, Vanderlei Nemitz 
escreveu:

> Está em um livro na parte de potenciação.
> Mas mesmo assim, como faria com essa ideia?
>
> Em sáb, 11 de jan de 2020 11:18, Esdras Muniz 
> escreveu:
>
>> Acho que é d) 04
>>
>> Em sáb, 11 de jan de 2020 11:01, Esdras Muniz 
>> escreveu:
>>
>>> Pode usar a função fi.
>>>
>>> Em sáb, 11 de jan de 2020 10:23, Vanderlei Nemitz 
>>> escreveu:
>>>
 Bom dia!
 Eu resolvi essa questão, mas creio que trabalhei demais!

 Alguém conhece um modo relativamente simples?

 Os dois últimos algarismos de 2^222 são:
 a) 84
 b) 24
 c) 64
 d) 04
 e) 44

 Muito obrigado!

 Vanderlei

 --
 Esta mensagem foi verificada pelo sistema de antivírus e
 acredita-se estar livre de perigo.
>>>
>>>
>> --
>> Esta mensagem foi verificada pelo sistema de antivírus e
>> acredita-se estar livre de perigo.
>
>
> --
> Esta mensagem foi verificada pelo sistema de antivírus e
> acredita-se estar livre de perigo.

-- 
Esta mensagem foi verificada pelo sistema de antiv�rus e
 acredita-se estar livre de perigo.



[obm-l] Re: [obm-l] Re: [obm-l] Re: [obm-l] Potência

2020-01-11 Por tôpico Pedro Cardoso
Vamos analisar 2^222 módulo 4 e módulo 25. Caso vc não seja familiar a
isso, dizer a = b (mod c) significa dizer que a e b tem o mesmo resto na
divisão por c.

2^222 = 0 (mod 4)

2^222 = 4^111 = (5-1)^111
Expandindo usando o binômio de newton, todos os termos são divisíveis por
25, exceto os dois últimos: (5^1)(1^110) - (5^0)(1^111) =
= 5 - 1 = 4
Ou seja, 2^222 = 4 (mod 25)

04 = 0 (mod 4) e 04 = 4 (mod 25)

Então os últimos dígitos são 04

-- 
Esta mensagem foi verificada pelo sistema de antiv�rus e
 acredita-se estar livre de perigo.



[obm-l] Re: [obm-l] Re: [obm-l] Re: [obm-l] Dúvida

2019-12-05 Por tôpico Bernardo Freitas Paulo da Costa
Oi Pedro e Pedro, e demais colegas da OBM-L

Eu também nunca lera a definição de elipses através da razão entre as
distâncias.  Achei interessante, porque talvez permita "interpolar"
entre elipses, parábolas e hipérboles.  Mas até hoje, todas as
definições que eu vira de elipses (inclusive a da soma das distâncias)
incluíram círculos.  Mas, como talvez tenha desejado indicar o Pedro
Fonini, o importante é *para que serve a definição*.  No caso das
elipses, é muitas vezes importante incluir os círculos com elas, por
exemplo para o teorema de álgebra linear que ele citou.  Talvez o caso
da definição por razão das distâncias indique um outro caminho, mas aí
minha impressão é que o caso seria que a reta diretriz está no
infinito para os círculos (o que, mais uma vez, reforça a unidade das
cônicas no plano projetivo).

Acho que o mais comum, *hoje em dia*, é definir elipses de forma a
incluir os círculos.  Talvez isto invalide a definição que você deu
via razões, que passa a ser um teorema apenas para as elipses com dois
focos distintos.  Mas, por outro lado, permite generalizar de forma
mais natural outros teoremas para os quais a inclusão dos círculos
como elipses simplifique o enunciado.  Talvez você prefira a definição
por razões, mas voltando às origens das cônicas, onde a "classe" é
determinada pela posição relativa do plano secante com relação ao cone
gerador, acredito que a inclusão dos círculos junto com as elipses
seja totalmente razoável.

Cônicas suaves me parecem um assunto avançado; esta terminologia mesmo
já faz pensar em funções (infinitamente) diferenciáveis, etc, típicas
o ensino superior.  Do ponto de vista da geometria algébrica, uma
cõnica é definida como zeros de um polinômio P(x,y) de grau dois,
então esta "definição" não pode servir para separar quem seja suave e
quem não seja.  Se há uma diferença entre um círculo e duas retas que
se intersectam, não é pela regularidade da função que os define
implicitamente: num caso é x^2 + y^2 = 1, no outro, xy = 0.  O que
acaba servindo é a definição de "variedade suave".  E daí eu estaria
puxando mais ainda para temas universitários...

On Wed, Dec 4, 2019 at 9:33 PM Pedro José  wrote:
>
> Boa noite!
> As retas são cônicas degeneradas. Mas são cônicas.
> Definição de cônica :   Dada duas retas g,l concorrentes (cuja interseção é 
> {V} no |R3 que não sejam perpendiculares e um plano Pi. A interseção desse 
> plano com o cone K, reto de vértice V e eixo l , obtido pela rotação da reta 
> g ao redor do ponto V é uma cônica. Podemos ter uma reta, duas retas ou um 
> ponto como cônicas degeneradas.
> Você poderia até ter mencionado o conjunto vazio que não é uma cônica. 
> x^2+y^2=-1.
> Mas na verdade, eu não me expressei com rigor, o que queria dizer é que se 
> escrevermos a função quadrática F(x,y)= 0, que represente a cônica 
> (degenerada ou não) F(x,y) é suave? Ou as cônicas suaves devem ser não 
> degeneradas apenas?
> Outrossim, discordo do seu argumento "...geralmente é mais útil que as 
> definições dos objetos importantes não excluam os casos particulares.."
> Geralmente não é o balizador e sim a definição.
> 1 não é primo. Pois define-se que um primo deve ter dois divisores positivos 
> e 1 só possui um. Poderia argumentar, na sua linha, os dois divisores 
> coincidentes (os que afirmam é divisível por si e pela unidade)
> O quadrado por definição está claro que é retângulo.
> A definição da elipse é de que a soma das distâncias a dois pontos fixos (e 
> não um) é constante. Aí tem a forçação de se considerar dois como um só. Não 
> existe dois pontos coincidentes. Se são dois são distintos. Podemos 
> representar algo de várias maneiras mas se são iguais é só um, representado 
> de várias maneiras. Qual o cardinal do conjunto de focos de uma elipse, no 
> caso de você aceitar a elipse com um único foco?
> Como é a prova que só existe um vazio. Por hipótese há mais de um vazio, 
> vazio1 e vazio2 e no fim chega-se a conclusão que vazio1 = vazio2 e portanto 
> absurdo.Ora, podemos ter vazios coincidentes.
> Amigo, você afirma: "Nunca vi ninguém definir elipse de uma forma que exclua 
> os círculos."
> Você nem se deu ao trabalho de ler a minha nota, antes de comentar, ou então 
> me corrija se o círculo atende à:
> Lugar geométrico do plano em que a razão entre a distância de um ponto ao 
> foco direito e a distância entre esse ponto e uma reta (diretriz direita) é 
> constante e menor que 1 e igual a excentricidade da cônica.
> Como a excentricidade da circunferência é zero, teríamos que ter um ponto 
> fixo em que a distância de cada ponto da circunferência até esse ponto fosse 
> zero. E se na definição tem foco direito está implícito que há um esquerdo. 
> Vale a definição para foco esquerdo. Só atenderia se considerarmos o ponto 
> como uma circunferência de raio zero. E só para esse caso e ainda aceitarmos 
> que quando há só um foco ele tanto é direito quanto esquerdo. Grato pelos 
> comentários. Mas as dúvida persistem.
>
> 

[obm-l] Re: [obm-l] Re: [obm-l] Re: [obm-l] Triângulos.

2019-11-24 Por tôpico Esdras Muniz
Verdade, não tinha percebido.

Em dom, 24 de nov de 2019 14:17, Pedro José  escreveu:

> Boa tarde!
> Esdras,
> Não seria z>=3.
> 3, 2, 2 dá um obtusângulo.
>
> Saudações,
> PJMS
>
> Em sáb, 23 de nov de 2019 01:52, Esdras Muniz 
> escreveu:
>
>> Acho que a questão pressupõe que os lados devem ser inteiros. Daí se os
>> lados são x, y e z, com x<=y> x^2+y^2x^2+y^2 e
>> z> Daí, z é ao menos 4, vc sai contando caso a caso...
>>
>> Em sex, 22 de nov de 2019 20:39, Claudio Buffara <
>> claudio.buff...@gmail.com> escreveu:
>>
>>> Do jeito que está escrito, uma infinidade.
>>>
>>> Enviado do meu iPhone
>>>
>>> > Em 22 de nov de 2019, à(s) 19:18, Guilherme Abbehusen <
>>> gui.abbehuse...@gmail.com> escreveu:
>>> >
>>> > 
>>> > Olá,Â
>>> >   Preciso de ajuda com a seguinte questão:Â
>>> >
>>> > Tendo em vista a leis dos Cossenos, marque a quantidade de triângulos
>>> obtusângulos que podemos formar com lados menores do que 7.
>>> > a) 6
>>> > b) 7
>>> > c) 8Â
>>> > d) 9
>>> > e) 10
>>> >
>>> > --
>>> > Esta mensagem foi verificada pelo sistema de antivírus e
>>> > acredita-se estar livre de perigo.
>>>
>>> --
>>> Esta mensagem foi verificada pelo sistema de antivírus e
>>>  acredita-se estar livre de perigo.
>>>
>>>
>>> =
>>> Instru�ões para entrar na lista, sair da lista e usar a lista em
>>> http://www.mat.puc-rio.br/~obmlistas/obm-l.html
>>> =
>>>
>>
>> --
>> Esta mensagem foi verificada pelo sistema de antivírus e
>> acredita-se estar livre de perigo.
>
>
> --
> Esta mensagem foi verificada pelo sistema de antivírus e
> acredita-se estar livre de perigo.

-- 
Esta mensagem foi verificada pelo sistema de antiv�rus e
 acredita-se estar livre de perigo.



[obm-l] Re: [obm-l] Re: [obm-l] Re: [obm-l] Qual o 2020º termo da sequência abaixo ?

2019-11-18 Por tôpico Jamil Silva
Correto: a(2020) = 1718.11.2019, 14:47, "Esdras Muniz" :Eu resolvi fazendo um programa, e deu 17. Mas a ideia é essa mesmo do mod 41. Se aparecerem dois números seguidos que já apareceram antes, a sequência começar a se repetir, tipo 1, 2,..., 1, 2,... E isso com certeza vai ocorrer, pois só há 41×40 duplas de números seguidos possíveis, considerando a ordem. Pelo que vi, a sequência começar a repetir a partir do termo 60.Em dom, 17 de nov de 2019 21:54, Claudio Buffara  escreveu:Pela definição da sequência.
Quando a a(n) + a(n+1) > 40, a(n+2) = resto da divisão de a(n) + a(n+1) por 40, sendo que neste caso os restos vão de 1 a 40 (ao invés de 0 a 39).

Enviado do meu iPhone

> Em 17 de nov de 2019, à(s) 18:59, Jamil Silva  escreveu:
> 
> Por que mod40 ?
> 
> 17.11.2019, 14:36, "Claudio Buffara" :
>> Me parece que basta calcular o 2020o termo sem a restrição de ser mod 40 (é uma sequência de Fibonacci começando por 5 e 2) e depois ver quanto e’ a(2020) mod 40, sendo que na redução mod 40, ao invés dos restos serem 0, 1, ..., 39, eles serão 1, 2, ..., 40.
>> 
>> Enviado do meu iPhone
>> 
>>> Â Em 17 de nov de 2019, Ã (s) 08:15, Jamil Silva  escreveu:
>>> 
>>>  5, 2, 7, 9, 16, 25, 1, 26, 27, 13, 40, 13, 13, 26, 39, 25, 24,...
>>> 
>>>  Sua lei de formação é a seguinte:
>>> 
>>> Â a(1) = 5
>>> Â a(2) = 2
>>>  a(n+2) = [a(n+1)+a(n)], sse [a(n+1) + a(n)] ≤ 40
>>> Â a(n+2) = [a(n+1)+a(n)] - 40, sse [a(n+1) + a(n)] > 40
>>> 
>>> Â --
>>>  Esta mensagem foi verificada pelo sistema de antivírus e
>>> Â acredita-se estar livre de perigo.
>>> 
>>> Â =
>>>  Instruções para entrar na lista, sair da lista e usar a lista em
>>> Â http://www.mat.puc-rio.br/~obmlistas/obm-l.html
>>> Â =
>> 
>> --
>> Esta mensagem foi verificada pelo sistema de antiv?rus e
>> Â acredita-se estar livre de perigo.
>> 
>> =
>> Instru??es para entrar na lista, sair da lista e usar a lista em
>> http://www.mat.puc-rio.br/~obmlistas/obm-l.html
>> =
> 
> -- 
> Esta mensagem foi verificada pelo sistema de antivírus e
> acredita-se estar livre de perigo.
> 
> =
> Instruções para entrar na lista, sair da lista e usar a lista em
> http://www.mat.puc-rio.br/~obmlistas/obm-l.html
> =

-- 
Esta mensagem foi verificada pelo sistema de antivírus e
 acredita-se estar livre de perigo.


=
Instru�ões para entrar na lista, sair da lista e usar a lista em
http://www.mat.puc-rio.br/~obmlistas/obm-l.html
=

--
Esta mensagem foi verificada pelo sistema de antiv?rus e 
 acredita-se estar livre de perigo.

--
Esta mensagem foi verificada pelo sistema de antivírus e 
 acredita-se estar livre de perigo.




[obm-l] Re: [obm-l] Re: [obm-l] Re: [obm-l] Re: [obm-l] Re: [obm-l] Re: [obm-l] ordem lexicográfica dos numerais

2019-11-12 Por tôpico lumpa lumpa
Sim, mas naquele problema eu ERRONEAMENTE falei em ordem lexicográfica, mas
quando descrevi a sequencia postei outra ordem em que
as sequencias de menor quantidade de letras sempre precedem qualquer outra
cuja quantidade de letras é maior, por isso ao invés de fazer assim:
a, aa, ac, ae, ai, am, at, c, ca, cc, ce, ci, cm, ct, e, ea, ec, ee, ei,
em, et, i, ia, ic, ie, ii, im, it, m, ma, mc, me, mi, mm, mt, t, ta, tc,
te, ti, tm ,tt

fiz, equivocadamente, assim:


a, c, e, i, m, t, aa, ac, ae, ai, am, at, ca, cc, ce, ci, cm, ct, ea, ec,
ee, ei, em, et, ia, ic, ie, ii, im, it, ma, mc, me, mi, mt, ta, tc, te. ti
,tm, tt, aaa, aac, aae, aai, aam, aat, etc































Em ter., 12 de nov. de 2019 às 21:45, Pedro José 
escreveu:

> É curioso, pois, no problema que você postou com letras às vinha depois de
> t.
>
> Saudações,
> PJMS
>
> Em ter, 12 de nov de 2019 21:22, lumpa lumpa <1vp4l...@gmail.com>
> escreveu:
>
>>
>>
>>
>>
>>
>>
>>
>>
>> Boa noite !
>>
>> Não. 01 vem depois de 00 que é o sucessor de 0, assim:
>>
>> 0, 00, 01, 02, 03, 04, 05, 06, 07, 08, 09, 1, 10, 11, 12, 13, 14, 15, 16,
>> 17, 18, 19, 2, 20, 21, ... etc.
>>
>> É óbvio que a sequencia acima mostra apenas as combinações de no máximo
>> dois algarismos, mas sabemos que há outros infinitos termos entre eles.
>> Mostrando as combinações de três algarismos, seria assim:
>>
>> 0, 00, 000, 001, 002, 003, 004, 00, 006, 007, 008, 009, 01, 010, 011,
>> 012, 013, 014, 01, 016, 017, 018, 019, 02, ., 03, 04,
>> ,,..., 09, 090, 091, 092, 093, 094, 095, 096, 097, 098, 099, 1
>>
>>
>>
>>
>>
>>
>>
>>
>>
>>
>>
>>
>>
>>
>>
>>
>>
>>
>>
>> Em ter., 12 de nov. de 2019 às 19:49, Pedro José 
>> escreveu:
>>
>>> Boa noite!
>>> Mas 1 ocorre antes de 01, não. Tenho que esgotar primeiro as de uma
>>> posição, para depois usar as de duas se não eu não andaria nunca. 0, 00,
>>> 000,  
>>> Só confirme que penso uma solução, caso consiga.
>>>
>>> Saudações,
>>> PJMS
>>>
>>> Em ter., 12 de nov. de 2019 às 18:15, lumpa lumpa <1vp4l...@gmail.com>
>>> escreveu:
>>>
 Boa tarde, Pedro.

 Por menor posição, estou considerando aquela que conta a partir de zero
 todas as sequencias de no máximo quatro algarismos até  2019.

 0, 00, 000,  são todos sequencias diferentes. Pense nos algarismos
 como símbolos quaisquer, como se fossem letras e as combinações palavras na
 ordem alfabética

 [0], 00, 01, 02, 03, 04, 05, 06, 07, 08, 09, [1], 10, 11, 12, 13, 14,
 15, 16, 17, 18, 19, [2], 20, 21, 22, 23 etc

 A linha acima é só um pequeno exemplo dos primeiros termos da sequencia
 com termos de no máximo dois algarismos..

 Em ter., 12 de nov. de 2019 às 17:45, Pedro José 
 escreveu:

> Boa tarde!
> Vai depender do conceito!
> 0 e 00 são contados como um só ou duas vezes?
> Não entendi a menor posição. No meu entender há uma bijeção entre a
> posição e o número.
> A menos, que se contem 02019 e 2019 como o mesmo número, porém como
> "palavras diferentes.
>
> Saudações,
> PJMS
>
>
> Em ter., 12 de nov. de 2019 às 15:31, lumpa lumpa <1vp4l...@gmail.com>
> escreveu:
>
>> Qual a menor posição do número 2019 na ordem lexicográfica de todas
>> as sequências possíveis dos algarismos indo-arábicos: {0, 1, 2, 3, 4, 5, 
>> 6,
>> 7, 8, 9} ?
>>
>> --
>> Esta mensagem foi verificada pelo sistema de antivírus e
>> acredita-se estar livre de perigo.
>
>
> --
> Esta mensagem foi verificada pelo sistema de antivírus e
> acredita-se estar livre de perigo.


 --
 Esta mensagem foi verificada pelo sistema de antivírus e
 acredita-se estar livre de perigo.
>>>
>>>
>>> --
>>> Esta mensagem foi verificada pelo sistema de antivírus e
>>> acredita-se estar livre de perigo.
>>
>>
>> --
>> Esta mensagem foi verificada pelo sistema de antivírus e
>> acredita-se estar livre de perigo.
>
>
> --
> Esta mensagem foi verificada pelo sistema de antivírus e
> acredita-se estar livre de perigo.

-- 
Esta mensagem foi verificada pelo sistema de antiv�rus e
 acredita-se estar livre de perigo.



  1   2   3   4   5   6   7   8   9   10   >